[obm-l] Re: [obm-l] Re: [obm-l] Re: [obm-l] Re: [obm-l] Combinatória

2024-03-16 Por tôpico Anderson Torres
Em qua., 13 de mar. de 2024 às 13:07, Claudio Buffara
 escreveu:
>
> Mas este caso tem 7 pessoas. E o enunciado fala em 3 A e 3 C.
>
> On Wed, Mar 13, 2024 at 9:28 AM Pedro Júnior  
> wrote:
>>
>> Eu pensei sim, mas e os casos do tipo ACCACAC. Esse caso não entra na conta 
>> 6! - 2* 3!* 3!.
>>
>> Em qua., 13 de mar. de 2024 às 09:09, Claudio Buffara 
>>  escreveu:
>>>
>>> Pense no oposto: de quantas maneiras as crianças e adultos podem se sentar 
>>> separados uns dos outros.
>>>
>>> On Wed, Mar 13, 2024 at 8:39 AM Pedro Júnior  
>>> wrote:

 Olá pessoal, bom dia.
 Alguém poderia me ajudar nesse problema?

 Seis poltronas enfileiradas em um cinema e entram 3 adultos e 3 crianças. 
 De quantas maneiras podem sentar-se 2 crianças juntas e dois adultos 
 juntos?


Eu pensei numa maneira muito feia: enumeração na raça!

AAA CCC +

AACACC +
AACCAC +
AACCCA +

ACAACC +
ACACAC -
ACACCA -

CAAACC +
CAACAC -
CAACCC +

ACCAAC +
ACCACA -
ACCCAA +

CACAAC -
CACACA -
CACCAA +

CCAAAC +
CCAACA +
CCACAA +

CCC AAA +

20 possibilidades. Apenas 14 servem.
Cada uma nos permite permutar as crianças seis vezes, os adultos outras seis.
6x6x14, que dá meh.


 Desde já fico grato!

 --
 Esta mensagem foi verificada pelo sistema de antivírus e
 acredita-se estar livre de perigo.
>>>
>>>
>>> --
>>> Esta mensagem foi verificada pelo sistema de antivírus e
>>> acredita-se estar livre de perigo.
>>
>>
>>
>> --
>>
>> Pedro Jerônimo S. de O. Júnior
>>
>> Professor de Matemática
>>
>> Geo João Pessoa – PB
>>
>>
>> --
>> Esta mensagem foi verificada pelo sistema de antivírus e
>> acredita-se estar livre de perigo.
>
>
> --
> Esta mensagem foi verificada pelo sistema de antivírus e
> acredita-se estar livre de perigo.

-- 
Esta mensagem foi verificada pelo sistema de antiv�rus e
 acredita-se estar livre de perigo.


=
Instru��es para entrar na lista, sair da lista e usar a lista em
http://www.mat.puc-rio.br/~obmlistas/obm-l.html
=


[obm-l] Re: [obm-l] Re: [obm-l] Re: [obm-l] Combinatória

2024-03-13 Por tôpico Claudio Buffara
Mas este caso tem 7 pessoas. E o enunciado fala em 3 A e 3 C.

On Wed, Mar 13, 2024 at 9:28 AM Pedro Júnior 
wrote:

> Eu pensei sim, mas e os casos do tipo ACCACAC. Esse caso não entra na
> conta 6! - 2* 3!* 3!.
>
> Em qua., 13 de mar. de 2024 às 09:09, Claudio Buffara <
> claudio.buff...@gmail.com> escreveu:
>
>> Pense no oposto: de quantas maneiras as crianças e adultos podem se
>> sentar separados uns dos outros.
>>
>> On Wed, Mar 13, 2024 at 8:39 AM Pedro Júnior 
>> wrote:
>>
>>> Olá pessoal, bom dia.
>>> Alguém poderia me ajudar nesse problema?
>>>
>>> Seis poltronas enfileiradas em um cinema e entram 3 adultos e 3
>>> crianças. De quantas maneiras podem sentar-se 2 crianças juntas e dois
>>> adultos juntos?
>>>
>>>
>>> Desde já fico grato!
>>>
>>> --
>>> Esta mensagem foi verificada pelo sistema de antivírus e
>>> acredita-se estar livre de perigo.
>>
>>
>> --
>> Esta mensagem foi verificada pelo sistema de antivírus e
>> acredita-se estar livre de perigo.
>
>
>
> --
>
> Pedro Jerônimo S. de O. Júnior
>
> Professor de Matemática
>
> Geo João Pessoa – PB
>
> --
> Esta mensagem foi verificada pelo sistema de antivírus e
> acredita-se estar livre de perigo.

-- 
Esta mensagem foi verificada pelo sistema de antiv�rus e
 acredita-se estar livre de perigo.



[obm-l] Re: [obm-l] Re: [obm-l] Combinatória

2024-03-13 Por tôpico Pedro Júnior
Eu pensei sim, mas e os casos do tipo ACCACAC. Esse caso não entra na conta
6! - 2* 3!* 3!.

Em qua., 13 de mar. de 2024 às 09:09, Claudio Buffara <
claudio.buff...@gmail.com> escreveu:

> Pense no oposto: de quantas maneiras as crianças e adultos podem se sentar
> separados uns dos outros.
>
> On Wed, Mar 13, 2024 at 8:39 AM Pedro Júnior 
> wrote:
>
>> Olá pessoal, bom dia.
>> Alguém poderia me ajudar nesse problema?
>>
>> Seis poltronas enfileiradas em um cinema e entram 3 adultos e 3 crianças.
>> De quantas maneiras podem sentar-se 2 crianças juntas e dois adultos juntos?
>>
>>
>> Desde já fico grato!
>>
>> --
>> Esta mensagem foi verificada pelo sistema de antivírus e
>> acredita-se estar livre de perigo.
>
>
> --
> Esta mensagem foi verificada pelo sistema de antivírus e
> acredita-se estar livre de perigo.



-- 

Pedro Jerônimo S. de O. Júnior

Professor de Matemática

Geo João Pessoa – PB

-- 
Esta mensagem foi verificada pelo sistema de antiv�rus e
 acredita-se estar livre de perigo.



[obm-l] Re: [obm-l] Re: [obm-l] Desigualdade e frações

2024-02-27 Por tôpico Pedro Júnior
Desculpas, Cláudio. É isso mesmo, com "a" e "b" inteiros e positivos.

Obrigado pela brilhante solução.

Em ter, 27 de fev de 2024 01:41, Claudio Buffara 
escreveu:

> Deveria ser a e b inteiros positivos, não?
> Pois se forem inteiros sem restrição, então como 2022/2023 < 2022,5/2023,5
> < 2023/2024, bastaria tomar a sequência:
> a(n) = -20225*n  e  b(n) = -20235*n.
> Daí teríamos 2022/2023 < a(n)/b(n) < 2023/2024 e a sequência a(n)+b(n)
> seria ilimitada inferiormente.
>
> Assim, suponhamos que a e b sejam inteiros positivos.
> 2022/2023 < a/b < 2023/2024 implica que b > a+1, já que a sequência
> (n/(n+1)) é crescente.
> Além disso, usando razões e proporções, achamos que:
> 2022 < a/(b-a) < 2023 < b/(b-a) < 2024
> ==> para que a+b seja o menor possível, b-a deverá ser o menor possível.
> E o menor valor possível de b-a é 2.
> Usando frações equivalentes, dá pra escrever 4044/4046 < a/b < 4046/4048 e
> daí teríamos uma única fração a/b com b - a = 2.
> Seria a/b = 4045/4047 ==> a+b mínimo = 8092.
>
> []s,
> Claudio.
>
>
>
>
> On Mon, Feb 26, 2024 at 10:12 PM Pedro Júnior 
> wrote:
>
>> Quem puder me ajudar, fixo grato.
>>
>> Sejam a e b dois números inteiros. Sabendo que 2022/2023 < a/b <
>> 2023/2024, determine o menos calor da soma a + b.
>>
>> --
>> Esta mensagem foi verificada pelo sistema de antivírus e
>> acredita-se estar livre de perigo.
>
>
> --
> Esta mensagem foi verificada pelo sistema de antivírus e
> acredita-se estar livre de perigo.

-- 
Esta mensagem foi verificada pelo sistema de antiv�rus e
 acredita-se estar livre de perigo.



[obm-l] Re: [obm-l] Re: Const. de triângulo

2024-01-16 Por tôpico Anderson Torres
Em ter, 16 de jan de 2024 12:23, Claudio Buffara 
escreveu:

> "Há vários problemas de CT com duas soluções."
>
> Claro!...   Fora o óbvio , com infinitas soluções (todas
> semelhantes entre si...) tem o  se, por exemplo, A for agudo e a < b
> < a/sen(A).
>
> O Geogebra certamente é uma tremenda ferramenta.
> Mas quantos professores sabem usá-lo adequadamente?
>


1. Quantos professores têm contato? Mesmo fatorando o acesso à internet,
ainda creio serem poucos

2. Seria interessante se fosse adotado, é bem lúdico!


> []s,
> Claudio.
>
>
> On Mon, Jan 15, 2024 at 7:53 PM Luís Lopes  wrote:
>
>> Oi Claudio,
>>
>> Eu acho que para os problemas no contexto que estamos falando a álgebra
>> pode decidir. Como o 17-gon. É construtível mas talvez a construção em si
>> poderia não ser conhecida. Os problemas dados 3 pontos da lista do Wernick
>> também precisaram de pesquisas para se decidir. Mas não sei muito sobre o
>> assunto.
>>
>> Há vários problemas de CT com duas soluções.
>>
>> O problema do quadrilátero é muito legal e também muito difícil acho que
>> para qualquer um. Há soluções (não sei se são fundamentalmente diferentes)
>> no livro do Virgílio, Court e do FG-M.
>>
>> Mas, pra mim, a principal função destes problemas de construção e’
>> pedagógica.
>>
>> É isso aí. Muita criatividade. E o Geogebra pode ajudar muito.
>>
>> Abs,
>> Luís
>>
>>
>>
>> On Jan 14, 2024, at 11:21 AM, Claudio Buffara 
>> wrote:
>>
>> Não tenho dúvidas de que o nível de dificuldade destes problemas varia de
>> “trivial” até “extremamente difícil”. Talvez até existam problemas em
>> aberto - ninguém acha uma solução e nem consegue provar que não existe
>> solução.
>>
>> O problem dos dados e’ interessante: existem triplas de dados que
>> resultam em dois ou mais triângulos não congruentes? Os casos clássicos de
>> congruência sugerem que a resposta é não. Mas talvez alguns tipos de dado
>> sejam mais “fracos” e não determinem totalmente o triângulo.
>>
>> Saindo dos triângulos, um legal e não muito fácil (pra mim…) é construir
>> um quadrilátero inscritível dados os comprimentos dos lados.
>>
>> Mas, pra mim, a principal função destes problemas de construção e’
>> pedagógica. Inseridos num curso de geometria, eles são uma variante
>> interessante de problemas métricos (a enorme maioria dos problemas vistos
>> na escola) nos quais os estudantes precisam usar a criatividade pra aplicar
>> propriedades básicas de figuras geométricas simples mas de um jeito
>> diferente, com muito mais necessidade de visualização.
>>
>> []s,
>> Claudio
>>
>> Em dom., 14 de jan. de 2024 às 11:41, Luís Lopes 
>> escreveu:
>>
>>> Oi Claudio,
>>>
>>> Mando pra vc com CC pra lista pra fazer mais um teste e ver se a lista
>>> recebe. Reply não funciona.
>>>
>>> Outra maneira seria usando o triângulo AMaMb. Esse problema é simples.
>>> Mais interessantes são (d_a; e_a bissetrizes interna e externa)  e
>>>  e os primos esquecidos <,e_a>.
>>>
>>> Problemas com e_a não são muito vistos. Como aquele que apareceu no
>>> WhatsApp do Madeira: construir o triângulo retângulo dados D_b, D_c e X,
>>> ponto do incírculo na reta BC. Não considerei com E_b , E_c, a gente acaba
>>> esquecendo. Nem sei como seria. Ou até com X_a, ponto do
>>> A-exincírculo. A lista é enorme.
>>>
>>> Considere agora . Tirei o < _a>. Bem fácil. E como dados dois qq
>>> entre  o terceiro fica determinado (sem falar em
>>> B-C), então  e  também são fáceis. E  cai na
>>> categoria  e .
>>>
>>> O que pode ser um desafio é a discussão sobre os dados nos problemas
>>> . Todos eles têm somente uma solução (considerando triângulos não
>>> congruentes, a segunda solução no , m>=h> não conta). No  os
>>> dados têm que satisfazer d sin(A/2) < h <= d. Para  não sei como
>>> determinar.
>>>
>>> Abs,
>>> Luís
>>>
>>>
>>> On Jan 14, 2024, at 7:48 AM, Claudio Buffara 
>>> wrote:
>>>
>>> 
>>>
>>> Trace AM com comprimento m_a.
>>> Trace a circunferência com diâmetro AM.
>>> Trace AP com comprimento h_a e P na circunferência.
>>>
>>> * M será o ponto médio de BC e P o pé da altura relativa a A.
>>>
>>> Prolonga AM até MA', com AM = MA'.
>>>
>>> * AA' será a diagonal do paralelogramo ABA'C, cujas diagonais se
>>> bissectam em M.
>>>
>>> Traça arco capaz de 180-A sobre AA'.
>>>
>>> * Já que, num paralelogramo, ângulos consecutivos são suplementares.
>>>
>>> Chame de B o ponto de intersecção deste arco capaz com a reta PM.
>>> Marque C na reta PM tal que B-M-C e MC = MB.
>>> E acabou.
>>>
>>> Há outra solução marcando P na outra semicircunferência de diâmetro AM
>>> (a menos que h_a = m_a).
>>>
>>> []s,
>>> Claudio.
>>>
>>>
>>> On Sun, Jan 14, 2024 at 12:58 AM Luís Lopes 
>>> wrote:
>>>
 Saudações, oi Anderson,

 Soluções usando fórmulas servem para mostrar que o triângulo é
 construtível e qual é sua forma e tamanho. Já ajuda naquela parte - suponha
 o problema resolvido. Mas a construção procurada deverá ser feita usando as
 propriedades da figura.

 Posso mandar no 

[obm-l] Re: [obm-l] Re: [obm-l] Const. de triângulo

2024-01-14 Por tôpico Claudio Buffara
Não tenho dúvidas de que o nível de dificuldade destes problemas varia de
“trivial” até “extremamente difícil”. Talvez até existam problemas em
aberto - ninguém acha uma solução e nem consegue provar que não existe
solução.

O problem dos dados e’ interessante: existem triplas de dados que resultam
em dois ou mais triângulos não congruentes? Os casos clássicos de
congruência sugerem que a resposta é não. Mas talvez alguns tipos de dado
sejam mais “fracos” e não determinem totalmente o triângulo.

Saindo dos triângulos, um legal e não muito fácil (pra mim…) é construir um
quadrilátero inscritível dados os comprimentos dos lados.

Mas, pra mim, a principal função destes problemas de construção e’
pedagógica. Inseridos num curso de geometria, eles são uma variante
interessante de problemas métricos (a enorme maioria dos problemas vistos
na escola) nos quais os estudantes precisam usar a criatividade pra aplicar
propriedades básicas de figuras geométricas simples mas de um jeito
diferente, com muito mais necessidade de visualização.

[]s,
Claudio

Em dom., 14 de jan. de 2024 às 11:41, Luís Lopes 
escreveu:

> Oi Claudio,
>
> Mando pra vc com CC pra lista pra fazer mais um teste e ver se a lista
> recebe. Reply não funciona.
>
> Outra maneira seria usando o triângulo AMaMb. Esse problema é simples.
> Mais interessantes são (d_a; e_a bissetrizes interna e externa)  e
>  e os primos esquecidos <,e_a>.
>
> Problemas com e_a não são muito vistos. Como aquele que apareceu no
> WhatsApp do Madeira: construir o triângulo retângulo dados D_b, D_c e X,
> ponto do incírculo na reta BC. Não considerei com E_b , E_c, a gente acaba
> esquecendo. Nem sei como seria. Ou até com X_a, ponto do
> A-exincírculo. A lista é enorme.
>
> Considere agora . Tirei o < _a>. Bem fácil. E como dados dois qq
> entre  o terceiro fica determinado (sem falar em
> B-C), então  e  também são fáceis. E  cai na
> categoria  e .
>
> O que pode ser um desafio é a discussão sobre os dados nos problemas
> . Todos eles têm somente uma solução (considerando triângulos não
> congruentes, a segunda solução no , m>=h> não conta). No  os
> dados têm que satisfazer d sin(A/2) < h <= d. Para  não sei como
> determinar.
>
> Abs,
> Luís
>
>
> On Jan 14, 2024, at 7:48 AM, Claudio Buffara 
> wrote:
>
> 
>
> Trace AM com comprimento m_a.
> Trace a circunferência com diâmetro AM.
> Trace AP com comprimento h_a e P na circunferência.
>
> * M será o ponto médio de BC e P o pé da altura relativa a A.
>
> Prolonga AM até MA', com AM = MA'.
>
> * AA' será a diagonal do paralelogramo ABA'C, cujas diagonais se bissectam
> em M.
>
> Traça arco capaz de 180-A sobre AA'.
>
> * Já que, num paralelogramo, ângulos consecutivos são suplementares.
>
> Chame de B o ponto de intersecção deste arco capaz com a reta PM.
> Marque C na reta PM tal que B-M-C e MC = MB.
> E acabou.
>
> Há outra solução marcando P na outra semicircunferência de diâmetro AM (a
> menos que h_a = m_a).
>
> []s,
> Claudio.
>
>
> On Sun, Jan 14, 2024 at 12:58 AM Luís Lopes  wrote:
>
>> Saudações, oi Anderson,
>>
>> Soluções usando fórmulas servem para mostrar que o triângulo é
>> construtível e qual é sua forma e tamanho. Já ajuda naquela parte - suponha
>> o problema resolvido. Mas a construção procurada deverá ser feita usando as
>> propriedades da figura.
>>
>> Posso mandar no privado para quem se interessar as construções com as
>> figuras que um correspondente me enviou. Esse que tem h_c/b como dado é bem
>> interessante.
>>
>> Agora o problema  pode ser resolvido de 3 ou mais maneiras.
>> Com medianas é sempre bom pensar em simetrias e paralelogramos.
>>
>> Luís
>>
>>
>> --
>> Esta mensagem foi verificada pelo sistema de antivírus e
>>  acredita-se estar livre de perigo.
>>
>>
>> =
>> Instru�ões para entrar na lista, sair da lista e usar a lista em
>> http://www.mat.puc-rio.br/~obmlistas/obm-l.html
>> =
>>
>
> --
>
> Esta mensagem foi verificada pelo sistema de antiv�rus e
>
> acredita-se estar livre de perigo.
>
>

-- 
Esta mensagem foi verificada pelo sistema de antiv�rus e
 acredita-se estar livre de perigo.



[obm-l] Re: [obm-l] Re: [obm-l] Re: [obm-l] Pesagens ( Balança Eletrônica)

2023-11-19 Por tôpico Claudio Buffara
Por que você não começa com um caso menor, tal como 4, 6 ou 9 moedas no
total?

Como você não consegue distinguir, numa dada pesagem, um grupo só com
moedas verdadeiras e um grupo com 2 moedas falsas, um algoritmo
pra resolver este problema com o menor número possível de pesagens não me
parece óbvio.  Daí, a análise de um caso menor pode dar alguma luz.

[]s,
Claudio.


On Sun, Nov 19, 2023 at 3:50 PM Jeferson Almir 
wrote:

> Eu tinha errado umas contas, mas sua cota está correta Ralph, preciso
> montar um exemplo com 21 pesagens
>
> Em dom., 19 de nov. de 2023 às 15:00, Jeferson Almir <
> jefersonram...@gmail.com> escreveu:
>
>> Pelo visto, está sim Ralph!! Agora temos que montar uma estratégia que
>> com 21 pesagens.
>>
>> Em dom., 19 de nov. de 2023 às 13:55, Ralph Costa Teixeira <
>> ralp...@gmail.com> escreveu:
>>
>>> Existem 2022*2021/2 possibilidades para as 2 falsas. Qualquer estratégia
>>> que seja criada com k pesagens que dão apenas 2 respostas cada distingue no
>>> máximo dentre 2^k possibilidades. Então devemos ter 2^k >= 2022*2021/2...
>>> hmm, isso daria k como pelo menos 21? Errei algo?
>>>
>>> On Sun, Nov 19, 2023, 12:16 Jeferson Almir 
>>> wrote:
>>>
 Amigos, encontrei como K mínimo o valor 11 mas desconfio que seja
 menos. Se alguém souber uma ideia que acabe o problema serei grato.


 Em Villa Par todas as moedas autênticas pesam uma quantidade par de
 gramas e todas as moedas falsas pesam uma quantidade impar de gramas.

 Se você tiver 2022 moedas entre as quais sabe que exatamente 2 são
 falsas.

 Se tiver uma balança eletrônica que informe apenas se o peso total dos
 objetos colocados nela é par ou impar.

 Determine o valor mínimo de k para qualquer estratégia que permita
 identificar as moedas falsas usando a balança  no máximo k vezes.

 --
 Esta mensagem foi verificada pelo sistema de antivírus e
 acredita-se estar livre de perigo.
>>>
>>>
>>> --
>>> Esta mensagem foi verificada pelo sistema de antivírus e
>>> acredita-se estar livre de perigo.
>>
>>
> --
> Esta mensagem foi verificada pelo sistema de antivírus e
> acredita-se estar livre de perigo.

-- 
Esta mensagem foi verificada pelo sistema de antiv�rus e
 acredita-se estar livre de perigo.



[obm-l] Re: [obm-l] Re: [obm-l] Pesagens ( Balança Eletrônica)

2023-11-19 Por tôpico Jeferson Almir
Eu tinha errado umas contas, mas sua cota está correta Ralph, preciso
montar um exemplo com 21 pesagens

Em dom., 19 de nov. de 2023 às 15:00, Jeferson Almir <
jefersonram...@gmail.com> escreveu:

> Pelo visto, está sim Ralph!! Agora temos que montar uma estratégia que com
> 21 pesagens.
>
> Em dom., 19 de nov. de 2023 às 13:55, Ralph Costa Teixeira <
> ralp...@gmail.com> escreveu:
>
>> Existem 2022*2021/2 possibilidades para as 2 falsas. Qualquer estratégia
>> que seja criada com k pesagens que dão apenas 2 respostas cada distingue no
>> máximo dentre 2^k possibilidades. Então devemos ter 2^k >= 2022*2021/2...
>> hmm, isso daria k como pelo menos 21? Errei algo?
>>
>> On Sun, Nov 19, 2023, 12:16 Jeferson Almir 
>> wrote:
>>
>>> Amigos, encontrei como K mínimo o valor 11 mas desconfio que seja menos.
>>> Se alguém souber uma ideia que acabe o problema serei grato.
>>>
>>>
>>> Em Villa Par todas as moedas autênticas pesam uma quantidade par de
>>> gramas e todas as moedas falsas pesam uma quantidade impar de gramas.
>>>
>>> Se você tiver 2022 moedas entre as quais sabe que exatamente 2 são
>>> falsas.
>>>
>>> Se tiver uma balança eletrônica que informe apenas se o peso total dos
>>> objetos colocados nela é par ou impar.
>>>
>>> Determine o valor mínimo de k para qualquer estratégia que permita
>>> identificar as moedas falsas usando a balança  no máximo k vezes.
>>>
>>> --
>>> Esta mensagem foi verificada pelo sistema de antivírus e
>>> acredita-se estar livre de perigo.
>>
>>
>> --
>> Esta mensagem foi verificada pelo sistema de antivírus e
>> acredita-se estar livre de perigo.
>
>

-- 
Esta mensagem foi verificada pelo sistema de antiv�rus e
 acredita-se estar livre de perigo.



[obm-l] Re: [obm-l] Re: [obm-l] Pesagens ( Balança Eletrônica)

2023-11-19 Por tôpico Jeferson Almir
Pelo visto, está sim Ralph!! Agora temos que montar uma estratégia que com
21 pesagens.

Em dom., 19 de nov. de 2023 às 13:55, Ralph Costa Teixeira <
ralp...@gmail.com> escreveu:

> Existem 2022*2021/2 possibilidades para as 2 falsas. Qualquer estratégia
> que seja criada com k pesagens que dão apenas 2 respostas cada distingue no
> máximo dentre 2^k possibilidades. Então devemos ter 2^k >= 2022*2021/2...
> hmm, isso daria k como pelo menos 21? Errei algo?
>
> On Sun, Nov 19, 2023, 12:16 Jeferson Almir 
> wrote:
>
>> Amigos, encontrei como K mínimo o valor 11 mas desconfio que seja menos.
>> Se alguém souber uma ideia que acabe o problema serei grato.
>>
>>
>> Em Villa Par todas as moedas autênticas pesam uma quantidade par de
>> gramas e todas as moedas falsas pesam uma quantidade impar de gramas.
>>
>> Se você tiver 2022 moedas entre as quais sabe que exatamente 2 são falsas.
>>
>> Se tiver uma balança eletrônica que informe apenas se o peso total dos
>> objetos colocados nela é par ou impar.
>>
>> Determine o valor mínimo de k para qualquer estratégia que permita
>> identificar as moedas falsas usando a balança  no máximo k vezes.
>>
>> --
>> Esta mensagem foi verificada pelo sistema de antivírus e
>> acredita-se estar livre de perigo.
>
>
> --
> Esta mensagem foi verificada pelo sistema de antivírus e
> acredita-se estar livre de perigo.

-- 
Esta mensagem foi verificada pelo sistema de antiv�rus e
 acredita-se estar livre de perigo.



[obm-l] Re: [obm-l] Re: [obm-l] equação

2023-10-25 Por tôpico Daniel Jelin
Obrigado, Marcelo, abs!

Em qua., 25 de out. de 2023 00:24, Marcelo Gonda Stangler <
marcelo.gonda.stang...@gmail.com> escreveu:

> Este problema, com um pouco de uso de substituição pode ser mostrado como
> análogo a isolar em x: k=x-e^(-1/x+1)
> Tu precisas limitar o "quanto" estás disposto a fatorar, pois poderiamos
> isolar x deixando-o em função de f(x) tal que f(x)-e^(-1/f(x)+1)=k. Mas
> suspeito que não é isto que queres.
> Se estamos falando de isolar algebricamente x, podemos notar alguns pontos:
>   Exp(x) para valores não triviais causa transformações relativas a x na
> base minimal que contém x de extensão sobre A, o corpo dos algébricos.
>   Se k é algébrico não nulo, x deve ser transcedental, visto que e é
> transcedental e (-1/x+1) pertence ao corpo dos A[x], assim x ser algebráico
> seria um absurdo.
>   Se x é algébrico, à exceção de 1, raiz de -1/x+1, k será transcedental
> uma vez que e o é.
> Assim, à exceção do caso (k,x)=(0,1), não haverá soluções em que x e k são
> algebráicos. Então, ao isolar o x, obteriamos algo em relação a "e" ou "ln".
> Como k=x-e^(-1/x+1), a base minimal de extensão que contém k é a união
> desta base de x, e da base transformada de x por Exp().
> Assim, a base minimal de x teria que ser a união da base de k e da base
> transformada de k por Exp() (1) ou Ln() (2).
> (1) implica que ambos são algébricos e (k,x)=(0,1)
> (2) implica que BM(x) = BM(k) U BM(Ln(k)) = BM(x) U BM(Exp(x)) U
> BM(Ln(k)), também implica (k,x)=(0,1)
>
> Dessa forma provamos que é impossível 'isolar' o x em função de k.
>
> Em ter, 24 de out de 2023 21:15, Daniel Jelin 
> escreveu:
>
>> Caros, olá. Tenho a seguinte equação: 1/ln(x) - 1/(x-1) = k, com x e k
>> reais. Quero isolar o x, mas não consigo. Pergunto: alguém tem alguma dica?
>> E pergunto tb: é possível que simplesmente não haja meios de isolar o x?
>> Nesse caso, como se prova isso? abs.
>>
>> --
>> Esta mensagem foi verificada pelo sistema de antivírus e
>> acredita-se estar livre de perigo.
>
>
> --
> Esta mensagem foi verificada pelo sistema de antivírus e
> acredita-se estar livre de perigo.

-- 
Esta mensagem foi verificada pelo sistema de antiv�rus e
 acredita-se estar livre de perigo.



[obm-l] Re: [obm-l] Re: [obm-l] Re: [obm-l] Comunicação

2023-08-28 Por tôpico Tiago Sandino
https://t.me/+jz8XW7bgRqNlOTg5

Criei esse grupo no telegram. A principal vantagem do Telegram em relação
ao Whatsapp é que quem entra pode ter acesso a todas as mensagens e
arquivos anteriores. A quantidade de membros que podem entrar é de 200.000.
Tô pensando aqui em umas regras também, tais como proibição de pirataria,
spam e algumas coisas a mais que podemos discutir lá. Melhor que isso, só
se alguém fizesse um fórum e fosse possível escrever em latex lá. Tem o
AOPS ok, mas nada nosso mesmo. Fiquem à vontade para entrar. Se o link
expirar, podem me solicitar outro por aqui.

Att.
*Prof. Tiago Sandino*
*85 999134896*

Em seg., 28 de ago. de 2023 02:02, Rogerio Ponce 
escreveu:

> Ola pessoal!
>
> Nesta lista, da qual participamos, qualquer um (mesmo que não esteja
> inscrito na lista) pode acessar os arquivos, fazer pesquisas e ler
> todos os problemas e suas solucoes.
>
> No whatsapp, isto seria impossivel, a menos que o individuo ja
> estivesse participando desde o inicio.
>
> []'s
> Rogerio Ponce
>
> On Sat, Aug 26, 2023 at 1:36 PM Esdras Muniz 
> wrote:
> >
> > Seria muito legal se existisse.
> >
> > Em sex, 25 de ago de 2023 18:24, Priscila Santana <
> priscila@hotmail.com> escreveu:
> >>
> >>
> >> Olá!
> >>
> >> Existe algum grupo de discussão de questões olímpicas no WhatsApp?
> >>
> >> Atte.
> >>
> >> Priscila S. da Paz
> >>
> >>
> >>
> >> --
> >> Esta mensagem foi verificada pelo sistema de antivírus e
> >> acredita-se estar livre de perigo.
> >
> >
> > --
> > Esta mensagem foi verificada pelo sistema de antivírus e
> > acredita-se estar livre de perigo.
>
> --
> Esta mensagem foi verificada pelo sistema de antivírus e
>  acredita-se estar livre de perigo.
>
>
> =
> Instru�ões para entrar na lista, sair da lista e usar a lista em
> http://www.mat.puc-rio.br/~obmlistas/obm-l.html
> =
>

-- 
Esta mensagem foi verificada pelo sistema de antiv�rus e
 acredita-se estar livre de perigo.



[obm-l] Re: [obm-l] Re: [obm-l] Comunicação

2023-08-27 Por tôpico Rogerio Ponce
Ola pessoal!

Nesta lista, da qual participamos, qualquer um (mesmo que não esteja
inscrito na lista) pode acessar os arquivos, fazer pesquisas e ler
todos os problemas e suas solucoes.

No whatsapp, isto seria impossivel, a menos que o individuo ja
estivesse participando desde o inicio.

[]'s
Rogerio Ponce

On Sat, Aug 26, 2023 at 1:36 PM Esdras Muniz  wrote:
>
> Seria muito legal se existisse.
>
> Em sex, 25 de ago de 2023 18:24, Priscila Santana  
> escreveu:
>>
>>
>> Olá!
>>
>> Existe algum grupo de discussão de questões olímpicas no WhatsApp?
>>
>> Atte.
>>
>> Priscila S. da Paz
>>
>>
>>
>> --
>> Esta mensagem foi verificada pelo sistema de antivírus e
>> acredita-se estar livre de perigo.
>
>
> --
> Esta mensagem foi verificada pelo sistema de antivírus e
> acredita-se estar livre de perigo.

-- 
Esta mensagem foi verificada pelo sistema de antiv�rus e
 acredita-se estar livre de perigo.


=
Instru��es para entrar na lista, sair da lista e usar a lista em
http://www.mat.puc-rio.br/~obmlistas/obm-l.html
=


[obm-l] Re: [obm-l] Re: [obm-l] Comunicação

2023-08-26 Por tôpico Marcelo Gonda Stangler
https://chat.whatsapp.com/CNGgk3NcgwY4AFsB61COXB
Quem sabe?

On Sat, Aug 26, 2023 at 1:36 PM Esdras Muniz 
wrote:

> Seria muito legal se existisse.
>
> Em sex, 25 de ago de 2023 18:24, Priscila Santana <
> priscila@hotmail.com> escreveu:
>
>>
>> Olá!
>>
>> Existe algum grupo de discussão de questões olímpicas no WhatsApp?
>>
>> Atte.
>>
>> *Priscila S. da Paz*
>>
>>
>>
>> --
>> Esta mensagem foi verificada pelo sistema de antivírus e
>> acredita-se estar livre de perigo.
>>
>
> --
> Esta mensagem foi verificada pelo sistema de antivírus e
> acredita-se estar livre de perigo.

-- 
Esta mensagem foi verificada pelo sistema de antiv�rus e
 acredita-se estar livre de perigo.



[obm-l] Re: [obm-l] Re: [obm-l] Re: teoria dos números

2023-08-13 Por tôpico Israel Meireles Chrisostomo
Muito obrigado!

Em qui., 10 de ago. de 2023 22:27, Ian Barquette <
ianbarquettelou...@gmail.com> escreveu:

> Se a função já está definida, e você quer apenas pontuar os limites dela,
> seria o conceito de imagem da função:
>
> Im(f) = (0, 1) = ]0, 1[
>
>
>
> Caso a função não esteja definida, a restrição seria o contradomínio da
> função:
>
> CD(f) = (0, 1) = ]0, 1[
>
> Ao definir a função, considerando C um conjunto qualquer,
>
> f: C -> (0, 1)
>
> Em qui., 10 de ago. de 2023 20:11, Israel Meireles Chrisostomo <
> israelmchrisost...@gmail.com> escreveu:
>
>> Como faço para definir em notação de conjuntos uma função com a
>> restrição, tipo 0> Seria (0,1]x(0,1]?
>>
>> Em qui., 10 de ago. de 2023 às 20:15, Israel Meireles Chrisostomo <
>> israelmchrisost...@gmail.com> escreveu:
>>
>>> Como faço para definir em notação de conjuntos uma função com a
>>> restrição, tipo f(x)<1
>>> Seria (0,1]x(0,1]?
>>>
>>> --
>>> Israel Meireles Chrisostomo
>>>
>>
>>
>> --
>> Israel Meireles Chrisostomo
>>
>> --
>> Esta mensagem foi verificada pelo sistema de antivírus e
>> acredita-se estar livre de perigo.
>
>
> --
> Esta mensagem foi verificada pelo sistema de antivírus e
> acredita-se estar livre de perigo.

-- 
Esta mensagem foi verificada pelo sistema de antiv�rus e
 acredita-se estar livre de perigo.



[obm-l] Re: [obm-l] Re: teoria dos números

2023-08-10 Por tôpico Ian Barquette
Se a função já está definida, e você quer apenas pontuar os limites dela,
seria o conceito de imagem da função:

Im(f) = (0, 1) = ]0, 1[



Caso a função não esteja definida, a restrição seria o contradomínio da
função:

CD(f) = (0, 1) = ]0, 1[

Ao definir a função, considerando C um conjunto qualquer,

f: C -> (0, 1)

Em qui., 10 de ago. de 2023 20:11, Israel Meireles Chrisostomo <
israelmchrisost...@gmail.com> escreveu:

> Como faço para definir em notação de conjuntos uma função com a restrição,
> tipo 0 Seria (0,1]x(0,1]?
>
> Em qui., 10 de ago. de 2023 às 20:15, Israel Meireles Chrisostomo <
> israelmchrisost...@gmail.com> escreveu:
>
>> Como faço para definir em notação de conjuntos uma função com a
>> restrição, tipo f(x)<1
>> Seria (0,1]x(0,1]?
>>
>> --
>> Israel Meireles Chrisostomo
>>
>
>
> --
> Israel Meireles Chrisostomo
>
> --
> Esta mensagem foi verificada pelo sistema de antivírus e
> acredita-se estar livre de perigo.

-- 
Esta mensagem foi verificada pelo sistema de antiv�rus e
 acredita-se estar livre de perigo.



[obm-l] Re: [obm-l] Re: [obm-l] Uma recorrência diferente

2023-04-06 Por tôpico Esdras Muniz
Vc pode pegar a função geratriz e usar a fórmula de Ramanujan pra calcular
o termo geral. Acredito que a função geratriz seja: e^{x^2/2+x}, já na
forma (x_n)x^n/n!...

Em qui, 6 de abr de 2023 19:03, Carlos Gustavo Tamm de Araujo Moreira <
g...@impa.br> escreveu:

> Caro Vanderlei,
> Não parece haver uma fórmula fechada muito simples. Veja
> https://oeis.org/A85 para várias referências
> sobre essa sequência.
> Abraços,
> Gugu
>
> On Wed, Apr 5, 2023 at 11:41 PM Professor Vanderlei Nemitz <
> vanderma...@gmail.com> wrote:
>
>> Oi, mestres!
>>
>> Estava resolvendo um problema de combinatória e obtive essa recorrência:
>>
>> *x(n) = x(n - 1) + (n - 1).x(n - 2), com x1 = 1 e x2 = 2*.
>>
>> Por exemplo, x3 = x2 + 2.x1 e x9 = x8 + 8.x7
>>
>> Como resolver quando os coeficientes não são todos constantes?
>>
>> Apenas como curiosidade, o problema que originou a recorrência é:
>> (IME - RJ) - Um professor dá um teste surpresa para uma turma de 9
>> alunos, e diz que o teste pode ser feito sozinho ou em grupos de 2 alunos.
>> De quantas formas a turma pode ser organizar para fazer o teste? (Por
>> exemplo, uma turma de 3 alunos pode ser organizar de 4 formas e uma turma
>> de 4 alunos pode se organizar de 10 formas)
>>
>>
>>
>> 
>>  Não
>> contém vírus.www.avast.com
>> 
>> <#m_-6699161896707809793_m_4118911927943204904_DAB4FAD8-2DD7-40BB-A1B8-4E2AA1F9FDF2>
>>
>> --
>> Esta mensagem foi verificada pelo sistema de antivírus e
>> acredita-se estar livre de perigo.
>
>
> --
> Esta mensagem foi verificada pelo sistema de antivírus e
> acredita-se estar livre de perigo.

-- 
Esta mensagem foi verificada pelo sistema de antiv�rus e
 acredita-se estar livre de perigo.



[obm-l] Re: [obm-l] Re: [obm-l] Re: [obm-l] cadeira de 3 pés

2023-01-23 Por tôpico Bernardo Freitas Paulo da Costa
On Mon, Jan 23, 2023 at 12:52 PM Claudio Buffara
 wrote:
>
> Obrigado, Wagner e Ponce:
>
> Eu tinha pensado em algo na linha do que o Ponce escreveu, ainda que em 
> certos casos patológicos (pelo menos de piso...) o terceiro pé pode não 
> encontrar apoio: imagine um piso com um pico fino em algum lugar (p.ex. a 
> superfície gerada pela revolução de z = 1/(1+(x^2+y^2)^20) em torno do eixo 
> z). Neste caso, precisaríamos de uma cadeira bem pequena, ou pelo menos com 
> as pontas dos pés bem próximas umas das outras.

Pois é, eu acho que tem uma questão interessante de estabilidade que é
a seguinte: se você pegar um banco de 3 pés e botar na encosta de uma
montanha (bem íngreme!), ele vai rolar ladeira abaixo.  Porque a
projeção do centro de gravidade cai fora do triângulo determinado pelo
contato com o solo (que define as normais que podem segurar a cadeira
em posição estática).  Uma cadeira de 4 pés só é bamba porque uma
pessoa não consegue ficar (infinitesimalmente) parada, e daí o centro
de gravidade oscila entre os triângulos formados por dois pares de
três apoios, por exemplo ABC e ACD.

Bernardo

-- 
Esta mensagem foi verificada pelo sistema de antiv�rus e
 acredita-se estar livre de perigo.


=
Instru��es para entrar na lista, sair da lista e usar a lista em
http://www.mat.puc-rio.br/~obmlistas/obm-l.html
=


[obm-l] Re: [obm-l] Re: [obm-l] cadeira de 3 pés

2023-01-23 Por tôpico Claudio Buffara
" No mundo 2D um banco de 2 pernas não balança, mas um de 3 pernas pode
balançar."

Aqui estamos falando de uma curva no plano e de um número real positivo d,
suficientemente pequeno de modo que, para todo ponto A na curva, existe um
ponto B na curva com dist(A,B) = d.
A e B são as pontas das pernas do banco 2D.
Isso será verdade se todo ponto da curva for o centro de uma circunferência
de raio d que intersecta a curva.

Um terceiro ponto C, a distâncias predeterminadas de A e B, pode não
pertencer à curva (ou seja, o triângulo ABC não estará inscrito na curva).
Mas é possível deslizar a corda AB, de comprimento constante, ao longo de
toda a curva.

No caso 3D, temos uma superfície no espaço e 3 números reais positivos a,
b, c, com cada um menor do que a soma dos outros dois, de modo que, para
cada A na superfície, é possível achar B e C na superfície com dist(A,B) =
c, dist(A,C) = b e dist(B,C) = a.
E é possível deslizar o triângulo ABC, com lados de comprimento constante,
ao longo de toda a superfície.
Mas, fixado A na superfície, não me parece tão simples construir B e C.


On Mon, Jan 23, 2023 at 11:29 AM Eduardo Wagner  wrote:

> O banco de 3 pernas não balança porque nosso mundo é tridimensional.
> Não tem nada a ver com plano ou triângulo. Um banco de 3 pernas não
> balança se for colocado
> no teto de um carro.
> No mundo 2D um banco de 2 pernas não balança, mas um de 3 pernas pode
> balançar.
> Em um mundo 4D uma cadeira de 4 pernas não balança.
> Wbs
> Wagner
>
>
> Em dom., 22 de jan. de 2023 às 23:24, Claudio Buffara <
> claudio.buff...@gmail.com> escreveu:
>
>> Achei na internet duas explicações distintas para a estabilidade de uma
>> cadeira (ou mesa ou banco) de 3 pés.
>> Aqui estão:
>> https://www.somatematica.com.br/curiosidades/c98.php
>>
>> http://colegiofarroupilha.com.br/site/qual-cadeira-e-mais-firme-a-que-tem-tres-ou-quatro-pes/
>>
>> Qual das duas é a explicação correta?
>> Ou nenhuma das duas? E, nesse caso, qual a explicação?
>>
>> []s,
>> Claudio.
>>
>> --
>> Esta mensagem foi verificada pelo sistema de antivírus e
>> acredita-se estar livre de perigo.
>
>
> --
> Esta mensagem foi verificada pelo sistema de antivírus e
> acredita-se estar livre de perigo.

-- 
Esta mensagem foi verificada pelo sistema de antiv�rus e
 acredita-se estar livre de perigo.



[obm-l] Re: [obm-l] Re: [obm-l] cadeira de 3 pés

2023-01-23 Por tôpico Claudio Buffara
Obrigado, Wagner e Ponce:

Eu tinha pensado em algo na linha do que o Ponce escreveu, ainda que em
certos casos patológicos (pelo menos de piso...) o terceiro pé pode não
encontrar apoio: imagine um piso com um pico fino em algum lugar (p.ex. a
superfície gerada pela revolução de z = 1/(1+(x^2+y^2)^20) em torno do eixo
z). Neste caso, precisaríamos de uma cadeira bem pequena, ou pelo menos com
as pontas dos pés bem próximas umas das outras.

Mas, pelo que o Wagner escreveu, acho que ainda tem um teorema mais
profundo aí.

[]s,
Claudio.


On Mon, Jan 23, 2023 at 11:54 AM Rogerio Ponce  wrote:

> Ola' Claudio!
> Eu diria que as duas explicações estão erradas, pois não se depende de
> ter apenas um plano definido pelas pontas dos pés, visto que uma
> cadeira de 4 pés pode, perfeitamente, ter as pontas dos 4 pés em
> apenas um plano, e, ainda assim, ela não é necessariamente estável.
>
> Um explicação menos ruim é que, numa cadeira de 3 pés, sempre podemos
> apoiar quaisquer 2 pés num piso (mesmo irregular), e, em torno do eixo
> definido pelos 2 pés já apoiados, podemos girar a cadeira até que o
> terceiro pé encontre o piso, de modo que a cadeira fique totalmente
> apoiada.
> Já numa cadeira de 4 pés, é comum que um dos pés fique sem contato com
> o chão, permitindo que a cadeira oscile em torno do eixo definido
> pelos 2 pés vizinhos ao pé sem contato.
>
> []'s
> Rogerio Ponce
>
> On Sun, Jan 22, 2023 at 11:23 PM Claudio Buffara
>  wrote:
> >
> > Achei na internet duas explicações distintas para a estabilidade de uma
> cadeira (ou mesa ou banco) de 3 pés.
> > Aqui estão:
> > https://www.somatematica.com.br/curiosidades/c98.php
> >
> http://colegiofarroupilha.com.br/site/qual-cadeira-e-mais-firme-a-que-tem-tres-ou-quatro-pes/
> >
> > Qual das duas é a explicação correta?
> > Ou nenhuma das duas? E, nesse caso, qual a explicação?
> >
> > []s,
> > Claudio.
> >
> > --
> > Esta mensagem foi verificada pelo sistema de antivírus e
> > acredita-se estar livre de perigo.
>
> --
> Esta mensagem foi verificada pelo sistema de antivírus e
>  acredita-se estar livre de perigo.
>
>
> =
> Instru�ões para entrar na lista, sair da lista e usar a lista em
> http://www.mat.puc-rio.br/~obmlistas/obm-l.html
> =
>

-- 
Esta mensagem foi verificada pelo sistema de antiv�rus e
 acredita-se estar livre de perigo.



[obm-l] Re: [obm-l] Re: [obm-l] cadeira de 3 pés

2023-01-23 Por tôpico Anderson Torres
Em seg, 23 de jan de 2023 11:54, Rogerio Ponce 
escreveu:

> Ola' Claudio!
> Eu diria que as duas explicações estão erradas, pois não se depende de
> ter apenas um plano definido pelas pontas dos pés, visto que uma
> cadeira de 4 pés pode, perfeitamente, ter as pontas dos 4 pés em
> apenas um plano, e, ainda assim, ela não é necessariamente estável.
>

Como não? A ideia de botar um calço é precisamente estabilizar o pé que não
encosta na superfície. E, exceto em feiras de ciências e circos, o chão
costuma ser plano.


> Um explicação menos ruim é que, numa cadeira de 3 pés, sempre podemos
> apoiar quaisquer 2 pés num piso (mesmo irregular), e, em torno do eixo
> definido pelos 2 pés já apoiados, podemos girar a cadeira até que o
> terceiro pé encontre o piso, de modo que a cadeira fique totalmente
> apoiada.
> Já numa cadeira de 4 pés, é comum que um dos pés fique sem contato com
> o chão, permitindo que a cadeira oscile em torno do eixo definido
> pelos 2 pés vizinhos ao pé sem contato.
>
> []'s
> Rogerio Ponce
>
> On Sun, Jan 22, 2023 at 11:23 PM Claudio Buffara
>  wrote:
> >
> > Achei na internet duas explicações distintas para a estabilidade de uma
> cadeira (ou mesa ou banco) de 3 pés.
> > Aqui estão:
> > https://www.somatematica.com.br/curiosidades/c98.php
> >
> http://colegiofarroupilha.com.br/site/qual-cadeira-e-mais-firme-a-que-tem-tres-ou-quatro-pes/
> >
> > Qual das duas é a explicação correta?
> > Ou nenhuma das duas? E, nesse caso, qual a explicação?
> >
> > []s,
> > Claudio.
> >
> > --
> > Esta mensagem foi verificada pelo sistema de antivírus e
> > acredita-se estar livre de perigo.
>
> --
> Esta mensagem foi verificada pelo sistema de antivírus e
>  acredita-se estar livre de perigo.
>
>
> =
> Instru�ões para entrar na lista, sair da lista e usar a lista em
> http://www.mat.puc-rio.br/~obmlistas/obm-l.html
> =
>

-- 
Esta mensagem foi verificada pelo sistema de antiv�rus e
 acredita-se estar livre de perigo.



[obm-l] Re: [obm-l] Re: [obm-l] Re: [obm-l] cadeira de 3 pés

2023-01-23 Por tôpico Anderson Torres
Em seg, 23 de jan de 2023 11:15, Claudio Buffara 
escreveu:

> Será que o argumento usando apenas o plano é suficiente?  Pois um banco de
> 3 pés também fica estável num piso irregular.
>

Mas nem toda cadeira de quatro pés fica estável em qualquer piso irregular.

A ideia subjacente ainda é a de "planidade". Os pés da cadeira estão na
intersecção entre o "plano dos pés" e o piso.
Mas o mesmo não se aplica às cadeiras quadrúpedes, pois nem sempre existe
um "plano dos pés".

Se bem que neste último caso é possível que os pés da mesa sejam coplanares
mas a superfície não o seja. Ou pior ainda, forçando um pouco na
continuidade, pode ser que qualquer cadeira de quatro pés se encaixe em
qualquer superfície não-plana.

Ou não. Se pegarmos por exemplo o mapa topográfico de uma planície com um
poço escavado e a distância entre dois pés da mesa sempre for maior que o
diâmetro do poço, não tem como encaixar uma mesa torta de quatro pés.

Estranho...

O resultado mais geral em que pensei foi o seguinte: dada qualquer
> superfície bi-dimensional contínua (por exemplo, que seja o gráfico de uma
> função contínua de RxR em R - uma suposição razoável se estamos tentando
> modelar um piso), você sempre consegue encostar nela as pontas dos 3 pés do
> banco, de modo que o banco fique "firme" ou sem folgas.
> Ou, mais formalmente, dado um triângulo ABC no espaço, existe uma
> isometria (do espaço) tal que as imagens de A, B e C por esta isometria
> estão em S.
>

Pensei algo do gênero, mas de maneira bem menos elaborada: dados um
triângulo ABC e dois pontos A',B' em uma superfície contínua que distam AB,
existe um ponto C´ tal que ABC = A'B'C'


> Enfim, esse talvez seja um problema mais de topologia do que de geometria.
> Pois, no fim das contas, "3 pontos não colineares determinam um único
> plano", assim como "2 pontos determinam uma única reta" são afirmações que
> têm um certo ar topológico, pelo menos pra mim.
>

> []s,
> Claudio.
>
> On Mon, Jan 23, 2023 at 7:02 AM Anderson Torres <
> torres.anderson...@gmail.com> wrote:
>
>>
>>
>> Em dom, 22 de jan de 2023 23:23, Claudio Buffara <
>> claudio.buff...@gmail.com> escreveu:
>>
>>> Achei na internet duas explicações distintas para a estabilidade de uma
>>> cadeira (ou mesa ou banco) de 3 pés.
>>> Aqui estão:
>>> https://www.somatematica.com.br/curiosidades/c98.php
>>>
>>> http://colegiofarroupilha.com.br/site/qual-cadeira-e-mais-firme-a-que-tem-tres-ou-quatro-pes/
>>>
>>
>> Nesse caso específico, a primeira me parece mais correta. Ou melhor, a
>> segunda tem falhas.
>>
>> A rigidez dos triângulos (TCC caso LLL de igualdade de triângulos) é
>> irrelevante para a questão da cadeira bamba. Em qualquer cadeira física e
>> palpável, as pernas são rígidas - portanto o polígono formado pelas
>> extremidades dessas pernas é rígido também.
>>
>> Mas um polígono rígido não é necessariamente um polígono bidimensional -
>> o que a cadeira bamba de 4 pernas exemplifica perfeitamente.
>>
>> Contate o webmaster da segunda página sugerindo correções!
>>
>>
>>> Qual das duas é a explicação correta?
>>> Ou nenhuma das duas? E, nesse caso, qual a explicação?
>>>
>>> []s,
>>> Claudio.
>>>
>>> --
>>> Esta mensagem foi verificada pelo sistema de antivírus e
>>> acredita-se estar livre de perigo.
>>
>>
>> --
>> Esta mensagem foi verificada pelo sistema de antivírus e
>> acredita-se estar livre de perigo.
>
>
> --
> Esta mensagem foi verificada pelo sistema de antivírus e
> acredita-se estar livre de perigo.

-- 
Esta mensagem foi verificada pelo sistema de antiv�rus e
 acredita-se estar livre de perigo.



[obm-l] Re: [obm-l] Re: [obm-l] cadeira de 3 pés

2023-01-23 Por tôpico Claudio Buffara
Será que o argumento usando apenas o plano é suficiente?  Pois um banco de
3 pés também fica estável num piso irregular.
O resultado mais geral em que pensei foi o seguinte: dada qualquer
superfície bi-dimensional contínua (por exemplo, que seja o gráfico de uma
função contínua de RxR em R - uma suposição razoável se estamos tentando
modelar um piso), você sempre consegue encostar nela as pontas dos 3 pés do
banco, de modo que o banco fique "firme" ou sem folgas.
Ou, mais formalmente, dado um triângulo ABC no espaço, existe uma isometria
(do espaço) tal que as imagens de A, B e C por esta isometria estão em S.

Enfim, esse talvez seja um problema mais de topologia do que de geometria.
Pois, no fim das contas, "3 pontos não colineares determinam um único
plano", assim como "2 pontos determinam uma única reta" são afirmações que
têm um certo ar topológico, pelo menos pra mim.

[]s,
Claudio.

On Mon, Jan 23, 2023 at 7:02 AM Anderson Torres <
torres.anderson...@gmail.com> wrote:

>
>
> Em dom, 22 de jan de 2023 23:23, Claudio Buffara <
> claudio.buff...@gmail.com> escreveu:
>
>> Achei na internet duas explicações distintas para a estabilidade de uma
>> cadeira (ou mesa ou banco) de 3 pés.
>> Aqui estão:
>> https://www.somatematica.com.br/curiosidades/c98.php
>>
>> http://colegiofarroupilha.com.br/site/qual-cadeira-e-mais-firme-a-que-tem-tres-ou-quatro-pes/
>>
>
> Nesse caso específico, a primeira me parece mais correta. Ou melhor, a
> segunda tem falhas.
>
> A rigidez dos triângulos (TCC caso LLL de igualdade de triângulos) é
> irrelevante para a questão da cadeira bamba. Em qualquer cadeira física e
> palpável, as pernas são rígidas - portanto o polígono formado pelas
> extremidades dessas pernas é rígido também.
>
> Mas um polígono rígido não é necessariamente um polígono bidimensional - o
> que a cadeira bamba de 4 pernas exemplifica perfeitamente.
>
> Contate o webmaster da segunda página sugerindo correções!
>
>
>> Qual das duas é a explicação correta?
>> Ou nenhuma das duas? E, nesse caso, qual a explicação?
>>
>> []s,
>> Claudio.
>>
>> --
>> Esta mensagem foi verificada pelo sistema de antivírus e
>> acredita-se estar livre de perigo.
>
>
> --
> Esta mensagem foi verificada pelo sistema de antivírus e
> acredita-se estar livre de perigo.

-- 
Esta mensagem foi verificada pelo sistema de antiv�rus e
 acredita-se estar livre de perigo.



[obm-l] Re: [obm-l] Re: [obm-l] Re: [obm-l] Caracterização de Inteiros

2022-11-17 Por tôpico Claudio Buffara
Só completando...

Apesar de números irracionais serem conhecidos desde a época de Pitágoras
(vide a famosa historinha do pitagórico Hipaso, que supostamente foi
afogado por ter "vazado" o segredo da existência dos irracionais), me
parece que eles só começaram a realmente fazer falta no século 19, quando
os matemáticos estavam preocupados em formalizar a análise. Foi naquele
ponto que o Dedekind se deu conta da importância da completeza (essa
palavra tá certa?...) dos reais e da necessidade dela ser garantida por um
axioma. Se não me engano, o teorema que ele queria provar é o de que uma
sequência monótona e limitada de números reais sempre converge. E descobriu
que não conseguiria sem fazer uma hipótese adicional sobre o corpo
dos reais, capaz de diferenciá-lo dos racionais (também um corpo ordenado),
mas dentro do qual nem toda sequência converge (por exemplo, x(n+1) =
(1/2)*(x(n) + 2/x(n)), com x(0) = 1, a qual converge pra raiz(2): um
irracional). A princípio, bastaria ele incluir o axioma de completeza dos
reais e provar que este axioma não contradiz os demais axiomas. Mas ele foi
mais longe e acabou inventando uma construção dos reais a partir dos
racionais (via os chamados "cortes de Dedekind"). Talvez (estou
conjecturando aqui) ele só tenha feito isso por uma necessidade psicológica
de provar que algo é possível (no caso a extensão dos racionais aos reais)
exibindo uma construção explícita. Isso talvez seja um reflexo da ojeriza
que matemáticos da época tinham (e alguns poucos têm até hoje) de
argumentos não construtivos pra provar a existência de fatos ou objetos
matemáticos. Por exemplo, demonstrações usando o princípio das casas de
pombos são desse tipo.

Sobre "entender sem compreender", lembrei de um ditado do John von
Neumann: "Em matemática, você nunca entende as coisas. Apenas se acostuma
com elas."

[]s,
Claudio.

On Wed, Nov 16, 2022 at 6:52 PM Claudio Buffara 
wrote:

> Não entendi como uma homotetia poderia reduzir um par ordenado a um único
> número... enfim...
>
> O que se faz, no caso da relação de equivalência que descrevi, é
> representar o par (a,b) pela notação a-b.
> Daí, (a,b) e (c,d) são equivalentes sss a-b = c-d.
> E a novidade são os números negativos: as classes de equivalência de pares
> (a,b) com a < b, representadas, por exemplo, pelo par (0,c), onde c = b-a.
> Ou, na notação usual, -c.
>
> Mas não acho que se deva perder muito tempo com a construção de sistemas
> numéricos via classes de equivalência, estendendo naturais para inteiros
> para racionais para reais e para complexos.  Até porque é extremamente
> sacal, a cada etapa, checar que as operações usuais (+ e *), quando
> aplicadas aos novos números, têm todas as propriedades que conhecemos da
> escola.
> Essas construções foram a maneira que os matemáticos acharam pra
> formalizar os sistemas numéricos, a partir de conceitos mais básicos (no
> caso, pares ordenados e relações de equivalência) - é o programa do Hilbert
> (ou de Russell e Whitehead), de reduzir toda a matemática à teoria dos
> conjuntos.
> Mas, no fundo, esta é uma construção artificial, ex post.  Pois
> matemáticos já usavam todos os números muito antes dessa formalização ser
> inventada.
> E não acho que ela renda muitos frutos, nem pedagógicos (a menos que seu
> objetivo seja "entender sem compreender") e nem pra ampliação da fronteira
> do conhecimento, exceto colocar os sistemas numéricos numa base axiomática
> sólida.
> Em particular, no que diz respeito aos números reais, a única coisa que
> interessa é que eles são um corpo ordenado completo. Tanto é que vários
> livros de análise partem deste axioma e não se preocupam em construir os
> reais a partir dos naturais.
>
> []s,
> Claudio.
>
>
>
> On Tue, Nov 15, 2022 at 5:07 PM Pedro José  wrote:
>
>> Obrigado a você e ao Cláudio. Mas não sou criativo para inventar. Mas já
>> vi que terei que fazer uma homotetia, para as classes de equivalência para
>> representar só como um número e não como um par, creio eu.
>>
>> Cordialmente,
>> PJMS
>>
>> Em ter., 15 de nov. de 2022 às 16:00, Anderson Torres <
>> torres.anderson...@gmail.com> escreveu:
>>
>>>
>>>
>>> Em ter, 15 de nov de 2022 14:33, Pedro José 
>>> escreveu:
>>>
 Boa tarde!
 Para os |Naturais, temos os postulados de Peano.

 Para os Inteiros há alguma formalização?

>>>
>>> invente uma!
>>>
>>> Pode ser por exemplo o conjunto de pares (p,q) tais que p-q é constante.
>>>
>>> ou melhor (p1,q1)=(p2,q2) se e só se p1+q2=p2+q1.
>>>
>>>
 Acho pobre dizer que é necessário ter outros números devido ao problema
 de fechamento nos naturais para a subtração que é fato e daí introduzir os
 simétricos que são inteiros e ainda não foram caracterizados.

 No meu antigo ginásio aprendi que os Reais era a união dos conjuntos
 disjuntos irracionais e racionais. Os racionais haviam sido bem definidos.
 Aí questionei e o que são irracionais? resposta: são os Reais que não são
 racionais, os que não podem 

[obm-l] Re: [obm-l] Re: [obm-l] Re: [obm-l] Caracterização de Inteiros

2022-11-16 Por tôpico Claudio Buffara
Não entendi como uma homotetia poderia reduzir um par ordenado a um único
número... enfim...

O que se faz, no caso da relação de equivalência que descrevi, é
representar o par (a,b) pela notação a-b.
Daí, (a,b) e (c,d) são equivalentes sss a-b = c-d.
E a novidade são os números negativos: as classes de equivalência de pares
(a,b) com a < b, representadas, por exemplo, pelo par (0,c), onde c = b-a.
Ou, na notação usual, -c.

Mas não acho que se deva perder muito tempo com a construção de sistemas
numéricos via classes de equivalência, estendendo naturais para inteiros
para racionais para reais e para complexos.  Até porque é extremamente
sacal, a cada etapa, checar que as operações usuais (+ e *), quando
aplicadas aos novos números, têm todas as propriedades que conhecemos da
escola.
Essas construções foram a maneira que os matemáticos acharam pra formalizar
os sistemas numéricos, a partir de conceitos mais básicos (no caso, pares
ordenados e relações de equivalência) - é o programa do Hilbert (ou de
Russell e Whitehead), de reduzir toda a matemática à teoria dos conjuntos.
Mas, no fundo, esta é uma construção artificial, ex post.  Pois matemáticos
já usavam todos os números muito antes dessa formalização ser inventada.
E não acho que ela renda muitos frutos, nem pedagógicos (a menos que seu
objetivo seja "entender sem compreender") e nem pra ampliação da fronteira
do conhecimento, exceto colocar os sistemas numéricos numa base axiomática
sólida.
Em particular, no que diz respeito aos números reais, a única coisa que
interessa é que eles são um corpo ordenado completo. Tanto é que vários
livros de análise partem deste axioma e não se preocupam em construir os
reais a partir dos naturais.

[]s,
Claudio.



On Tue, Nov 15, 2022 at 5:07 PM Pedro José  wrote:

> Obrigado a você e ao Cláudio. Mas não sou criativo para inventar. Mas já
> vi que terei que fazer uma homotetia, para as classes de equivalência para
> representar só como um número e não como um par, creio eu.
>
> Cordialmente,
> PJMS
>
> Em ter., 15 de nov. de 2022 às 16:00, Anderson Torres <
> torres.anderson...@gmail.com> escreveu:
>
>>
>>
>> Em ter, 15 de nov de 2022 14:33, Pedro José 
>> escreveu:
>>
>>> Boa tarde!
>>> Para os |Naturais, temos os postulados de Peano.
>>>
>>> Para os Inteiros há alguma formalização?
>>>
>>
>> invente uma!
>>
>> Pode ser por exemplo o conjunto de pares (p,q) tais que p-q é constante.
>>
>> ou melhor (p1,q1)=(p2,q2) se e só se p1+q2=p2+q1.
>>
>>
>>> Acho pobre dizer que é necessário ter outros números devido ao problema
>>> de fechamento nos naturais para a subtração que é fato e daí introduzir os
>>> simétricos que são inteiros e ainda não foram caracterizados.
>>>
>>> No meu antigo ginásio aprendi que os Reais era a união dos conjuntos
>>> disjuntos irracionais e racionais. Os racionais haviam sido bem definidos.
>>> Aí questionei e o que são irracionais? resposta: são os Reais que não são
>>> racionais, os que não podem ser escritos na forma p/q p e q inteiros e
>>> q<>0. Mas me deram um tombo. Definiram os |Reais com base nos irracionais e
>>> os irracionais com base nos |Reais. 3 +2i também não pode ser inscrito na
>>> forma p/q. Só mais tarde no científico, é que meu professor definiu
>>> irracional como um número que não podia ser escrito na forma p/q e cuja
>>> representação decimal tinha uma infinidade de algarismos, sem haver uma
>>> periodicidade.
>>> Na época foi o maior nó que tive com a matemática. O mestre demonstrou
>>> que os racionais eram densos, mas entre eles ainda cabiam os irracionais.
>>> Não satisfeito mostrou que os racionais eram enumeráveis e por absurdo
>>> mostrou que os |Reais não. Não satisfeito mostrou que a cardinalidade do
>>> intervalo [0,1] era maior que a dos |Naturais. Não conseguia conceber que
>>> havia um infinito maior que outro. Outra coisa que demorei a aceitar,mesmo
>>> vendo a bijeção, era que os inteiros e naturais tinham a mesma
>>> cardinalidade. Na minha cabeça, os inteiros têm todos os naturais ainda
>>> sobram os negativos, como é igual?
>>> Hoje, depois de velho, arrumei uma enteada, que muito me pergunta e
>>> estou enrolado. Para dar um ar de superioridade, questionei se conhecia os
>>> inteiros de Gaus, que 5 não era primo nos inteiros de Gaus. Estrepei-me, a
>>> danada foi pesquisar e me questiona sobre o que não tenho um domínio pleno.
>>> Em suma, como apresentei a ela os postulados de Peano para a
>>> caracterização dos Naturais, ela me cobra por algo semelhante para os
>>> Inteiros, e não sei responder.
>>> HELP! SOCORRO! AU SECOURS! AYUDA! AIUTO! HILFE!
>>> Cordialmente,
>>> PJMS
>>>
>>> --
>>> Esta mensagem foi verificada pelo sistema de antivírus e
>>> acredita-se estar livre de perigo.
>>
>>
>> --
>> Esta mensagem foi verificada pelo sistema de antivírus e
>> acredita-se estar livre de perigo.
>
>
> --
> Esta mensagem foi verificada pelo sistema de antivírus e
> acredita-se estar livre de perigo.

-- 
Esta mensagem foi verificada pelo sistema de 

[obm-l] Re: [obm-l] Re: [obm-l] Re: [obm-l] Caracterização de Inteiros

2022-11-15 Por tôpico Anderson Torres
Em ter, 15 de nov de 2022 17:07, Pedro José  escreveu:

> Obrigado a você e ao Cláudio. Mas não sou criativo para inventar. Mas já
> vi que terei que fazer uma homotetia, para as classes de equivalência para
> representar só como um número e não como um par, creio eu.
>

Eu lembro de quando li o Guidorizzi formalizando os reais. Até hoje sinto
que entendo sem compreender, haha!

Por outro lado, números reais (irracionais, no caso) são bem menos
palpáveis que os outros. Dívidas e frações são fáceis de entender, afinal.


> Cordialmente,
> PJMS
>
> Em ter., 15 de nov. de 2022 às 16:00, Anderson Torres <
> torres.anderson...@gmail.com> escreveu:
>
>>
>>
>> Em ter, 15 de nov de 2022 14:33, Pedro José 
>> escreveu:
>>
>>> Boa tarde!
>>> Para os |Naturais, temos os postulados de Peano.
>>>
>>> Para os Inteiros há alguma formalização?
>>>
>>
>> invente uma!
>>
>> Pode ser por exemplo o conjunto de pares (p,q) tais que p-q é constante.
>>
>> ou melhor (p1,q1)=(p2,q2) se e só se p1+q2=p2+q1.
>>
>>
>>> Acho pobre dizer que é necessário ter outros números devido ao problema
>>> de fechamento nos naturais para a subtração que é fato e daí introduzir os
>>> simétricos que são inteiros e ainda não foram caracterizados.
>>>
>>> No meu antigo ginásio aprendi que os Reais era a união dos conjuntos
>>> disjuntos irracionais e racionais. Os racionais haviam sido bem definidos.
>>> Aí questionei e o que são irracionais? resposta: são os Reais que não são
>>> racionais, os que não podem ser escritos na forma p/q p e q inteiros e
>>> q<>0. Mas me deram um tombo. Definiram os |Reais com base nos irracionais e
>>> os irracionais com base nos |Reais. 3 +2i também não pode ser inscrito na
>>> forma p/q. Só mais tarde no científico, é que meu professor definiu
>>> irracional como um número que não podia ser escrito na forma p/q e cuja
>>> representação decimal tinha uma infinidade de algarismos, sem haver uma
>>> periodicidade.
>>> Na época foi o maior nó que tive com a matemática. O mestre demonstrou
>>> que os racionais eram densos, mas entre eles ainda cabiam os irracionais.
>>> Não satisfeito mostrou que os racionais eram enumeráveis e por absurdo
>>> mostrou que os |Reais não. Não satisfeito mostrou que a cardinalidade do
>>> intervalo [0,1] era maior que a dos |Naturais. Não conseguia conceber que
>>> havia um infinito maior que outro. Outra coisa que demorei a aceitar,mesmo
>>> vendo a bijeção, era que os inteiros e naturais tinham a mesma
>>> cardinalidade. Na minha cabeça, os inteiros têm todos os naturais ainda
>>> sobram os negativos, como é igual?
>>> Hoje, depois de velho, arrumei uma enteada, que muito me pergunta e
>>> estou enrolado. Para dar um ar de superioridade, questionei se conhecia os
>>> inteiros de Gaus, que 5 não era primo nos inteiros de Gaus. Estrepei-me, a
>>> danada foi pesquisar e me questiona sobre o que não tenho um domínio pleno.
>>> Em suma, como apresentei a ela os postulados de Peano para a
>>> caracterização dos Naturais, ela me cobra por algo semelhante para os
>>> Inteiros, e não sei responder.
>>> HELP! SOCORRO! AU SECOURS! AYUDA! AIUTO! HILFE!
>>> Cordialmente,
>>> PJMS
>>>
>>> --
>>> Esta mensagem foi verificada pelo sistema de antivírus e
>>> acredita-se estar livre de perigo.
>>
>>
>> --
>> Esta mensagem foi verificada pelo sistema de antivírus e
>> acredita-se estar livre de perigo.
>
>
> --
> Esta mensagem foi verificada pelo sistema de antivírus e
> acredita-se estar livre de perigo.

-- 
Esta mensagem foi verificada pelo sistema de antiv�rus e
 acredita-se estar livre de perigo.



[obm-l] Re: [obm-l] Re: [obm-l] Caracterização de Inteiros

2022-11-15 Por tôpico Pedro José
Obrigado a você e ao Cláudio. Mas não sou criativo para inventar. Mas já vi
que terei que fazer uma homotetia, para as classes de equivalência para
representar só como um número e não como um par, creio eu.

Cordialmente,
PJMS

Em ter., 15 de nov. de 2022 às 16:00, Anderson Torres <
torres.anderson...@gmail.com> escreveu:

>
>
> Em ter, 15 de nov de 2022 14:33, Pedro José 
> escreveu:
>
>> Boa tarde!
>> Para os |Naturais, temos os postulados de Peano.
>>
>> Para os Inteiros há alguma formalização?
>>
>
> invente uma!
>
> Pode ser por exemplo o conjunto de pares (p,q) tais que p-q é constante.
>
> ou melhor (p1,q1)=(p2,q2) se e só se p1+q2=p2+q1.
>
>
>> Acho pobre dizer que é necessário ter outros números devido ao problema
>> de fechamento nos naturais para a subtração que é fato e daí introduzir os
>> simétricos que são inteiros e ainda não foram caracterizados.
>>
>> No meu antigo ginásio aprendi que os Reais era a união dos conjuntos
>> disjuntos irracionais e racionais. Os racionais haviam sido bem definidos.
>> Aí questionei e o que são irracionais? resposta: são os Reais que não são
>> racionais, os que não podem ser escritos na forma p/q p e q inteiros e
>> q<>0. Mas me deram um tombo. Definiram os |Reais com base nos irracionais e
>> os irracionais com base nos |Reais. 3 +2i também não pode ser inscrito na
>> forma p/q. Só mais tarde no científico, é que meu professor definiu
>> irracional como um número que não podia ser escrito na forma p/q e cuja
>> representação decimal tinha uma infinidade de algarismos, sem haver uma
>> periodicidade.
>> Na época foi o maior nó que tive com a matemática. O mestre demonstrou
>> que os racionais eram densos, mas entre eles ainda cabiam os irracionais.
>> Não satisfeito mostrou que os racionais eram enumeráveis e por absurdo
>> mostrou que os |Reais não. Não satisfeito mostrou que a cardinalidade do
>> intervalo [0,1] era maior que a dos |Naturais. Não conseguia conceber que
>> havia um infinito maior que outro. Outra coisa que demorei a aceitar,mesmo
>> vendo a bijeção, era que os inteiros e naturais tinham a mesma
>> cardinalidade. Na minha cabeça, os inteiros têm todos os naturais ainda
>> sobram os negativos, como é igual?
>> Hoje, depois de velho, arrumei uma enteada, que muito me pergunta e estou
>> enrolado. Para dar um ar de superioridade, questionei se conhecia os
>> inteiros de Gaus, que 5 não era primo nos inteiros de Gaus. Estrepei-me, a
>> danada foi pesquisar e me questiona sobre o que não tenho um domínio pleno.
>> Em suma, como apresentei a ela os postulados de Peano para a
>> caracterização dos Naturais, ela me cobra por algo semelhante para os
>> Inteiros, e não sei responder.
>> HELP! SOCORRO! AU SECOURS! AYUDA! AIUTO! HILFE!
>> Cordialmente,
>> PJMS
>>
>> --
>> Esta mensagem foi verificada pelo sistema de antivírus e
>> acredita-se estar livre de perigo.
>
>
> --
> Esta mensagem foi verificada pelo sistema de antivírus e
> acredita-se estar livre de perigo.

-- 
Esta mensagem foi verificada pelo sistema de antiv�rus e
 acredita-se estar livre de perigo.



[obm-l] Re: [obm-l] Re: [obm-l] Fwd: Módulo

2022-08-14 Por tôpico Rubens Vilhena Fonseca
Uma coisa que você deve definir é a paridade de n. Vamos reescrever em
linguagem de congruências :
2^n==1 (mod 3). Sabendo que 2== -1 (mod 3), então  (-1)^n == 1 (mod 3). O
que só será verdade se n for par.
Então, para n = 2k, temos 4^k = 3x +1.  Por experimentação, você pode
concluir alguns pares (k, x) de solução, (0, 0);
(1, 1); (2, 5); (3, 21)...Então, seu trabalho é mostrar que o par (k,
(4^k-1)/3 ) é uma solução.

Em qui., 11 de ago. de 2022 às 17:38, Anderson Torres <
torres.anderson...@gmail.com> escreveu:

>
>
> Em qui, 11 de ago de 2022 16:12, Esaú Gomes 
> escreveu:
>
>> Alguém poderia me falar o que estudar mais especificamente na questão
>> abaixo?
>>
>> Para quais valores naturais  de *n* e *x*, existe solução
>> 2^n = 3x + 1.
>>
>
>   Provas antigas.
>
> Esses problemas são resolvidos geralmente apelando para fatos padrão de
> congruências, em especial potenciação, ordem etc.
>
> E, no geral, a melhor maneira de entender e aplicar estes fatos é mediante
> treino, treino e mais treino.
>
> --
>> Esta mensagem foi verificada pelo sistema de antivírus e
>> acredita-se estar livre de perigo.
>
>
> --
> Esta mensagem foi verificada pelo sistema de antivírus e
> acredita-se estar livre de perigo.

-- 
Esta mensagem foi verificada pelo sistema de antiv�rus e
 acredita-se estar livre de perigo.



[obm-l] Re: [obm-l] Re: [obm-l] Função phi de Euler

2022-07-15 Por tôpico Anderson Torres
Em qui, 14 de jul de 2022 12:19, Esdras Muniz 
escreveu:

> Quis dizer φ(p)=p-1.
>
> Em qui, 14 de jul de 2022 12:02, Esdras Muniz 
> escreveu:
>
>> Oi(o)=p-1, aí isso só vale se o primo for da firma 6k+1.
>>
>
phi(4+3)=7-1


>> Em qui, 14 de jul de 2022 11:52, Rubens Vilhena Fonseca <
>> rubens.vilhen...@gmail.com> escreveu:
>>
>>> Saudações a todos da lista.
>>> É um fato que para primos p ímpares, a função de Euler phi(p)=p-1 é
>>> sempre um valor par.
>>> Os primos 7, 13, 19, 31, 37, 67, 73, 79, 97, ... tem valores pares
>>> múltiplos de 3.
>>> Existe algum caminho a tomar para determinar quando phi(p) = 3 .(2k)?
>>> Agradeço qualquer solução ou  informação ou indicação de leituras sobre
>>> o  problema.
>>> Att
>>>
>>>
>>> --
>>> Esta mensagem foi verificada pelo sistema de antivírus e
>>> acredita-se estar livre de perigo.
>>
>>
> --
> Esta mensagem foi verificada pelo sistema de antivírus e
> acredita-se estar livre de perigo.

-- 
Esta mensagem foi verificada pelo sistema de antiv�rus e
 acredita-se estar livre de perigo.



[obm-l] Re: [obm-l] Re: [obm-l] Re: [obm-l] Re: [obm-l] Re: [obm-l] Questão de probabilidade

2022-06-24 Por tôpico Rogerio Ponce
Otima explicacao!
Obrigado, Ralph!

PS: e sim, a provocacao foi pra voce mesmo!
:)

[]'s
Rogerio Ponce


On Wed, Jun 22, 2022 at 1:00 PM Ralph Costa Teixeira  wrote:
>
> Ponce está provocando a gente... senti que esta flecha tinha um bocado a 
> minha direção...  :D :D :D
>
> Olha, tem duas "visões" sobre o que "probabilidade" significa.
>
> A primeira vai na linha de que só podemos falar de probabilidade sobre coisas 
> que ainda não aconteceram. Vai nessa linha: se os eventos estão no passado, 
> então já aconteceram, já estão definidos, e não faz sentido dizer que tinha x 
> de chance de ser assim ou y de ser assado. Se você já jogou a moeda justa, 
> não é mais 50/50 --  é 100% de ser cara, ou 100% de ser coroa, dependendo do 
> que ocorreu. Quem pensa assim vai dizer que dado um certo evento (sempre no 
> futuro), ele tem uma probabilidade dada; se duas pessoas diferentes derem 
> duas probabilidades diferentes para o mesmo evento, uma delas errou.
>
> Outra linha diz que podemos falar de probabilidade sempre que houver 
> incerteza; não interessa o que aconteceu ou o que vai acontecer, o que 
> interessa é o que você SABE sobre o acontecimento. Se você jogou a moeda 
> justa mas eu não sei nada mais sobre o lançamento, continua sendo 50/50 
> **PARA MIM**. Probabilidade passa a ser um conceito sobre INFORMAÇÃO, não 
> sobre os fatos em si (a probabilidade não está na moeda, está no que você 
> sabe sobre a moeda). Quem pensa assim vai dizer que a probabilidade do evento 
> depende não apenas do evento em si, mas da informação que se tem em mãos. 
> Quem pensa assim admite que duas pessoas diferentes podem dar probabilidades 
> diferentes ao mesmo evento SE SOUBEREM FATOS DIFERENTES a respeito do evento, 
> ou seja, probabilidade passa a ser bastante "subjetivo".
>
> Eu talvez tenha descrito mal a primeira interpretação, pois sou ferrenho 
> defensor da segunda. Ela simplesmente engloba a primeira, porque você pode 
> ter informação parcial sobre fatos que ocorrem no futuro. E falar de 
> probabilidade para descrever incerteza presente ou passada é MUITO útil! Eu 
> quero poder expressar incerteza sobre fatos passados com frases do tipo 
> "fulano tem x% de probabilidade de ter cometido tal crime", ou "tem y% de 
> probabilidade de ter petróleo nesse poço", ou "tem z% de chance de eu ter 
> COVID"... Se você tem uma reação negativa a essas frases, lembre o que elas 
> realmente significam (na segunda interpretação): claro que ou o cara cometeu 
> o crime ou não, não faz sentido dizer que ele cometeu o crime x% das vezes em 
> que fizermos um experimento de ele cometer o crime... mas o que aquilo 
> significa é "com a informação que eu tenho, numa escala de 0 a 1, eu tenho 
> x/100 de certeza que fulano cometeu o crime". E "certeza baseada em informaç�!
 �es" é sim quantificável -- e satisfaz exatamente as leis das probabilidades 
com as quais concordamos. "Subjetivo" não significa "posso falar qualquer 
coisa", significa apenas que a conta pode variar de pessoa para pessoa... mas, 
de novo, SE ESSAS PESSOAS TIVEREM INFORMAÇÕES DIFERENTES sobre o evento.
>
> Abraço, Ralph.
>
> On Wed, Jun 22, 2022 at 12:09 PM Rogerio Ponce  wrote:
>>
>> Olá Pedro e pessoal da lista!
>>
>> Segundo a opinião do Pedro, nao faz sentido perguntar qual a probabilidade 
>> de Jose ter conseguido um 6 ao jogar o dado ontem, pois isso ja' aconteceu, 
>> e, portanto, ja' esta' definido.
>>
>> Sera' que e' isso mesmo?
>>
>> []'s
>> Rogerio Ponce
>>
>>
>> On Mon, Jun 20, 2022 at 9:45 PM Pedro José  wrote:
>>>
>>> Eu na minha humilde opinião creio que a probabilidade exista quando pode 
>>> ser uma coisa ou outra. No caso já é definido o que os animais são. Então 
>>> já está tudo errado. A questão seria viável se dessem esses limitantes para 
>>> uma criança que pintaria os desenhos dos animais. Aí sim há probabilidade.
>>>
>>> Em sáb., 18 de jun. de 2022 03:33, Rogerio Ponce  
>>> escreveu:

 Ola' Vanderlei e pessoal da lista!

 Sem perda de generalidade, podemos imaginar que vamos fazer o seguinte:

 - uma pintura preta em um dos caes, escolhido aleatoriamente

 - uma pintura "malhada" em um dos animais, escolhido aleatoriamente entre 
 os 7 animais nao pintados

 - duas pintura pretas, em dois animais, escolhidos aleatoriamente entre os 
 6 animais restantes,

 - quatro pinturas brancas nos 4 animais restantes


 Analisando a afirmacao 04, por exemplo, verificamos que, no segundo passo 
 (pintura malhada) existem 4 opcoes de cachorro e 3 opcoes de gato.

 Assim, a probabilidade de haver um cachorro malhado (4/7) e' maior que a 
 probabilidade de haver um gato malhado (3/7).
 Portanto, a afirmacao 04 esta' correta.
 (e o gabarito esta' errado).

 []'s
 Rogerio Ponce



 On Wed, Mar 16, 2022 at 8:08 AM Professor Vanderlei Nemitz 
  wrote:
>
> Bom dia!
> Na questão a seguir, do vestibular da UEM, 

[obm-l] Re: [obm-l] Re: [obm-l] Re: [obm-l] Re: [obm-l] Questão de probabilidade

2022-06-22 Por tôpico Ralph Costa Teixeira
Ponce está provocando a gente... senti que esta flecha tinha um bocado a
minha direção...  :D :D :D

Olha, tem duas "visões" sobre o que "probabilidade" significa.

A primeira vai na linha de que só podemos falar de probabilidade sobre
coisas que ainda não aconteceram. Vai nessa linha: se os eventos estão no
passado, então já aconteceram, já estão definidos, e não faz sentido dizer
que tinha x de chance de ser assim ou y de ser assado. Se você já jogou a
moeda justa, não é mais 50/50 --  é 100% de ser cara, ou 100% de ser coroa,
dependendo do que ocorreu. Quem pensa assim vai dizer que dado um certo
evento (sempre no futuro), ele tem uma probabilidade dada; se duas pessoas
diferentes derem duas probabilidades diferentes para o mesmo evento, uma
delas errou.

Outra linha diz que podemos falar de probabilidade sempre que houver
incerteza; não interessa o que aconteceu ou o que vai acontecer, o que
interessa é o que você SABE sobre o acontecimento. Se você jogou a moeda
justa mas eu não sei nada mais sobre o lançamento, continua sendo 50/50
**PARA MIM**. Probabilidade passa a ser um conceito sobre INFORMAÇÃO, não
sobre os fatos em si (a probabilidade não está na moeda, está no que você
sabe sobre a moeda). Quem pensa assim vai dizer que a probabilidade do
evento depende não apenas do evento em si, mas da informação que se tem em
mãos. Quem pensa assim admite que duas pessoas diferentes podem dar
probabilidades diferentes ao mesmo evento SE SOUBEREM FATOS DIFERENTES a
respeito do evento, ou seja, probabilidade passa a ser bastante "subjetivo".

Eu talvez tenha descrito mal a primeira interpretação, pois sou ferrenho
defensor da segunda. Ela simplesmente engloba a primeira, porque você pode
ter informação parcial sobre fatos que ocorrem no futuro. E falar de
probabilidade para descrever incerteza presente ou passada é MUITO útil! Eu
quero poder expressar incerteza sobre fatos passados com frases do tipo
"fulano tem x% de probabilidade de ter cometido tal crime", ou "tem y% de
probabilidade de ter petróleo nesse poço", ou "tem z% de chance de eu ter
COVID"... Se você tem uma reação negativa a essas frases, lembre o que elas
realmente significam (na segunda interpretação): claro que ou o cara
cometeu o crime ou não, não faz sentido dizer que ele cometeu o crime x%
das vezes em que fizermos um experimento de ele cometer o crime... mas o
que aquilo significa é "com a informação que eu tenho, numa escala de 0 a
1, eu tenho x/100 de certeza que fulano cometeu o crime". E "certeza
baseada em informações" é sim quantificável -- e satisfaz exatamente as
leis das probabilidades com as quais concordamos. "Subjetivo" não significa
"posso falar qualquer coisa", significa apenas que a conta pode variar de
pessoa para pessoa... mas, de novo, SE ESSAS PESSOAS TIVEREM INFORMAÇÕES
DIFERENTES sobre o evento.

Abraço, Ralph.

On Wed, Jun 22, 2022 at 12:09 PM Rogerio Ponce  wrote:

> Olá Pedro e pessoal da lista!
>
> Segundo a opinião do Pedro, nao faz sentido perguntar qual a probabilidade
> de Jose ter conseguido um 6 ao jogar o dado ontem, pois isso ja' aconteceu,
> e, portanto, ja' esta' definido.
>
> Sera' que e' isso mesmo?
>
> []'s
> Rogerio Ponce
>
>
> On Mon, Jun 20, 2022 at 9:45 PM Pedro José  wrote:
>
>> Eu na minha humilde opinião creio que a probabilidade exista quando pode
>> ser uma coisa ou outra. No caso já é definido o que os animais são. Então
>> já está tudo errado. A questão seria viável se dessem esses limitantes para
>> uma criança que pintaria os desenhos dos animais. Aí sim há probabilidade.
>>
>> Em sáb., 18 de jun. de 2022 03:33, Rogerio Ponce da Silva <
>> abrlw...@gmail.com> escreveu:
>>
>>> Ola' Vanderlei e pessoal da lista!
>>>
>>> Sem perda de generalidade, podemos imaginar que vamos fazer o seguinte:
>>>
>>> - uma pintura preta em um dos caes, escolhido aleatoriamente
>>>
>>> - uma pintura "malhada" em um dos animais, escolhido aleatoriamente
>>> entre os 7 animais nao pintados
>>>
>>> - duas pintura pretas, em dois animais, escolhidos aleatoriamente entre
>>> os 6 animais restantes,
>>>
>>> - quatro pinturas brancas nos 4 animais restantes
>>>
>>>
>>> Analisando a afirmacao 04, por exemplo, verificamos que, no segundo
>>> passo (pintura malhada) existem 4 opcoes de cachorro e 3 opcoes de gato.
>>>
>>> Assim, a probabilidade de haver um cachorro malhado (4/7) e' maior que a
>>> probabilidade de haver um gato malhado (3/7).
>>> Portanto, a afirmacao 04 esta' correta.
>>> (e o gabarito esta' errado).
>>>
>>> []'s
>>> Rogerio Ponce
>>>
>>>
>>>
>>> On Wed, Mar 16, 2022 at 8:08 AM Professor Vanderlei Nemitz <
>>> vanderma...@gmail.com> wrote:
>>>
 Bom dia!
 Na questão a seguir, do vestibular da UEM, penso que o espaço amostral
 tem 105 elementos, pois um cachorro é preto (desconsideramos esse). Porém,
 com esse pensamento, não consigo obter o gabarito, que diz que 02 e 16 são
 corretas.
 Alguém poderia ajudar?
 Muito obrigado!

 *Em um pet shop há 3 gatos e 5 cães. 

[obm-l] Re: [obm-l] Re: [obm-l] Re: [obm-l] Questão de probabilidade

2022-06-22 Por tôpico Rogerio Ponce
Olá Pedro e pessoal da lista!

Segundo a opinião do Pedro, nao faz sentido perguntar qual a probabilidade
de Jose ter conseguido um 6 ao jogar o dado ontem, pois isso ja' aconteceu,
e, portanto, ja' esta' definido.

Sera' que e' isso mesmo?

[]'s
Rogerio Ponce


On Mon, Jun 20, 2022 at 9:45 PM Pedro José  wrote:

> Eu na minha humilde opinião creio que a probabilidade exista quando pode
> ser uma coisa ou outra. No caso já é definido o que os animais são. Então
> já está tudo errado. A questão seria viável se dessem esses limitantes para
> uma criança que pintaria os desenhos dos animais. Aí sim há probabilidade.
>
> Em sáb., 18 de jun. de 2022 03:33, Rogerio Ponce da Silva <
> abrlw...@gmail.com> escreveu:
>
>> Ola' Vanderlei e pessoal da lista!
>>
>> Sem perda de generalidade, podemos imaginar que vamos fazer o seguinte:
>>
>> - uma pintura preta em um dos caes, escolhido aleatoriamente
>>
>> - uma pintura "malhada" em um dos animais, escolhido aleatoriamente entre
>> os 7 animais nao pintados
>>
>> - duas pintura pretas, em dois animais, escolhidos aleatoriamente entre
>> os 6 animais restantes,
>>
>> - quatro pinturas brancas nos 4 animais restantes
>>
>>
>> Analisando a afirmacao 04, por exemplo, verificamos que, no segundo passo
>> (pintura malhada) existem 4 opcoes de cachorro e 3 opcoes de gato.
>>
>> Assim, a probabilidade de haver um cachorro malhado (4/7) e' maior que a
>> probabilidade de haver um gato malhado (3/7).
>> Portanto, a afirmacao 04 esta' correta.
>> (e o gabarito esta' errado).
>>
>> []'s
>> Rogerio Ponce
>>
>>
>>
>> On Wed, Mar 16, 2022 at 8:08 AM Professor Vanderlei Nemitz <
>> vanderma...@gmail.com> wrote:
>>
>>> Bom dia!
>>> Na questão a seguir, do vestibular da UEM, penso que o espaço amostral
>>> tem 105 elementos, pois um cachorro é preto (desconsideramos esse). Porém,
>>> com esse pensamento, não consigo obter o gabarito, que diz que 02 e 16 são
>>> corretas.
>>> Alguém poderia ajudar?
>>> Muito obrigado!
>>>
>>> *Em um pet shop há 3 gatos e 5 cães. Sabemos que 3 desses animais são
>>> pretos, 4 são brancos e 1 é malhado. Além disso, pelo menos 1 cachorro é
>>> preto. Assinale o que for correto. *
>>> *01) A probabilidade de haver exatamente 1 cachorro preto é de 1/6. *
>>> *02) A probabilidade de haver pelo menos 1 gato branco e pelo menos 2
>>> cachorros brancos é de 2/3.*
>>> *04) A probabilidade de haver um cachorro malhado é maior do que a
>>> probabilidade de haver um gato malhado. *
>>> *08) Se um animal for escolhido ao acaso, a probabilidade de ele ser um
>>> cachorro preto é de 1/8. *
>>> *16) Se um animal for escolhido ao acaso, a probabilidade de ele ser um
>>> gato malhado é de 1/16.   *
>>>
>>> --
>>> Esta mensagem foi verificada pelo sistema de antivírus e
>>> acredita-se estar livre de perigo.
>>
>>
>> --
>> Esta mensagem foi verificada pelo sistema de antivírus e
>> acredita-se estar livre de perigo.
>
>
> --
> Esta mensagem foi verificada pelo sistema de antivírus e
> acredita-se estar livre de perigo.

-- 
Esta mensagem foi verificada pelo sistema de antiv�rus e
 acredita-se estar livre de perigo.



[obm-l] Re: [obm-l] Re: [obm-l] Questão de probabilidade

2022-06-20 Por tôpico Pedro José
Eu na minha humilde opinião creio que a probabilidade exista quando pode
ser uma coisa ou outra. No caso já é definido o que os animais são. Então
já está tudo errado. A questão seria viável se dessem esses limitantes para
uma criança que pintaria os desenhos dos animais. Aí sim há probabilidade.

Em sáb., 18 de jun. de 2022 03:33, Rogerio Ponce da Silva <
abrlw...@gmail.com> escreveu:

> Ola' Vanderlei e pessoal da lista!
>
> Sem perda de generalidade, podemos imaginar que vamos fazer o seguinte:
>
> - uma pintura preta em um dos caes, escolhido aleatoriamente
>
> - uma pintura "malhada" em um dos animais, escolhido aleatoriamente entre
> os 7 animais nao pintados
>
> - duas pintura pretas, em dois animais, escolhidos aleatoriamente entre os
> 6 animais restantes,
>
> - quatro pinturas brancas nos 4 animais restantes
>
>
> Analisando a afirmacao 04, por exemplo, verificamos que, no segundo passo
> (pintura malhada) existem 4 opcoes de cachorro e 3 opcoes de gato.
>
> Assim, a probabilidade de haver um cachorro malhado (4/7) e' maior que a
> probabilidade de haver um gato malhado (3/7).
> Portanto, a afirmacao 04 esta' correta.
> (e o gabarito esta' errado).
>
> []'s
> Rogerio Ponce
>
>
>
> On Wed, Mar 16, 2022 at 8:08 AM Professor Vanderlei Nemitz <
> vanderma...@gmail.com> wrote:
>
>> Bom dia!
>> Na questão a seguir, do vestibular da UEM, penso que o espaço amostral
>> tem 105 elementos, pois um cachorro é preto (desconsideramos esse). Porém,
>> com esse pensamento, não consigo obter o gabarito, que diz que 02 e 16 são
>> corretas.
>> Alguém poderia ajudar?
>> Muito obrigado!
>>
>> *Em um pet shop há 3 gatos e 5 cães. Sabemos que 3 desses animais são
>> pretos, 4 são brancos e 1 é malhado. Além disso, pelo menos 1 cachorro é
>> preto. Assinale o que for correto. *
>> *01) A probabilidade de haver exatamente 1 cachorro preto é de 1/6. *
>> *02) A probabilidade de haver pelo menos 1 gato branco e pelo menos 2
>> cachorros brancos é de 2/3.*
>> *04) A probabilidade de haver um cachorro malhado é maior do que a
>> probabilidade de haver um gato malhado. *
>> *08) Se um animal for escolhido ao acaso, a probabilidade de ele ser um
>> cachorro preto é de 1/8. *
>> *16) Se um animal for escolhido ao acaso, a probabilidade de ele ser um
>> gato malhado é de 1/16.   *
>>
>> --
>> Esta mensagem foi verificada pelo sistema de antivírus e
>> acredita-se estar livre de perigo.
>
>
> --
> Esta mensagem foi verificada pelo sistema de antivírus e
> acredita-se estar livre de perigo.

-- 
Esta mensagem foi verificada pelo sistema de antiv�rus e
 acredita-se estar livre de perigo.



[obm-l] Re: [obm-l] Re: [obm-l] Dúvida e ajuda.

2022-04-08 Por tôpico Pedro José
Grato a todos!
Já, já tenho de voltar ao trabalho.
Depois dou uma olhada.
Mas achei a demonstração usando casa de pombos, simples e prática.
Já que tem de haver um p/q com pp temos w=x+p/q,
onde x é a parte inteira de w/q, então pq e os restos só podem q-1, uma hora tem de
repetir e aí volta a sequência.
Mas saindo do trabalho dou uma olhada.
Mais uma vez, minha gratidão.

Cordialmente,
PJMS



Em sex., 8 de abr. de 2022 às 13:02, Claudio Buffara <
claudio.buff...@gmail.com> escreveu:

> A volta é fácil também: ao calcular a representação decimal de a/b (a e b
> naturais), nas divisões sucessivas por b só existem b-1 restos possíveis
> (resto = 0 em alguma etapa implica numa decimal finita) e, portanto, após
> não mais do que b-1 divisões, um resto vai se repetir, marcando o início de
> um novo período na representação decimal.
>
> Agora, suponha que  X =
> 0,123456789112233445566778899111222333444555666777888999... seja racional.
> Então existirão n e p naturais tais que, a partir da n-ésima casa decimal
> (1/10^n), os algarismos de X vão se repetir numa sequência com período p.
>
> Mas, pela lei de formação de X, vai existir uma sequência de n+p+1
> algarismos iguais a 1, e esta sequência vai começar após a n-ésima casa
> decimal.
> Ou seja, a sequência vai estar incluída na parte periódica da
> representação decimal de X.
> Mas como o período é p, isso implica que a parte periódica teria que
> ser 111..11 (p algarismos 1) ==> contradição à lei de formação de X.
>
> []s,
> Claudio.
>
>
> On Fri, Apr 8, 2022 at 11:17 AM Pedro José  wrote:
>
>> Bom dia!
>> Posso concluir que um número representado por uma infinidade de
>> algarismos decimais é racional se e somente se tem um período de repetições
>> desses algarismos?
>> A ida é fácil se tiver o período é racional.
>> Já a volta não sei se é verdade e se for há como provar?
>>
>> Meu objetivo primário é saber se:
>> 0,123456789112233445566778899111222333444555666777888999... é racional.
>> As reticências se referem ao aumento de mais um algarismo repetido a cada
>> sequência, ou seja a primeira aparição de 1 será 1, a 2a 11 a 3a 111 e
>> assim sucessivamente, o mesmo vale para os demais algarismos.
>>
>> Alguém poderia me ajudar?
>> Grato,
>> PJMS
>>
>> --
>> Esta mensagem foi verificada pelo sistema de antivírus e
>> acredita-se estar livre de perigo.
>
>
> --
> Esta mensagem foi verificada pelo sistema de antivírus e
> acredita-se estar livre de perigo.

-- 
Esta mensagem foi verificada pelo sistema de antiv�rus e
 acredita-se estar livre de perigo.



Re: [obm-l] Re: Polinomio

2022-01-29 Por tôpico Israel Meireles Chrisostomo
Muito obrigado pessoal

Em sáb., 29 de jan. de 2022 19:14, Artur Costa Steiner <
artur.costa.stei...@gmail.com> escreveu:

> Creio que vc se refere a polinômios reais.
>
> Se P tiver grau par positivo  então:
>   Se o coeficiente líder for positivo, P tem  um mínimo global. Se for
> negativo,  P tem um máximo global.
>
> Se P tiver  grau ímpar, P não tem mínimo nem máximo globais.
>
> Limitado inferior e superiormente, só se P for constante
>
> Artur
>
>
>
> Em sáb., 29 de jan. de 2022 às 18:41, Esdras Muniz <
> esdrasmunizm...@gmail.com> escreveu:
>
>> O único polinômio limitado é o constante.
>>
>> Em sáb, 29 de jan de 2022 14:03, Carlos Juarez <
>> carlosjuarezmart...@gmail.com> escreveu:
>>
>>> k=p(c)+1 não vale sempre?
>>>
>>> Em sáb, 29 de jan de 2022 09:27, Israel Meireles Chrisostomo <
>>> israelmchrisost...@gmail.com> escreveu:
>>>
 Desculpe me o que eu quis dizer é que dado um c real existe um k
 positivo tal que p(c)>>>
 Em sáb., 29 de jan. de 2022 09:12, Israel Meireles Chrisostomo <
 israelmchrisost...@gmail.com> escreveu:

> Olá pessoal.Eu gostaria de saber se um polinomio é limitado, isto é,
> dado P(x) existe um k positivo tal que P(x)

 --
 Esta mensagem foi verificada pelo sistema de antivírus e
 acredita-se estar livre de perigo.
>>>
>>>
>>> --
>>> Esta mensagem foi verificada pelo sistema de antivírus e
>>> acredita-se estar livre de perigo.
>>
>>
>> --
>> Esta mensagem foi verificada pelo sistema de antivírus e
>> acredita-se estar livre de perigo.
>
>
> --
> Esta mensagem foi verificada pelo sistema de antivírus e
> acredita-se estar livre de perigo.

-- 
Esta mensagem foi verificada pelo sistema de antiv�rus e
 acredita-se estar livre de perigo.



Re: [obm-l] Re: Polinomio

2022-01-29 Por tôpico Artur Costa Steiner
Creio que vc se refere a polinômios reais.

Se P tiver grau par positivo  então:
  Se o coeficiente líder for positivo, P tem  um mínimo global. Se for
negativo,  P tem um máximo global.

Se P tiver  grau ímpar, P não tem mínimo nem máximo globais.

Limitado inferior e superiormente, só se P for constante

Artur



Em sáb., 29 de jan. de 2022 às 18:41, Esdras Muniz <
esdrasmunizm...@gmail.com> escreveu:

> O único polinômio limitado é o constante.
>
> Em sáb, 29 de jan de 2022 14:03, Carlos Juarez <
> carlosjuarezmart...@gmail.com> escreveu:
>
>> k=p(c)+1 não vale sempre?
>>
>> Em sáb, 29 de jan de 2022 09:27, Israel Meireles Chrisostomo <
>> israelmchrisost...@gmail.com> escreveu:
>>
>>> Desculpe me o que eu quis dizer é que dado um c real existe um k
>>> positivo tal que p(c)>>
>>> Em sáb., 29 de jan. de 2022 09:12, Israel Meireles Chrisostomo <
>>> israelmchrisost...@gmail.com> escreveu:
>>>
 Olá pessoal.Eu gostaria de saber se um polinomio é limitado, isto é,
 dado P(x) existe um k positivo tal que P(x)>>>
>>>
>>> --
>>> Esta mensagem foi verificada pelo sistema de antivírus e
>>> acredita-se estar livre de perigo.
>>
>>
>> --
>> Esta mensagem foi verificada pelo sistema de antivírus e
>> acredita-se estar livre de perigo.
>
>
> --
> Esta mensagem foi verificada pelo sistema de antivírus e
> acredita-se estar livre de perigo.

-- 
Esta mensagem foi verificada pelo sistema de antiv�rus e
 acredita-se estar livre de perigo.



Re: [obm-l] Re: Polinomio

2022-01-29 Por tôpico Esdras Muniz
O único polinômio limitado é o constante.

Em sáb, 29 de jan de 2022 14:03, Carlos Juarez <
carlosjuarezmart...@gmail.com> escreveu:

> k=p(c)+1 não vale sempre?
>
> Em sáb, 29 de jan de 2022 09:27, Israel Meireles Chrisostomo <
> israelmchrisost...@gmail.com> escreveu:
>
>> Desculpe me o que eu quis dizer é que dado um c real existe um k positivo
>> tal que p(c)>
>> Em sáb., 29 de jan. de 2022 09:12, Israel Meireles Chrisostomo <
>> israelmchrisost...@gmail.com> escreveu:
>>
>>> Olá pessoal.Eu gostaria de saber se um polinomio é limitado, isto é,
>>> dado P(x) existe um k positivo tal que P(x)>>
>>
>> --
>> Esta mensagem foi verificada pelo sistema de antivírus e
>> acredita-se estar livre de perigo.
>
>
> --
> Esta mensagem foi verificada pelo sistema de antivírus e
> acredita-se estar livre de perigo.

-- 
Esta mensagem foi verificada pelo sistema de antiv�rus e
 acredita-se estar livre de perigo.



Re: [obm-l] Re: Polinomio

2022-01-29 Por tôpico Carlos Juarez
k=p(c)+1 não vale sempre?

Em sáb, 29 de jan de 2022 09:27, Israel Meireles Chrisostomo <
israelmchrisost...@gmail.com> escreveu:

> Desculpe me o que eu quis dizer é que dado um c real existe um k positivo
> tal que p(c)
> Em sáb., 29 de jan. de 2022 09:12, Israel Meireles Chrisostomo <
> israelmchrisost...@gmail.com> escreveu:
>
>> Olá pessoal.Eu gostaria de saber se um polinomio é limitado, isto é, dado
>> P(x) existe um k positivo tal que P(x)>
>
> --
> Esta mensagem foi verificada pelo sistema de antivírus e
> acredita-se estar livre de perigo.

-- 
Esta mensagem foi verificada pelo sistema de antiv�rus e
 acredita-se estar livre de perigo.



[obm-l] Re: [obm-l] Re: [obm-l] Experiência mental

2022-01-26 Por tôpico Israel Meireles Chrisostomo
Muito obrigado, bem que eu achei meio estranho ninguém ter percebido kkk

Em qua., 26 de jan. de 2022 10:40, Fernando Villar 
escreveu:

>
> Olá Israel. A primeira vez que vi também tive essa impressão, mas ao ler o
> livro descobri que os seres de Planolandia identificam uns aos outros por
> meio do tato, identificando os ângulos. A referência ao formato é para
> estabelecer uma correspondência com o que conhecemos.
> O livro é muito bom, propõe uma discussão da hierarquia social vigente no
> século XIX.
>
> Abraços e uma excelente semana para você.
>
> Fernando Villar
>
>
> Em qua., 26 de jan. de 2022 às 09:20, Israel Meireles Chrisostomo <
> israelmchrisost...@gmail.com> escreveu:
>
>> olá pessoal, eu estava no youtube assistindo a um vídeo de Carl Sagan
>> falando sobre a planolandia.Para quem ñ sabe, a planolandia é uma
>> experiencia mental que considera seres em universos planos.Sem delongas, eu
>> refuto a ideia de que os habitantes de tal universo enxerguem figuras
>> geométricas planas, como o triângulo, quadrado, retângulo, circulo...O
>> argumento é bem simples: só é possível ver figuras planas fora do plano,
>> mas quem está no plano só consegue ver linhas retas.
>>
>> --
>> Esta mensagem foi verificada pelo sistema de antivírus e
>> acredita-se estar livre de perigo.
>
> --
> Fernando Villar
>
>
> --
> Esta mensagem foi verificada pelo sistema de antivírus e
> acredita-se estar livre de perigo.

-- 
Esta mensagem foi verificada pelo sistema de antiv�rus e
 acredita-se estar livre de perigo.



[obm-l] Re: [obm-l] Re: [obm-l] Invertíveis e Divisores de Zero

2021-11-30 Por tôpico Pedro Júnior
Sim...

Em ter., 30 de nov. de 2021 às 15:21, Claudio Buffara <
claudio.buff...@gmail.com> escreveu:

> Z_4 x Z_5 é isomorfo a Z_20.
> Talvez isso ajude.
>
> On Tue, Nov 30, 2021 at 2:33 PM Pedro Júnior 
> wrote:
>
>> Quem puder ajudar...
>> Encontre todos os invertíveis e divisores de zero em Z_4 x Z_5.
>>
>>
>>
>> --
>> Esta mensagem foi verificada pelo sistema de antivírus e
>> acredita-se estar livre de perigo.
>
>
> --
> Esta mensagem foi verificada pelo sistema de antivírus e
> acredita-se estar livre de perigo.



-- 

Pedro Jerônimo S. de O. Júnior

Professor de Matemática

Geo João Pessoa – PB

-- 
Esta mensagem foi verificada pelo sistema de antiv�rus e
 acredita-se estar livre de perigo.



[obm-l] Re: [obm-l] Re: [obm-l] Valor máximo

2021-11-29 Por tôpico Artur Costa Steiner
>
> Se a, b e c são positivos e a^2+b^2+c^2 = 1, qual o valor máximo de
> (1-a)(1-b)(1-c)?
>
>> Desde já agradeço
>>
>
Podemos usar multiplicadores de Lagrange. Seja

f(a,b,c,L) = (1-a)(1-b)(1-c) -L(a^2 + b^2 + c^2 - 1)

Tomando as derivadas parciais de f com relação a a, b, c e L e igualando a
0,  obtemos

2a = L(1- b)(1 - c)
2b= L(1- a)(1 - c)
2c = L(1- a)(1 -c)
a^2+b^2+c^2 = 1

Se L <> 0 e se  nenhuma variável for 1, obtemos

a/(1 - b) = b/(1 - a), sendo as outras 2 equações permutações circulares da
1a. Segue-se que

a - a^2 = b - b^2
a - b = (a - b)(a + b) ==> a + b = 1. Considerando as outras 2 equações
chegamos a

a = b = c = raiz(3)/3. Isto leva a que a que (1 - a)(1-b)(1- c) = (1 -
raiz(3)/3)^3

Se L = 0 as equações conduzem às ternas (1,0,0), (0,1,0) e a (0, 0, 1) para
as quais (1 - a)(1-b)(1- c) = 0 < (1 - raiz(3)/3)^3

Como se trata de função contínua em conjunto compacto, as ternas acima dão
o mínimo absoluto e (raiz(3)/3,raiz(3)/3), raiz(3)/3) dá o máximo absoluto
no valor já citado

Nesse problemas geralmente há tambén uma solução baseada em desugualdades
como MA, MG, etc

Artur




















Km

-- 
Esta mensagem foi verificada pelo sistema de antiv�rus e
 acredita-se estar livre de perigo.



[obm-l] Re: [obm-l] Re: [obm-l] Solução do problema de fatoração (quebra do RSA)

2021-11-25 Por tôpico Rodrigo Ângelo
Sobre o passo 6, você quis dizer aleatório com distribuição uniforme?

On Thu, Nov 25, 2021, 09:59 Eric Campos Bastos Guedes <
ebastosgue...@gmail.com> wrote:

>
> Estou trabalhando num algoritmo melhor, mas preciso de acesso a um
> computador com o software de computação algébrica  Maple que é o que eu sei
> usar. Espero ter o retorno de pessoas que sabem mais do que eu.
>
> Em dom., 14 de nov. de 2021 12:58, Claudio Buffara <
> claudio.buff...@gmail.com> escreveu:
>
>> Por que vc não testa?
>>
>> On Sun, Nov 14, 2021 at 9:53 AM Eric Campos Bastos Guedes <
>> ebastosgue...@gmail.com> wrote:
>>
>>> Eu preciso de um retorno sobre o meu algoritmo que quebra o RSA
>>> resolvendo o problema de fatoracao.
>>>
>>> O Passo 8 talvez possa ser substituido por: PASSO 8': P = P(P+C)
>>>
>>> Em seg., 6 de set. de 2021 07:47, Eric Campos Bastos Guedes <
>>> ebastosgue...@gmail.com> escreveu:
>>>
 Aparentemente minha conexão está raqueada por gente do Bolsonaro e eu
 não estou recebendo respostas para minha postagem e também não estou
 conseguindo acessar os sites de discussão sobre o RSA. Há pessoas se
 fazendo passar por mim também.

 -- Forwarded message -
 De: Eric Campos Bastos Guedes 
 Date: sáb., 4 de set. de 2021 00:33
 Subject: [obm-l]
 To: 


 Olá a todos. Gostaria de pedir licença para que vocês avaliem um
 algoritmo que eu fiz para fatorar números grandes com fatores primos também
 grandes. Eu acredito que esse algoritmo quebre o RSA

 O algoritmo é o seguinte:

 PASSO 1: faça A=3
 PASSO 2: N é o inteiro a ser fatorado
 PASSO 3: M = N**16 (potência)
 PASSO 4: faça A=A+1
 PASSO 5: faça P=A
 PASSO 6: faça B=número aleatório entre 0 e 1
 PASSO 7: se B eh maior que 0.5 faça C=1 senão faça C = -1
 PASSO 8: faça P=(PP+CP)/2=P(P+C)/2
 PASSO 9: se P eh menor que M  vá para o PASSO 6
 PASSO 10: se mdc(P, N) = 1 faça M=MM e vá para o PASSO 4
 PASSO 11: se mdc(P, N) = N faça M = raiz quadrada de M e vá para o
 PASSO 4
 PASSO 12: mdc(P, N) é fator de N
 FIM

 AUTOR: ERIC CAMPOS BASTOS GUEDES  (DIA 4 DE SETEMBRO)

 Creio ter resolvido o problema de fatoração. Alguém pode verificar isso
 para mim.

 --
 Esta mensagem foi verificada pelo sistema de antivírus e
 acredita-se estar livre de perigo.

>>>
>>> --
>>> Esta mensagem foi verificada pelo sistema de antivírus e
>>> acredita-se estar livre de perigo.
>>
>>
>> --
>> Esta mensagem foi verificada pelo sistema de antivírus e
>> acredita-se estar livre de perigo.
>
>
> --
> Esta mensagem foi verificada pelo sistema de antivírus e
> acredita-se estar livre de perigo.

-- 
Esta mensagem foi verificada pelo sistema de antiv�rus e
 acredita-se estar livre de perigo.



[obm-l] Re: [obm-l] Re: [obm-l] Dúvida

2021-11-22 Por tôpico Pedro José
Boa tarde!

Grato, pela ajuda!
Não conheço.
Vou abrir um leque de estudo para tentar entender!
Valeu a curiosidade, com o que cheguei consegui matar o problema.
Genericamente, consegui que a solução levaria a uma expressão que era um
quadrado perfeito,esse era o objetivo. Só que me deu curiosidade, quanto a
resolução. Vou me enveredar no tema.

Cordialmente,
PJMS.

Em ter., 16 de nov. de 2021 às 17:29, Prof. Douglas Oliveira <
profdouglaso.del...@gmail.com> escreveu:

> Equação de Pell
>
> Em seg., 15 de nov. de 2021 13:36, Pedro José 
> escreveu:
>
>> Boa tarde!
>>
>> Alguém saberia como resolver a seguinte equação:
>>
>> x^2-7y^2=1, x,y em Z?
>>
>> Fiz a-7b=1 e achei a= 8 +7k e b=1 +K
>> Logo fica fácil que para k=-1 funciona x^2=1 e y^2=0.
>> Também funciona para k=8 x^2=64 e y^2=9.
>> Mas não sei nem como achar mais soluções nem como provar que só são essas.
>> Alguém poderia me dar uma orientação?
>>
>> Cordialmente,
>> PJMS
>>
>> --
>> Esta mensagem foi verificada pelo sistema de antivírus e
>> acredita-se estar livre de perigo.
>
>
> --
> Esta mensagem foi verificada pelo sistema de antivírus e
> acredita-se estar livre de perigo.

-- 
Esta mensagem foi verificada pelo sistema de antiv�rus e
 acredita-se estar livre de perigo.



[obm-l] Re: [obm-l] Re: [obm-l] Re: [obm-l] Re: matemática discreta

2021-09-20 Por tôpico Anderson Torres
Não consegui entender esse texto.

Em seg., 20 de set. de 2021 às 22:37, Israel Meireles Chrisostomo
 escreveu:
>
> Obrigado
>
> Em seg, 20 de set de 2021 22:00, Israel Meireles Chrisostomo 
>  escreveu:
>>
>> Tome n maior que n
>>
>> Em seg, 20 de set de 2021 20:49, Marcelo Salhab Brogliato 
>>  escreveu:
>>>
>>> Oi Israel,
>>>
>>> Não consegui entender a questão.
>>>
>>> Exemplo:
>>>
>>> n = 10, m = 3, Fib(10 - 3 + 1) = Fib(8) = 21
>>>
>>> (alpha**(2*n)) / (alpha**(n - m)) = alpha**(n + m) = 521.0019193787257
>>>
>>> Pela sua igualdade, alpha**(n + m) deveria ser 1/21, correto?
>>>
>>> Abraços,
>>> Marcelo
>>>
>>> Il giorno lun 20 set 2021 alle ore 15:54 Israel Meireles Chrisostomo 
>>>  ha scritto:

 já tentei de tudo, por favor me ajudem.

 Em seg., 20 de set. de 2021 às 19:39, Israel Meireles Chrisostomo 
  escreveu:
>
> Alguém poderia resolver o problema no link abaixo?
>
> https://mathoverflow.net/questions/404417/alpha2n-fracf-n-m1-alphan-m-1-how-to-prove-that-equality-is-true
>
> --
> Israel Meireles Chrisostomo



 --
 Israel Meireles Chrisostomo

 --
 Esta mensagem foi verificada pelo sistema de antivírus e
 acredita-se estar livre de perigo.
>>>
>>>
>>> --
>>> Esta mensagem foi verificada pelo sistema de antivírus e
>>> acredita-se estar livre de perigo.
>
>
> --
> Esta mensagem foi verificada pelo sistema de antivírus e
> acredita-se estar livre de perigo.

-- 
Esta mensagem foi verificada pelo sistema de antiv�rus e
 acredita-se estar livre de perigo.


=
Instru��es para entrar na lista, sair da lista e usar a lista em
http://www.mat.puc-rio.br/~obmlistas/obm-l.html
=


[obm-l] Re: [obm-l] Re: [obm-l] Re: matemática discreta

2021-09-20 Por tôpico Israel Meireles Chrisostomo
Obrigado

Em seg, 20 de set de 2021 22:00, Israel Meireles Chrisostomo <
israelmchrisost...@gmail.com> escreveu:

> Tome n maior que n
>
> Em seg, 20 de set de 2021 20:49, Marcelo Salhab Brogliato <
> msbro...@gmail.com> escreveu:
>
>> Oi Israel,
>>
>> Não consegui entender a questão.
>>
>> Exemplo:
>>
>> n = 10, m = 3, Fib(10 - 3 + 1) = Fib(8) = 21
>>
>> (alpha**(2*n)) / (alpha**(n - m)) = alpha**(n + m) = 521.0019193787257
>>
>> Pela sua igualdade, alpha**(n + m) deveria ser 1/21, correto?
>>
>> Abraços,
>> Marcelo
>>
>> Il giorno lun 20 set 2021 alle ore 15:54 Israel Meireles Chrisostomo <
>> israelmchrisost...@gmail.com> ha scritto:
>>
>>> já tentei de tudo, por favor me ajudem.
>>>
>>> Em seg., 20 de set. de 2021 às 19:39, Israel Meireles Chrisostomo <
>>> israelmchrisost...@gmail.com> escreveu:
>>>
 Alguém poderia resolver o problema no link abaixo?


 https://mathoverflow.net/questions/404417/alpha2n-fracf-n-m1-alphan-m-1-how-to-prove-that-equality-is-true

 --
 Israel Meireles Chrisostomo

>>>
>>>
>>> --
>>> Israel Meireles Chrisostomo
>>>
>>> --
>>> Esta mensagem foi verificada pelo sistema de antivírus e
>>> acredita-se estar livre de perigo.
>>
>>
>> --
>> Esta mensagem foi verificada pelo sistema de antivírus e
>> acredita-se estar livre de perigo.
>
>

-- 
Esta mensagem foi verificada pelo sistema de antiv�rus e
 acredita-se estar livre de perigo.



[obm-l] Re: [obm-l] Re: [obm-l] Re: matemática discreta

2021-09-20 Por tôpico Israel Meireles Chrisostomo
Tome n maior que n

Em seg, 20 de set de 2021 20:49, Marcelo Salhab Brogliato <
msbro...@gmail.com> escreveu:

> Oi Israel,
>
> Não consegui entender a questão.
>
> Exemplo:
>
> n = 10, m = 3, Fib(10 - 3 + 1) = Fib(8) = 21
>
> (alpha**(2*n)) / (alpha**(n - m)) = alpha**(n + m) = 521.0019193787257
>
> Pela sua igualdade, alpha**(n + m) deveria ser 1/21, correto?
>
> Abraços,
> Marcelo
>
> Il giorno lun 20 set 2021 alle ore 15:54 Israel Meireles Chrisostomo <
> israelmchrisost...@gmail.com> ha scritto:
>
>> já tentei de tudo, por favor me ajudem.
>>
>> Em seg., 20 de set. de 2021 às 19:39, Israel Meireles Chrisostomo <
>> israelmchrisost...@gmail.com> escreveu:
>>
>>> Alguém poderia resolver o problema no link abaixo?
>>>
>>>
>>> https://mathoverflow.net/questions/404417/alpha2n-fracf-n-m1-alphan-m-1-how-to-prove-that-equality-is-true
>>>
>>> --
>>> Israel Meireles Chrisostomo
>>>
>>
>>
>> --
>> Israel Meireles Chrisostomo
>>
>> --
>> Esta mensagem foi verificada pelo sistema de antivírus e
>> acredita-se estar livre de perigo.
>
>
> --
> Esta mensagem foi verificada pelo sistema de antivírus e
> acredita-se estar livre de perigo.

-- 
Esta mensagem foi verificada pelo sistema de antiv�rus e
 acredita-se estar livre de perigo.



[obm-l] Re: [obm-l] Re: matemática discreta

2021-09-20 Por tôpico Marcelo Salhab Brogliato
Oi Israel,

Não consegui entender a questão.

Exemplo:

n = 10, m = 3, Fib(10 - 3 + 1) = Fib(8) = 21

(alpha**(2*n)) / (alpha**(n - m)) = alpha**(n + m) = 521.0019193787257

Pela sua igualdade, alpha**(n + m) deveria ser 1/21, correto?

Abraços,
Marcelo

Il giorno lun 20 set 2021 alle ore 15:54 Israel Meireles Chrisostomo <
israelmchrisost...@gmail.com> ha scritto:

> já tentei de tudo, por favor me ajudem.
>
> Em seg., 20 de set. de 2021 às 19:39, Israel Meireles Chrisostomo <
> israelmchrisost...@gmail.com> escreveu:
>
>> Alguém poderia resolver o problema no link abaixo?
>>
>>
>> https://mathoverflow.net/questions/404417/alpha2n-fracf-n-m1-alphan-m-1-how-to-prove-that-equality-is-true
>>
>> --
>> Israel Meireles Chrisostomo
>>
>
>
> --
> Israel Meireles Chrisostomo
>
> --
> Esta mensagem foi verificada pelo sistema de antivírus e
> acredita-se estar livre de perigo.

-- 
Esta mensagem foi verificada pelo sistema de antiv�rus e
 acredita-se estar livre de perigo.



[obm-l] Re: [obm-l] Re: [obm-l] cálculo

2021-09-15 Por tôpico Israel Meireles Chrisostomo
Muito obrigado

Em qua, 15 de set de 2021 11:36, Esdras Muniz 
escreveu:

> O ponto é que tanto o conjunto dos números racionais quanto o conjunto dos
> números irracionais são densos em R. Portanto, para todo intervalo não
> degenerado, o máximo de f será 1 e o mínimo de f será zero. Daí, a integral
> superior será sempre maior que a integral inferior, portanto a função não é
> integravel.
>
> Em qua, 15 de set de 2021 00:11, Israel Meireles Chrisostomo <
> israelmchrisost...@gmail.com> escreveu:
>
>> Olá pessoal. eu estou me esforçando para entender esse exemplo do
>> guidorizzi, alguém poderia me explicar?Aqui vai:
>> Seja f uma função, tal que se x é racional então f igual a 1, se x é
>> irracional então f igual a zero. Mostre que a função não é riemann
>> integrável.
>>
>> --
>> Israel Meireles Chrisostomo
>>
>> --
>> Esta mensagem foi verificada pelo sistema de antivírus e
>> acredita-se estar livre de perigo.
>
>
> --
> Esta mensagem foi verificada pelo sistema de antivírus e
> acredita-se estar livre de perigo.

-- 
Esta mensagem foi verificada pelo sistema de antiv�rus e
 acredita-se estar livre de perigo.



[obm-l] Re: [obm-l] Re: [obm-l] Álgebra

2021-07-27 Por tôpico Anderson Torres
Em dom., 25 de jul. de 2021 às 15:23, Ralph Costa Teixeira
 escreveu:
>
> Sem ser muito formal: (a,b) e (c,d) sao dois vetores do plano, unitários e 
> ortogonais. Ou seja, um deles eh igual ao outro girado de 90 graus. Assim 
> (c,d)=(-b,a) ou (c,d)=(b,-a). De um jeito ou de outro, cd=-ab, ou seja, 
> resposta 0.
>
> On Sun, Jul 25, 2021 at 10:03 AM marcone augusto araújo borges 
>  wrote:
>>
>> a, b, c, d são números reais tais que a^2+b^2 = c^2 + d^2 = 1, ac + bd = 0. 
>> Calcule ab + cd
>> Desde já agradeço

Poderíamos escrever a=sinX, c=cosY. Assim sendo, b=cosX, d=sinY e daí
0=ac+bd=sinXcosY + cosXsinY = sin(X+Y), assim podemos usar X=-Y e daí
sinXcosX+sinYcosY = sinXcosX-sinXcosX=0
>>
>> --
>> Esta mensagem foi verificada pelo sistema de antivírus e
>> acredita-se estar livre de perigo.
>
>
> --
> Esta mensagem foi verificada pelo sistema de antivírus e
> acredita-se estar livre de perigo.

-- 
Esta mensagem foi verificada pelo sistema de antiv�rus e
 acredita-se estar livre de perigo.


=
Instru��es para entrar na lista, sair da lista e usar a lista em
http://www.mat.puc-rio.br/~obmlistas/obm-l.html
=


[obm-l] Re: [obm-l] Re: [obm-l] Álgebra

2021-07-25 Por tôpico Pacini Bores
 

Vi também assim : 

(ac+bd)(ad+bc) = cd(a^2+b^2)+ab(c^2+d^2). 

0= cd.1 + ab.1, logo ab+cd =0. 

É claro que a solução do Ralph é mais elegante... 

Abraços 

Pacini 

Em 25/07/2021 15:10, Ralph Costa Teixeira escreveu: 

> Sem ser muito formal: (a,b) e (c,d) sao dois vetores do plano, unitários e 
> ortogonais. Ou seja, um deles eh igual ao outro girado de 90 graus. Assim 
> (c,d)=(-b,a) ou (c,d)=(b,-a). De um jeito ou de outro, cd=-ab, ou seja, 
> resposta 0. 
> 
> On Sun, Jul 25, 2021 at 10:03 AM marcone augusto araújo borges 
>  wrote: 
> 
>> a, b, c, d são números reais tais que a^2+b^2 = c^2 + d^2 = 1, ac + bd = 0. 
>> Calcule ab + cd 
>> Desde já agradeço 
>> -- 
>> Esta mensagem foi verificada pelo sistema de antivírus e 
>> acredita-se estar livre de perigo.
> 
> -- 
> Esta mensagem foi verificada pelo sistema de antivrus e 
> acredita-se estar livre de perigo.

 
-- 
Esta mensagem foi verificada pelo sistema de antiv�rus e
 acredita-se estar livre de perigo.



[obm-l] Re: [obm-l] Re: [obm-l] Re: [obm-l] Re: [obm-l] Função

2021-05-29 Por tôpico Israel Meireles Chrisostomo
obrigado


Livre
de vírus. www.avast.com
.
<#DAB4FAD8-2DD7-40BB-A1B8-4E2AA1F9FDF2>

Em qua., 19 de mai. de 2021 às 10:56, Anderson Torres <
torres.anderson...@gmail.com> escreveu:

> Em seg., 26 de abr. de 2021 às 17:18, Israel Meireles Chrisostomo
>  escreveu:
> >
> > Mas aí então a+bi e b+ai são os mesmos números
>
> Não são.
>
> 4+5i e 5+4i são diferentes, e 4+5i < 5+4i por essas regras.
>
> >
> > Em seg, 26 de abr de 2021 13:36, Anderson Torres <
> torres.anderson...@gmail.com> escreveu:
> >>
> >> Em qui., 22 de abr. de 2021 às 07:19, Israel Meireles Chrisostomo
> >>  escreveu:
> >> >
> >> > Me desculpem se eu estou falando bobagem, mas considere uma função
> com domínio complexo, então essa função não pode ser bijetora, pois toda
> função bijetora ou é crescente ou é decrescente, mas não há ordem nos
> complexos
>
> Você não entendeu nada aqui, suponho. Primeiramente, funções não são
> coisas limitadas a números.
>
> Segundamente, quando usamos esse teorema de que funções contínuas são
> monótonas, é óbvio que estamos supondo de antemão que estamos
> trabalhando com um sistema numérico que admita a ideia de ordem.
> Especialmente, a de um corpo ordenado completo.
>
> Por exemplo, não faz sentido falar de "continuidade" quando se fala de
> funções de naturais para naturais, porque números naturais não formam
> um sistema numérico contínuo.
>
> >>
> >> Não é correto dizer que não existe ordem nos complexos. É só atribuir
> >> o seguinte: o complexo A é maior que o complexo B se e somente se ou o
> >> módulo de A é maior que o de B ou os módulos são iguais mas o
> >> argumento de A é maior que o de B (tomando este módulo no intervalo de
> >> 0 a tau).
> >>
> >> >
> >> > --
> >> > Esta mensagem foi verificada pelo sistema de antivírus e
> >> > acredita-se estar livre de perigo.
> >>
> >> --
> >> Esta mensagem foi verificada pelo sistema de antivírus e
> >>  acredita-se estar livre de perigo.
> >>
> >>
> >>
> =
> >> Instru�ões para entrar na lista, sair da lista e usar a lista em
> >> http://www.mat.puc-rio.br/~obmlistas/obm-l.html
> >>
> =
> >
> >
> > --
> > Esta mensagem foi verificada pelo sistema de antivírus e
> > acredita-se estar livre de perigo.
>
> --
> Esta mensagem foi verificada pelo sistema de antivírus e
>  acredita-se estar livre de perigo.
>
>
> =
> Instru�ões para entrar na lista, sair da lista e usar a lista em
> http://www.mat.puc-rio.br/~obmlistas/obm-l.html
> =
>


-- 
Israel Meireles Chrisostomo

-- 
Esta mensagem foi verificada pelo sistema de antiv�rus e
 acredita-se estar livre de perigo.



[obm-l] Re: [obm-l] Re: [obm-l] Re: [obm-l] Função

2021-05-19 Por tôpico Anderson Torres
Em seg., 26 de abr. de 2021 às 17:18, Israel Meireles Chrisostomo
 escreveu:
>
> Mas aí então a+bi e b+ai são os mesmos números

Não são.

4+5i e 5+4i são diferentes, e 4+5i < 5+4i por essas regras.

>
> Em seg, 26 de abr de 2021 13:36, Anderson Torres 
>  escreveu:
>>
>> Em qui., 22 de abr. de 2021 às 07:19, Israel Meireles Chrisostomo
>>  escreveu:
>> >
>> > Me desculpem se eu estou falando bobagem, mas considere uma função com 
>> > domínio complexo, então essa função não pode ser bijetora, pois toda 
>> > função bijetora ou é crescente ou é decrescente, mas não há ordem nos 
>> > complexos

Você não entendeu nada aqui, suponho. Primeiramente, funções não são
coisas limitadas a números.

Segundamente, quando usamos esse teorema de que funções contínuas são
monótonas, é óbvio que estamos supondo de antemão que estamos
trabalhando com um sistema numérico que admita a ideia de ordem.
Especialmente, a de um corpo ordenado completo.

Por exemplo, não faz sentido falar de "continuidade" quando se fala de
funções de naturais para naturais, porque números naturais não formam
um sistema numérico contínuo.

>>
>> Não é correto dizer que não existe ordem nos complexos. É só atribuir
>> o seguinte: o complexo A é maior que o complexo B se e somente se ou o
>> módulo de A é maior que o de B ou os módulos são iguais mas o
>> argumento de A é maior que o de B (tomando este módulo no intervalo de
>> 0 a tau).
>>
>> >
>> > --
>> > Esta mensagem foi verificada pelo sistema de antivírus e
>> > acredita-se estar livre de perigo.
>>
>> --
>> Esta mensagem foi verificada pelo sistema de antivírus e
>>  acredita-se estar livre de perigo.
>>
>>
>> =
>> Instru�ões para entrar na lista, sair da lista e usar a lista em
>> http://www.mat.puc-rio.br/~obmlistas/obm-l.html
>> =
>
>
> --
> Esta mensagem foi verificada pelo sistema de antivírus e
> acredita-se estar livre de perigo.

-- 
Esta mensagem foi verificada pelo sistema de antiv�rus e
 acredita-se estar livre de perigo.


=
Instru��es para entrar na lista, sair da lista e usar a lista em
http://www.mat.puc-rio.br/~obmlistas/obm-l.html
=


[obm-l] Re: [obm-l] Re: [obm-l] Função

2021-04-26 Por tôpico Israel Meireles Chrisostomo
Mas aí então a+bi e b+ai são os mesmos números

Em seg, 26 de abr de 2021 13:36, Anderson Torres <
torres.anderson...@gmail.com> escreveu:

> Em qui., 22 de abr. de 2021 às 07:19, Israel Meireles Chrisostomo
>  escreveu:
> >
> > Me desculpem se eu estou falando bobagem, mas considere uma função com
> domínio complexo, então essa função não pode ser bijetora, pois toda função
> bijetora ou é crescente ou é decrescente, mas não há ordem nos complexos
>
> Não é correto dizer que não existe ordem nos complexos. É só atribuir
> o seguinte: o complexo A é maior que o complexo B se e somente se ou o
> módulo de A é maior que o de B ou os módulos são iguais mas o
> argumento de A é maior que o de B (tomando este módulo no intervalo de
> 0 a tau).
>
> >
> > --
> > Esta mensagem foi verificada pelo sistema de antivírus e
> > acredita-se estar livre de perigo.
>
> --
> Esta mensagem foi verificada pelo sistema de antivírus e
>  acredita-se estar livre de perigo.
>
>
> =
> Instru�ões para entrar na lista, sair da lista e usar a lista em
> http://www.mat.puc-rio.br/~obmlistas/obm-l.html
> =
>

-- 
Esta mensagem foi verificada pelo sistema de antiv�rus e
 acredita-se estar livre de perigo.



[obm-l] Re: [obm-l] Re: [obm-l] Função

2021-04-23 Por tôpico Israel Meireles Chrisostomo
Obrigado

Em qui, 22 de abr de 2021 11:25, Artur Costa Steiner <
artur.costa.stei...@gmail.com> escreveu:

> O que vc disse só vale para funções contínuas de R em R. No domínio
> complexo, não vale.
> Nos complexos, uma função inteira é injetora se, e somente se, for um
> mapeamento afim não constante, caso em que é bijetora.
>
> Artur
>
>
> Em qui., 22 de abr. de 2021 07:19, Israel Meireles Chrisostomo <
> israelmchrisost...@gmail.com> escreveu:
>
>> Me desculpem se eu estou falando bobagem, mas considere uma função com
>> domínio complexo, então essa função não pode ser bijetora, pois toda função
>> bijetora ou é crescente ou é decrescente, mas não há ordem nos complexos
>>
>> --
>> Esta mensagem foi verificada pelo sistema de antivírus e
>> acredita-se estar livre de perigo.
>
>
> --
> Esta mensagem foi verificada pelo sistema de antivírus e
> acredita-se estar livre de perigo.

-- 
Esta mensagem foi verificada pelo sistema de antiv�rus e
 acredita-se estar livre de perigo.



[obm-l] Re: [obm-l] Re: [obm-l] Questão sobre desigualdades

2021-04-14 Por tôpico Carlos Monteiro
De onde saiu essa desigualdade?

Em qua., 14 de abr. de 2021 às 20:39, Anderson Torres <
torres.anderson...@gmail.com> escreveu:

> Em qua., 14 de abr. de 2021 às 15:54, Carlos Monteiro
>  escreveu:
> >
> > Encontre os valores máximo e mínimo da expressão:  x/(x^2+1) + y/(y^2+1)
> + z/(z^2+1) , onde x, y e z são números reais que satisfazem x+y+z = 1.
> >
> >
>
> Verifica-se que 3(12x+1)/50 >= x/(x^2+1), e assim o valor máximo é 3/10
>
> >
> >
> > --
> > Esta mensagem foi verificada pelo sistema de antivírus e
> > acredita-se estar livre de perigo.
>
> --
> Esta mensagem foi verificada pelo sistema de antivírus e
>  acredita-se estar livre de perigo.
>
>
> =
> Instru�ões para entrar na lista, sair da lista e usar a lista em
> http://www.mat.puc-rio.br/~obmlistas/obm-l.html
> =
>

-- 
Esta mensagem foi verificada pelo sistema de antiv�rus e
 acredita-se estar livre de perigo.



Re: [obm-l] Re: transcendencia

2021-04-02 Por tôpico Carlos Gustavo Tamm de Araujo Moreira
Por outro lado, se v é algébrico e u é algébrico sobre o corpo Q(v) então u
é algébrico.
O meu exemplo é um pouco "roubado": parece que b satisfaz a equação
(a^2-2)b+a(a^2-2)=0, mas, como
a^2-2=0, essa equação é identicamente nula...
Abraços,
 Gugu

On Fri, Apr 2, 2021 at 4:57 PM Israel Meireles Chrisostomo <
israelmchrisost...@gmail.com> wrote:

> Muito obrigado professor gugu
>
> Em sex, 2 de abr de 2021 16:00, Carlos Gustavo Tamm de Araujo Moreira <
> g...@impa.br> escreveu:
>
>> Não. Se a=sqrt(2) e b=pi então a^3+b.a^2-2a-2b=0, por exemplo.
>>
>> Em sex, 2 de abr de 2021 15:31, Israel Meireles Chrisostomo <
>> israelmchrisost...@gmail.com> escreveu:
>>
>>>   Se u é um número transcendente e v é um número, se u,v são
>>> algebricamente dependentes então v é transcendente?
>>>
>>>
>>> Em sex., 2 de abr. de 2021 às 14:58, Israel Meireles Chrisostomo <
>>> israelmchrisost...@gmail.com> escreveu:
>>>
 Se a é um número transcendente e v é um número, se u,v são
 algebricamente dependentes então v é transcendente?

 --
 Israel Meireles Chrisostomo

>>>
>>>
>>> --
>>> Israel Meireles Chrisostomo
>>>
>>> --
>>> Esta mensagem foi verificada pelo sistema de antivírus e
>>> acredita-se estar livre de perigo.
>>
>>
>> --
>> Esta mensagem foi verificada pelo sistema de antivírus e
>> acredita-se estar livre de perigo.
>
>
> --
> Esta mensagem foi verificada pelo sistema de antivírus e
> acredita-se estar livre de perigo.

-- 
Esta mensagem foi verificada pelo sistema de antiv�rus e
 acredita-se estar livre de perigo.



Re: [obm-l] Re: transcendencia

2021-04-02 Por tôpico Israel Meireles Chrisostomo
Muito obrigado professor gugu

Em sex, 2 de abr de 2021 16:00, Carlos Gustavo Tamm de Araujo Moreira <
g...@impa.br> escreveu:

> Não. Se a=sqrt(2) e b=pi então a^3+b.a^2-2a-2b=0, por exemplo.
>
> Em sex, 2 de abr de 2021 15:31, Israel Meireles Chrisostomo <
> israelmchrisost...@gmail.com> escreveu:
>
>>   Se u é um número transcendente e v é um número, se u,v são
>> algebricamente dependentes então v é transcendente?
>>
>>
>> Em sex., 2 de abr. de 2021 às 14:58, Israel Meireles Chrisostomo <
>> israelmchrisost...@gmail.com> escreveu:
>>
>>> Se a é um número transcendente e v é um número, se u,v são
>>> algebricamente dependentes então v é transcendente?
>>>
>>> --
>>> Israel Meireles Chrisostomo
>>>
>>
>>
>> --
>> Israel Meireles Chrisostomo
>>
>> --
>> Esta mensagem foi verificada pelo sistema de antivírus e
>> acredita-se estar livre de perigo.
>
>
> --
> Esta mensagem foi verificada pelo sistema de antivírus e
> acredita-se estar livre de perigo.

-- 
Esta mensagem foi verificada pelo sistema de antiv�rus e
 acredita-se estar livre de perigo.



Re: [obm-l] Re: transcendencia

2021-04-02 Por tôpico Carlos Gustavo Tamm de Araujo Moreira
Não. Se a=sqrt(2) e b=pi então a^3+b.a^2-2a-2b=0, por exemplo.

Em sex, 2 de abr de 2021 15:31, Israel Meireles Chrisostomo <
israelmchrisost...@gmail.com> escreveu:

>   Se u é um número transcendente e v é um número, se u,v são
> algebricamente dependentes então v é transcendente?
>
>
> Em sex., 2 de abr. de 2021 às 14:58, Israel Meireles Chrisostomo <
> israelmchrisost...@gmail.com> escreveu:
>
>> Se a é um número transcendente e v é um número, se u,v são algebricamente
>> dependentes então v é transcendente?
>>
>> --
>> Israel Meireles Chrisostomo
>>
>
>
> --
> Israel Meireles Chrisostomo
>
> --
> Esta mensagem foi verificada pelo sistema de antivírus e
> acredita-se estar livre de perigo.

-- 
Esta mensagem foi verificada pelo sistema de antiv�rus e
 acredita-se estar livre de perigo.



[obm-l] Re: [obm-l] Re: [obm-l] Re: [obm-l] Re: [obm-l] Re: [obm-l] Re: [obm-l] Re: [obm-l] Re: Equações funcionais

2021-02-16 Por tôpico Anderson Torres
Em ter., 16 de fev. de 2021 às 21:26, joao pedro b menezes
 escreveu:
>
> Eu sei, temos f(-1)= 0, f(0) = 1, e f é bijetora. Após trabalhar a equação 
> que cheguei na expressão:
> f( x + f(x) ) - f( f(x)) = x.  Queria saber se essa identidade, junto com a 
> do enunciado, é suficiente para provar a linearidade de f.
>

Seriosamente, não me parece útil perder tempo provando que isso é
linear. O processo que você levaria provando que f(x)=Ax+B basicamente
se resumiria a finalizar o problema.

Outra identidade que pode ser útil para você é provar que f(f(x)) -
f(x) = f(x) -x.
Essa, junto com a identidade acima que você provou, te deixam em 70%
do problema.

> --
> Esta mensagem foi verificada pelo sistema de antivírus e
> acredita-se estar livre de perigo.

-- 
Esta mensagem foi verificada pelo sistema de antiv�rus e
 acredita-se estar livre de perigo.


=
Instru��es para entrar na lista, sair da lista e usar a lista em
http://www.mat.puc-rio.br/~obmlistas/obm-l.html
=


[obm-l] Re: [obm-l] Re: [obm-l] Re: [obm-l] Re: [obm-l] Re: [obm-l] Re: [obm-l] Re: Equações funcionais

2021-02-16 Por tôpico joao pedro b menezes
Eu sei, temos f(-1)= 0, f(0) = 1, e f é bijetora. Após trabalhar a equação
que cheguei na expressão:
f( x + f(x) ) - f( f(x)) = x.  Queria saber se essa identidade, junto com a
do enunciado, é suficiente para provar a linearidade de f.

-- 
Esta mensagem foi verificada pelo sistema de antiv�rus e
 acredita-se estar livre de perigo.



[obm-l] Re: [obm-l] Re: [obm-l] Re: [obm-l] Re: [obm-l] Re: [obm-l] Re: Equações funcionais

2021-02-16 Por tôpico Anderson Torres
Em ter., 16 de fev. de 2021 às 20:43, joao pedro b menezes
 escreveu:
>
> Foi da OBM 2006, nível 3,  3° fase:
> “Determine todas as funções f: R -> R tais que
> f( xf(y) + f(x) ) = 2f(x) + xy

Isso dá bem mais informação!

Por exemplo essa função é sobrejetora. Afinal, qualquer número pode
ser escrito na forma 2f(x)+xy - faça por exemplo x=1 e y=z-2f(1).

Daí a ideia é resolver as equacoes f(A)=0 e f(B)=1.

> para todos x,y reais”
>
> --
> Esta mensagem foi verificada pelo sistema de antivírus e
> acredita-se estar livre de perigo.

-- 
Esta mensagem foi verificada pelo sistema de antiv�rus e
 acredita-se estar livre de perigo.


=
Instru��es para entrar na lista, sair da lista e usar a lista em
http://www.mat.puc-rio.br/~obmlistas/obm-l.html
=


[obm-l] Re: [obm-l] Re: [obm-l] Re: [obm-l] Re: [obm-l] Sequência Injetiva

2021-02-16 Por tôpico Anderson Torres
Nada como uma bijeção N -> Q para encerrar o dia!

Se pensar nas operacoes INC e REV, podemos usar um algoritmo assim:

- Se o número é maior que 1, usa DEC (inversa de INC)
- Se o número é menor que 1, usa INV
- Se o número é 1, pare

Como demonstrar que este procedimento sempre encerrará em 1, não
importando que número racional começou? Acho que no fundo isso é só
uma maneira de encodar fracoes continuas mesmo.

Em ter., 16 de fev. de 2021 às 20:35, Matheus Secco
 escreveu:
>
> Esse problema caiu na Olimpíada Iberoamericana de 2009 que eu participei. Foi 
> o problema 5 da prova e lá pedia para provar injetividade e sobrejetividade.
>
> Em qua, 17 de fev de 2021 00:16, Anderson Torres 
>  escreveu:
>>
>> Em dom., 14 de fev. de 2021 às 17:20, Claudio Buffara
>>  escreveu:
>> >
>> > Será que essa sequência é sobrejetiva (sobre os racionais positivos)?
>> > Porque como a(2^n) = n+1, ela certamente atinge todos os naturais, de modo 
>> > que é ilimitada, superiormente e inferiormente (já que a(2^n + 1) = 
>> > 1/(n+1) ).
>> > Mesmo que não seja, seria interessante descobrir que racionais positivos 
>> > ela não atinge.
>> > É suficiente provar que todos os racionais entre 0 e 1 são atingidos (no 
>> > caso, pelos termos de ordem ímpar), mas não sei se isso facilita.
>> > Vale uma exploração numérica, talvez com uma planilha.
>>
>>
>> Se eu não errei as contas, acredito que sim. Afinal basta reverter a
>> fracao continua.
>>
>> As operacoes parecem ser bem limitadas, contudo nao e necessario muito
>> mais que isso para gerar um racional qualquer:
>>
>> - Função INC: x -> x+1
>> - Função REV: x -> 1/x
>>
>> Talvez haja algum invariante que permita prever que cada operacao esta
>> fadada a cair em 1
>>
>> >
>> >
>> > Abs,
>> > Claudio.
>> >
>> > Enviado do meu iPhone
>> >
>> > Em 14 de fev. de 2021, à(s) 13:57, Anderson Torres 
>> >  escreveu:
>> >
>> > 
>> >
>> >
>> > Em sáb., 13 de fev. de 2021 às 17:56, Jeferson Almir 
>> >  escreveu:
>> >>
>> >> Amigos, peço ajuda em provar a injetividade dessa sequência que seria 
>> >> uma saída para provar a unica ocorrência do racional que aparece nela. 
>> >> Estou andando em círculos tentando montar uma possível indução.
>> >>
>> >>
>> >> Dado a sequência a_1 = 1 e a_2n = a_n  + 1 e a_2n+1 = 1/a_2n.
>> >>
>> >> Prove que para todo racional positivo que ocorre na sequência, ocorre 
>> >> uma única vez.
>> >>
>> >>
>> >
>> > Acho que e uma boa usar fracao continua aqui.
>> >
>> > Se a_n = [c0; c1, c2, ..., ck], temos entao a_1 = [1] e
>> >
>> > a_2n =Â [(1+c0); c1, c2, ..., ck] (chamemos isso de operacao E)
>> > a_2n+1 = [0; (1+c0), c1, c2, ..., ck] (chamemos isso de operacao O)
>> >
>> >
>> > A partir disso, acredito que a bijecao fica quase obvia, bastando 
>> > formalizar algumas inducoes marotas.
>> >
>> > Primeiramente, nenhuma representacao da forma [...,N,1] vai surgir dai a 
>> > partir de a_2. Isso pode ser demonstrado por inducao mesmo: ck=1 somente 
>> > no caso [1], e depois dele a funcao a_n so modifica o comeco da cadeia, 
>> > nunca o final dela.
>> >
>> > Assim sendo, temos certeza que nao tem como um racional aparecer uma vez 
>> > na forma canonica e outra na forma alternativa. E, por conseguinte, se 
>> > duas fracoes tem comprimentos diferentes, elas devem ser diferentes. E 
>> > fracoes com comprimentos iguais diferem se e somente se pelo menos um dos 
>> > componentes diferir.
>> >
>> > Agora, a funcao recursiva age de duas formas. Uma delas altera o 
>> > comprimento em 1, e a outra mantém. A que altera, só altera 
>> > acrescentando o 0 na cabeceira. A que não altera, incrementa a cabeceira.
>> >
>> > Desta forma, é possível gerar de maneira unica qualquer numero racional 
>> > comecando do 1.
>> >
>> > - Qualquer fracao de comprimento 1 pode ser gerada simplesmente aplicando 
>> > a operacao E tantas vezes quantas forem necessarias. E tambem nao e 
>> > possivel fazer isso de outra maneira, pois a operacao O aumentara o 
>> > comprimento de maneira irreversivel.
>> >
>> > - Dada uma fracao com comprimento K, temos duas sub inducoes para fazer:
>> >
>> > + A fracao tem comprimento K e comeca com 0.
>> >
>> > Â  Entao ela foi gerada por uma operacao O. O elemento que a gerou tinha 
>> > menos componentes, os quais satisfazem a hipotese de inducao.
>> >
>> > + A fracao tem comprimento K e comeca com algo maior que 0.
>> >
>> > Entao ela foi gerada por uma operacao E. A fracao da qual ela foi gerada 
>> > difere unicamente no primeiro elemento, o qual antes era menor. Assim 
>> > sendo, e possivel reduzir isso ate chegar no caso anterior.
>> >
>> > E isso demonstra recursivamente a unicidade e existencia!
>> >
>> >
>> >
>> >> --
>> >> Esta mensagem foi verificada pelo sistema de antivírus e
>> >> acredita-se estar livre de perigo.
>> >
>> >
>> > --
>> > Esta mensagem foi verificada pelo sistema de antivírus e
>> > acredita-se estar livre de perigo.
>> >
>> >
>> > --
>> > Esta mensagem foi verificada pelo sistema de 

[obm-l] Re: [obm-l] Re: [obm-l] Re: [obm-l] Re: [obm-l] Re: Equações funcionais

2021-02-16 Por tôpico joao pedro b menezes
Foi da OBM 2006, nível 3,  3° fase:
“Determine todas as funções f: R -> R tais que
f( xf(y) + f(x) ) = 2f(x) + xy
para todos x,y reais”

-- 
Esta mensagem foi verificada pelo sistema de antiv�rus e
 acredita-se estar livre de perigo.



[obm-l] Re: [obm-l] Re: [obm-l] Re: [obm-l] Sequência Injetiva

2021-02-16 Por tôpico Matheus Secco
Esse problema caiu na Olimpíada Iberoamericana de 2009 que eu participei.
Foi o problema 5 da prova e lá pedia para provar injetividade e
sobrejetividade.

Em qua, 17 de fev de 2021 00:16, Anderson Torres <
torres.anderson...@gmail.com> escreveu:

> Em dom., 14 de fev. de 2021 às 17:20, Claudio Buffara
>  escreveu:
> >
> > Será que essa sequência é sobrejetiva (sobre os racionais positivos)?
> > Porque como a(2^n) = n+1, ela certamente atinge todos os naturais, de
> modo que é ilimitada, superiormente e inferiormente (já que a(2^n + 1) =
> 1/(n+1) ).
> > Mesmo que não seja, seria interessante descobrir que racionais positivos
> ela não atinge.
> > É suficiente provar que todos os racionais entre 0 e 1 são atingidos (no
> caso, pelos termos de ordem ímpar), mas não sei se isso facilita.
> > Vale uma exploração numérica, talvez com uma planilha.
>
>
> Se eu não errei as contas, acredito que sim. Afinal basta reverter a
> fracao continua.
>
> As operacoes parecem ser bem limitadas, contudo nao e necessario muito
> mais que isso para gerar um racional qualquer:
>
> - Função INC: x -> x+1
> - Função REV: x -> 1/x
>
> Talvez haja algum invariante que permita prever que cada operacao esta
> fadada a cair em 1
>
> >
> >
> > Abs,
> > Claudio.
> >
> > Enviado do meu iPhone
> >
> > Em 14 de fev. de 2021, à(s) 13:57, Anderson Torres <
> torres.anderson...@gmail.com> escreveu:
> >
> > 
> >
> >
> > Em sáb., 13 de fev. de 2021 às 17:56, Jeferson Almir <
> jefersonram...@gmail.com> escreveu:
> >>
> >> Amigos, peço ajuda em provar a injetividade dessa sequência que seria
> uma saída para provar a unica ocorrência do racional que aparece nela.
> Estou andando em círculos tentando montar uma possível indução.
> >>
> >>
> >> Dado a sequência a_1 = 1 e a_2n = a_n  + 1 e a_2n+1 = 1/a_2n.
> >>
> >> Prove que para todo racional positivo que ocorre na sequência, ocorre
> uma única vez.
> >>
> >>
> >
> > Acho que e uma boa usar fracao continua aqui.
> >
> > Se a_n = [c0; c1, c2, ..., ck], temos entao a_1 = [1] e
> >
> > a_2n =Â [(1+c0); c1, c2, ..., ck] (chamemos isso de operacao E)
> > a_2n+1 = [0; (1+c0), c1, c2, ..., ck] (chamemos isso de operacao O)
> >
> >
> > A partir disso, acredito que a bijecao fica quase obvia, bastando
> formalizar algumas inducoes marotas.
> >
> > Primeiramente, nenhuma representacao da forma [...,N,1] vai surgir dai a
> partir de a_2. Isso pode ser demonstrado por inducao mesmo: ck=1 somente
> no caso [1], e depois dele a funcao a_n so modifica o comeco da cadeia,
> nunca o final dela.
> >
> > Assim sendo, temos certeza que nao tem como um racional aparecer uma vez
> na forma canonica e outra na forma alternativa. E, por conseguinte, se duas
> fracoes tem comprimentos diferentes, elas devem ser diferentes. E fracoes
> com comprimentos iguais diferem se e somente se pelo menos um dos
> componentes diferir.
> >
> > Agora, a funcao recursiva age de duas formas. Uma delas altera o
> comprimento em 1, e a outra mantém. A que altera, só altera acrescentando
> o 0 na cabeceira. A que não altera, incrementa a cabeceira.
> >
> > Desta forma, é possível gerar de maneira unica qualquer numeroÂ
> racional comecando do 1.
> >
> > - Qualquer fracao de comprimento 1 pode ser gerada simplesmente
> aplicando a operacao E tantas vezes quantas forem necessarias. E tambem
> nao e possivel fazer isso de outra maneira, pois a operacao O aumentara o
> comprimento de maneira irreversivel.
> >
> > - Dada uma fracao com comprimento K, temos duas sub inducoes para fazer:
> >
> > + A fracao tem comprimento K e comeca com 0.
> >
> > Â  Entao ela foi gerada por uma operacao O. O elemento que a gerou tinha
> menos componentes, os quais satisfazem a hipotese de inducao.
> >
> > + A fracao tem comprimento K e comeca com algo maior que 0.
> >
> > Entao ela foi gerada por uma operacao E. A fracao da qual ela foi gerada
> difere unicamente no primeiro elemento, o qual antes era menor. Assim
> sendo, e possivel reduzir isso ate chegar no caso anterior.
> >
> > E isso demonstra recursivamente a unicidade e existencia!
> >
> >
> >
> >> --
> >> Esta mensagem foi verificada pelo sistema de antivírus e
> >> acredita-se estar livre de perigo.
> >
> >
> > --
> > Esta mensagem foi verificada pelo sistema de antivírus e
> > acredita-se estar livre de perigo.
> >
> >
> > --
> > Esta mensagem foi verificada pelo sistema de antivírus e
> > acredita-se estar livre de perigo.
>
> --
> Esta mensagem foi verificada pelo sistema de antivírus e
>  acredita-se estar livre de perigo.
>
>
> =
> Instru�ões para entrar na lista, sair da lista e usar a lista em
> http://www.mat.puc-rio.br/~obmlistas/obm-l.html
> =
>

-- 
Esta mensagem foi verificada pelo sistema de antiv�rus e
 acredita-se estar livre de perigo.



[obm-l] Re: [obm-l] Re: [obm-l] Re: [obm-l] Re: Equações funcionais

2021-02-16 Por tôpico Anderson Torres
Eu gostaria de saber da origem desse problema...

Em dom., 14 de fev. de 2021 às 14:32, joao pedro b menezes <
joaopedrobmene...@gmail.com> escreveu:

> Obrigado pela resposta, mas ainda tenho umas dúvidas. Poderia dar um
> exemplo de tal função ou explicar como construí-la? E se f fosse somente
> injetora, mudaria alguma coisa?
>
>
> --
> Esta mensagem foi verificada pelo sistema de antivírus e
> acredita-se estar livre de perigo.

-- 
Esta mensagem foi verificada pelo sistema de antiv�rus e
 acredita-se estar livre de perigo.



[obm-l] Re: [obm-l] Re: [obm-l] Sequência Injetiva

2021-02-16 Por tôpico Anderson Torres
Em dom., 14 de fev. de 2021 às 17:20, Claudio Buffara
 escreveu:
>
> Será que essa sequência é sobrejetiva (sobre os racionais positivos)?
> Porque como a(2^n) = n+1, ela certamente atinge todos os naturais, de modo 
> que é ilimitada, superiormente e inferiormente (já que a(2^n + 1) = 1/(n+1) ).
> Mesmo que não seja, seria interessante descobrir que racionais positivos ela 
> não atinge.
> É suficiente provar que todos os racionais entre 0 e 1 são atingidos (no 
> caso, pelos termos de ordem ímpar), mas não sei se isso facilita.
> Vale uma exploração numérica, talvez com uma planilha.


Se eu não errei as contas, acredito que sim. Afinal basta reverter a
fracao continua.

As operacoes parecem ser bem limitadas, contudo nao e necessario muito
mais que isso para gerar um racional qualquer:

- Função INC: x -> x+1
- Função REV: x -> 1/x

Talvez haja algum invariante que permita prever que cada operacao esta
fadada a cair em 1

>
>
> Abs,
> Claudio.
>
> Enviado do meu iPhone
>
> Em 14 de fev. de 2021, à(s) 13:57, Anderson Torres 
>  escreveu:
>
> 
>
>
> Em sáb., 13 de fev. de 2021 às 17:56, Jeferson Almir 
>  escreveu:
>>
>> Amigos, peço ajuda em provar a injetividade dessa sequência que seria uma 
>> saída para provar a unica ocorrência do racional que aparece nela. Estou 
>> andando em círculos tentando montar uma possível indução.
>>
>>
>> Dado a sequência a_1 = 1 e a_2n = a_n  + 1 e a_2n+1 = 1/a_2n.
>>
>> Prove que para todo racional positivo que ocorre na sequência, ocorre uma 
>> única vez.
>>
>>
>
> Acho que e uma boa usar fracao continua aqui.
>
> Se a_n = [c0; c1, c2, ..., ck], temos entao a_1 = [1] e
>
> a_2n =Â [(1+c0); c1, c2, ..., ck] (chamemos isso de operacao E)
> a_2n+1 = [0; (1+c0), c1, c2, ..., ck] (chamemos isso de operacao O)
>
>
> A partir disso, acredito que a bijecao fica quase obvia, bastando formalizar 
> algumas inducoes marotas.
>
> Primeiramente, nenhuma representacao da forma [...,N,1] vai surgir dai a 
> partir de a_2. Isso pode ser demonstrado por inducao mesmo: ck=1 somente no 
> caso [1], e depois dele a funcao a_n so modifica o comeco da cadeia, nunca o 
> final dela.
>
> Assim sendo, temos certeza que nao tem como um racional aparecer uma vez na 
> forma canonica e outra na forma alternativa. E, por conseguinte, se duas 
> fracoes tem comprimentos diferentes, elas devem ser diferentes. E fracoes com 
> comprimentos iguais diferem se e somente se pelo menos um dos componentes 
> diferir.
>
> Agora, a funcao recursiva age de duas formas. Uma delas altera o comprimento 
> em 1, e a outra mantém. A que altera, só altera acrescentando o 0 na 
> cabeceira. A que não altera, incrementa a cabeceira.
>
> Desta forma, é possível gerar de maneira unica qualquer numero racional 
> comecando do 1.
>
> - Qualquer fracao de comprimento 1 pode ser gerada simplesmente aplicando a 
> operacao E tantas vezes quantas forem necessarias. E tambem nao e possivel 
> fazer isso de outra maneira, pois a operacao O aumentara o comprimento de 
> maneira irreversivel.
>
> - Dada uma fracao com comprimento K, temos duas sub inducoes para fazer:
>
> + A fracao tem comprimento K e comeca com 0.
>
> Â  Entao ela foi gerada por uma operacao O. O elemento que a gerou tinha 
> menos componentes, os quais satisfazem a hipotese de inducao.
>
> + A fracao tem comprimento K e comeca com algo maior que 0.
>
> Entao ela foi gerada por uma operacao E. A fracao da qual ela foi gerada 
> difere unicamente no primeiro elemento, o qual antes era menor. Assim sendo, 
> e possivel reduzir isso ate chegar no caso anterior.
>
> E isso demonstra recursivamente a unicidade e existencia!
>
>
>
>> --
>> Esta mensagem foi verificada pelo sistema de antivírus e
>> acredita-se estar livre de perigo.
>
>
> --
> Esta mensagem foi verificada pelo sistema de antivírus e
> acredita-se estar livre de perigo.
>
>
> --
> Esta mensagem foi verificada pelo sistema de antivírus e
> acredita-se estar livre de perigo.

-- 
Esta mensagem foi verificada pelo sistema de antiv�rus e
 acredita-se estar livre de perigo.


=
Instru��es para entrar na lista, sair da lista e usar a lista em
http://www.mat.puc-rio.br/~obmlistas/obm-l.html
=


Re: [obm-l] Re: [obm-l] Sequência Injetiva

2021-02-14 Por tôpico Claudio Buffara
Será que essa sequência é sobrejetiva (sobre os racionais positivos)?
Porque como a(2^n) = n+1, ela certamente atinge todos os naturais, de modo que 
é ilimitada, superiormente e inferiormente (já que a(2^n + 1) = 1/(n+1) ).
Mesmo que não seja, seria interessante descobrir que racionais positivos ela 
não atinge.
É suficiente provar que todos os racionais entre 0 e 1 são atingidos (no caso, 
pelos termos de ordem ímpar), mas não sei se isso facilita.
Vale uma exploração numérica, talvez com uma planilha.

Abs,
Claudio.

Enviado do meu iPhone

> Em 14 de fev. de 2021, à(s) 13:57, Anderson Torres 
>  escreveu:
> 
> 
> 
> 
> Em sáb., 13 de fev. de 2021 às 17:56, Jeferson Almir 
>  escreveu:
>> Amigos, peço ajuda em provar a injetividade dessa sequência que seria uma 
>> saída para provar a unica ocorrência do racional que aparece nela. Estou 
>> andando em círculos tentando montar uma possível indução.
>> 
>> Dado a sequência a_1 = 1 e a_2n = a_n  + 1 e a_2n+1 = 1/a_2n.
>> Prove que para todo racional positivo que ocorre na sequência, ocorre uma 
>> única vez.
>> 
> 
> Acho que e uma boa usar fracao continua aqui.
> 
> Se a_n = [c0; c1, c2, ..., ck], temos entao a_1 = [1] e
> 
> a_2n =Â [(1+c0); c1, c2, ..., ck] (chamemos isso de operacao E)
> a_2n+1 = [0; (1+c0), c1, c2, ..., ck] (chamemos isso de operacao O)
> 
> 
> A partir disso, acredito que a bijecao fica quase obvia, bastando formalizar 
> algumas inducoes marotas.
> 
> Primeiramente, nenhuma representacao da forma [...,N,1] vai surgir dai a 
> partir de a_2. Isso pode ser demonstrado por inducao mesmo: ck=1 somente no 
> caso [1], e depois dele a funcao a_n so modifica o comeco da cadeia, nunca o 
> final dela.
> 
> Assim sendo, temos certeza que nao tem como um racional aparecer uma vez na 
> forma canonica e outra na forma alternativa. E, por conseguinte, se duas 
> fracoes tem comprimentos diferentes, elas devem ser diferentes. E fracoes com 
> comprimentos iguais diferem se e somente se pelo menos um dos componentes 
> diferir.
> 
> Agora, a funcao recursiva age de duas formas. Uma delas altera o comprimento 
> em 1, e a outra mantém. A que altera, só altera acrescentando o 0 na 
> cabeceira. A que não altera, incrementa a cabeceira.
> 
> Desta forma, é possível gerar de maneira unica qualquer numero racional 
> comecando do 1.
> 
> - Qualquer fracao de comprimento 1 pode ser gerada simplesmente aplicando a 
> operacao E tantas vezes quantas forem necessarias. E tambem nao e possivel 
> fazer isso de outra maneira, pois a operacao O aumentara o comprimento de 
> maneira irreversivel.
> 
> - Dada uma fracao com comprimento K, temos duas sub inducoes para fazer:
> 
> + A fracao tem comprimento K e comeca com 0.
> 
> Â  Entao ela foi gerada por uma operacao O. O elemento que a gerou tinha 
> menos componentes, os quais satisfazem a hipotese de inducao.
> 
> + A fracao tem comprimento K e comeca com algo maior que 0.
> 
> Entao ela foi gerada por uma operacao E. A fracao da qual ela foi gerada 
> difere unicamente no primeiro elemento, o qual antes era menor. Assim sendo, 
> e possivel reduzir isso ate chegar no caso anterior.
> 
> E isso demonstra recursivamente a unicidade e existencia!
> 
> 
> 
>> -- 
>> Esta mensagem foi verificada pelo sistema de antivírus e 
>> acredita-se estar livre de perigo.
> 
> -- 
> Esta mensagem foi verificada pelo sistema de antivírus e 
> acredita-se estar livre de perigo.

-- 
Esta mensagem foi verificada pelo sistema de antiv�rus e
 acredita-se estar livre de perigo.



[obm-l] Re: [obm-l] Re: [obm-l] Re: Equações funcionais

2021-02-14 Por tôpico joao pedro b menezes
Obrigado pela resposta, mas ainda tenho umas dúvidas. Poderia dar um
exemplo de tal função ou explicar como construí-la? E se f fosse somente
injetora, mudaria alguma coisa?

-- 
Esta mensagem foi verificada pelo sistema de antiv�rus e
 acredita-se estar livre de perigo.



[obm-l] Re: [obm-l] Re: Equações funcionais

2021-02-14 Por tôpico Anderson Torres
Em dom., 14 de fev. de 2021 às 11:30, joao pedro b menezes <
joaopedrobmene...@gmail.com> escreveu:

> Obs: f é bijetora
>
>>
>
Acho que nao basta. Se f(x)=y entao f(x+y)=x+f(y).

Com isso, poderiamos fazer uma funcao que nao aja linearmente em (0,1) mas
aja linearmente fora dele.



> --
> Esta mensagem foi verificada pelo sistema de antivírus e
> acredita-se estar livre de perigo.

-- 
Esta mensagem foi verificada pelo sistema de antiv�rus e
 acredita-se estar livre de perigo.



[obm-l] Re: [obm-l] Re: [obm-l] Re: [obm-l] Re: [obm-l] Sequência Injetiva

2021-02-14 Por tôpico Claudio Buffara
a(1) = 1
a(2n) = a(2n-1) + 1
a(2n+1) = 1/a(2n)

Fazendo a(n) = p(n)/q(n), obtemos duas sequências: p(n) e q(n).
E elas são tais que:
p(1) = q(1) = 1
p(2n) = p(2n-1) + q(2n-1)
q(2n) = q(2n-1)
p(2n+1) = q(2n)
q(2n+1) = p(2n)

Como as sequências começam com 1 e 1, que são primos entre si, e como
mdc(p,q) = mdc(q,p) = mdc(p+q,q), p(n) e q(n) sempre serão primos entre si.

Usando a definição de p e q:
p(2n+1) = q(2n) = q(2n-1) = p(2n-2) = p(2n-3) + q(2n-3) = p(2n-3) + q(2n-2)
= p(2n-3) + p(2n-1)
e
p(2n+2) = p(2n+1) + q(2n+1) = q(2n) + p(2n) = q(2n-1) + p(2n) = p(2n-2) +
p(2n)

Ou seja, os termos p(n) de ordem ímpar e de ordem par realmente formam uma
sequência de Fibonacci.
Os de ordem ímpar começam com p(1) = p(3) = 1 e os de ordem par com p(2) =
2 e p(4) = 3.

[]s,
Claudio.

On Sun, Feb 14, 2021 at 10:03 AM Claudio Buffara 
wrote:

> Ué!  Continua sendo. Só que é outra questão...
>
>
> On Sun, Feb 14, 2021 at 3:34 AM Ralph Costa Teixeira 
> wrote:
>
>> Sim, voce tem razao -- eu achei que era a_2n = a_{2n-1} +1. Que pena, era
>> uma boa questao com Fibonacci. :)
>>
>> On Sun, Feb 14, 2021 at 12:35 AM Claudio Buffara <
>> claudio.buff...@gmail.com> wrote:
>>
>>> Oi, Ralph:
>>>
>>> Eu posso ter entendido errado a definição da sequência, mas achei termos
>>> diferentes dos seus:
>>> 1:  1
>>> 2:  2
>>> 3:  1/2
>>> 4:  3
>>> 5:  1/3
>>> 6:  3/2
>>> 7:  2/3
>>> 8:  4
>>> 9:  1/4
>>> 10:  4/3
>>> 11:  3/4
>>> 12:  5/2
>>> 13:  2/5
>>> 14:  5/3
>>> 15:  3/5
>>> 16:  5
>>> ...
>>>
>>> []s,
>>> Claudio.
>>>
>>>
>>> On Sat, Feb 13, 2021 at 7:59 PM Ralph Costa Teixeira 
>>> wrote:
>>>
 Meio enrolado, vou escrever meio vagamente.

 Eu sugiro olhar primeiro para os caras com indice impar. Sao eles:
 a1=1/1
 a3=1/2
 a5=2/3
 a7=3/5
 a8=5/8
 ...
 Ou seja, mostre que eles sao quocientes de numeros de Fibonacci
 consecutivos (os caras de indice par sao os inversos desses). Agora tem
 varias maneiras de continuar:

 -- Voce pode mostrar que os numeros de Fibonacci consecutivos sao
 primos entre si; portanto cada fracao dessas fica unicamente determinada
 por numerador e denominador, e (como os numeros de Fibonacci formam uma
 sequencia crescente) vao ser distintos entre si;
 -- Se voce nao quiser entrar no merito do Fibonacci, tente mostrar
 (pode ser por inducao) que a3 < a7 < a11 <...>>> a_(4k+1) < ... < a13 < a9 < a5 < 1  (phi ali seria (raiz(5)-1) / 2, acho).

 De qualquer forma, como a_(2n+1)<1, a1=1 e os "a_2n" sao os inversos
 dos "a_2n+1, vao ser todos diferentes.

 Abraco, Ralph.


 On Sat, Feb 13, 2021 at 5:56 PM Jeferson Almir <
 jefersonram...@gmail.com> wrote:

> Amigos, peço ajuda em provar a injetividade dessa sequência que seria
> uma saída para provar a unica ocorrência do racional que aparece nela.
> Estou andando em círculos tentando montar uma possível indução.
>
>
> Dado a sequência a_1 = 1 e a_2n = a_n  + 1 e a_2n+1 = 1/a_2n.
>
> Prove que para todo racional positivo que ocorre na sequência, ocorre
> uma única vez.
>
> --
> Esta mensagem foi verificada pelo sistema de antivírus e
> acredita-se estar livre de perigo.


 --
 Esta mensagem foi verificada pelo sistema de antivírus e
 acredita-se estar livre de perigo.
>>>
>>>
>>> --
>>> Esta mensagem foi verificada pelo sistema de antivírus e
>>> acredita-se estar livre de perigo.
>>
>>
>> --
>> Esta mensagem foi verificada pelo sistema de antivírus e
>> acredita-se estar livre de perigo.
>
>

-- 
Esta mensagem foi verificada pelo sistema de antiv�rus e
 acredita-se estar livre de perigo.



[obm-l] Re: [obm-l] Re: [obm-l] Re: [obm-l] Re: [obm-l] Sequência Injetiva

2021-02-14 Por tôpico Claudio Buffara
Ué!  Continua sendo. Só que é outra questão...


On Sun, Feb 14, 2021 at 3:34 AM Ralph Costa Teixeira 
wrote:

> Sim, voce tem razao -- eu achei que era a_2n = a_{2n-1} +1. Que pena, era
> uma boa questao com Fibonacci. :)
>
> On Sun, Feb 14, 2021 at 12:35 AM Claudio Buffara <
> claudio.buff...@gmail.com> wrote:
>
>> Oi, Ralph:
>>
>> Eu posso ter entendido errado a definição da sequência, mas achei termos
>> diferentes dos seus:
>> 1:  1
>> 2:  2
>> 3:  1/2
>> 4:  3
>> 5:  1/3
>> 6:  3/2
>> 7:  2/3
>> 8:  4
>> 9:  1/4
>> 10:  4/3
>> 11:  3/4
>> 12:  5/2
>> 13:  2/5
>> 14:  5/3
>> 15:  3/5
>> 16:  5
>> ...
>>
>> []s,
>> Claudio.
>>
>>
>> On Sat, Feb 13, 2021 at 7:59 PM Ralph Costa Teixeira 
>> wrote:
>>
>>> Meio enrolado, vou escrever meio vagamente.
>>>
>>> Eu sugiro olhar primeiro para os caras com indice impar. Sao eles:
>>> a1=1/1
>>> a3=1/2
>>> a5=2/3
>>> a7=3/5
>>> a8=5/8
>>> ...
>>> Ou seja, mostre que eles sao quocientes de numeros de Fibonacci
>>> consecutivos (os caras de indice par sao os inversos desses). Agora tem
>>> varias maneiras de continuar:
>>>
>>> -- Voce pode mostrar que os numeros de Fibonacci consecutivos sao primos
>>> entre si; portanto cada fracao dessas fica unicamente determinada por
>>> numerador e denominador, e (como os numeros de Fibonacci formam uma
>>> sequencia crescente) vao ser distintos entre si;
>>> -- Se voce nao quiser entrar no merito do Fibonacci, tente mostrar (pode
>>> ser por inducao) que a3 < a7 < a11 <...>> < ... < a13 < a9 < a5 < 1  (phi ali seria (raiz(5)-1) / 2, acho).
>>>
>>> De qualquer forma, como a_(2n+1)<1, a1=1 e os "a_2n" sao os inversos dos
>>> "a_2n+1, vao ser todos diferentes.
>>>
>>> Abraco, Ralph.
>>>
>>>
>>> On Sat, Feb 13, 2021 at 5:56 PM Jeferson Almir 
>>> wrote:
>>>
 Amigos, peço ajuda em provar a injetividade dessa sequência que seria
 uma saída para provar a unica ocorrência do racional que aparece nela.
 Estou andando em círculos tentando montar uma possível indução.


 Dado a sequência a_1 = 1 e a_2n = a_n  + 1 e a_2n+1 = 1/a_2n.

 Prove que para todo racional positivo que ocorre na sequência, ocorre
 uma única vez.

 --
 Esta mensagem foi verificada pelo sistema de antivírus e
 acredita-se estar livre de perigo.
>>>
>>>
>>> --
>>> Esta mensagem foi verificada pelo sistema de antivírus e
>>> acredita-se estar livre de perigo.
>>
>>
>> --
>> Esta mensagem foi verificada pelo sistema de antivírus e
>> acredita-se estar livre de perigo.
>
>
> --
> Esta mensagem foi verificada pelo sistema de antivírus e
> acredita-se estar livre de perigo.

-- 
Esta mensagem foi verificada pelo sistema de antiv�rus e
 acredita-se estar livre de perigo.



[obm-l] Re: [obm-l] Re: [obm-l] Re: [obm-l] Sequência Injetiva

2021-02-13 Por tôpico Ralph Costa Teixeira
Sim, voce tem razao -- eu achei que era a_2n = a_{2n-1} +1. Que pena, era
uma boa questao com Fibonacci. :)

On Sun, Feb 14, 2021 at 12:35 AM Claudio Buffara 
wrote:

> Oi, Ralph:
>
> Eu posso ter entendido errado a definição da sequência, mas achei termos
> diferentes dos seus:
> 1:  1
> 2:  2
> 3:  1/2
> 4:  3
> 5:  1/3
> 6:  3/2
> 7:  2/3
> 8:  4
> 9:  1/4
> 10:  4/3
> 11:  3/4
> 12:  5/2
> 13:  2/5
> 14:  5/3
> 15:  3/5
> 16:  5
> ...
>
> []s,
> Claudio.
>
>
> On Sat, Feb 13, 2021 at 7:59 PM Ralph Costa Teixeira 
> wrote:
>
>> Meio enrolado, vou escrever meio vagamente.
>>
>> Eu sugiro olhar primeiro para os caras com indice impar. Sao eles:
>> a1=1/1
>> a3=1/2
>> a5=2/3
>> a7=3/5
>> a8=5/8
>> ...
>> Ou seja, mostre que eles sao quocientes de numeros de Fibonacci
>> consecutivos (os caras de indice par sao os inversos desses). Agora tem
>> varias maneiras de continuar:
>>
>> -- Voce pode mostrar que os numeros de Fibonacci consecutivos sao primos
>> entre si; portanto cada fracao dessas fica unicamente determinada por
>> numerador e denominador, e (como os numeros de Fibonacci formam uma
>> sequencia crescente) vao ser distintos entre si;
>> -- Se voce nao quiser entrar no merito do Fibonacci, tente mostrar (pode
>> ser por inducao) que a3 < a7 < a11 <...> < ... < a13 < a9 < a5 < 1  (phi ali seria (raiz(5)-1) / 2, acho).
>>
>> De qualquer forma, como a_(2n+1)<1, a1=1 e os "a_2n" sao os inversos dos
>> "a_2n+1, vao ser todos diferentes.
>>
>> Abraco, Ralph.
>>
>>
>> On Sat, Feb 13, 2021 at 5:56 PM Jeferson Almir 
>> wrote:
>>
>>> Amigos, peço ajuda em provar a injetividade dessa sequência que seria
>>> uma saída para provar a unica ocorrência do racional que aparece nela.
>>> Estou andando em círculos tentando montar uma possível indução.
>>>
>>>
>>> Dado a sequência a_1 = 1 e a_2n = a_n  + 1 e a_2n+1 = 1/a_2n.
>>>
>>> Prove que para todo racional positivo que ocorre na sequência, ocorre
>>> uma única vez.
>>>
>>> --
>>> Esta mensagem foi verificada pelo sistema de antivírus e
>>> acredita-se estar livre de perigo.
>>
>>
>> --
>> Esta mensagem foi verificada pelo sistema de antivírus e
>> acredita-se estar livre de perigo.
>
>
> --
> Esta mensagem foi verificada pelo sistema de antivírus e
> acredita-se estar livre de perigo.

-- 
Esta mensagem foi verificada pelo sistema de antiv�rus e
 acredita-se estar livre de perigo.



[obm-l] Re: [obm-l] Re: [obm-l] Sequência Injetiva

2021-02-13 Por tôpico Claudio Buffara
Oi, Ralph:

Eu posso ter entendido errado a definição da sequência, mas achei termos
diferentes dos seus:
1:  1
2:  2
3:  1/2
4:  3
5:  1/3
6:  3/2
7:  2/3
8:  4
9:  1/4
10:  4/3
11:  3/4
12:  5/2
13:  2/5
14:  5/3
15:  3/5
16:  5
...

[]s,
Claudio.


On Sat, Feb 13, 2021 at 7:59 PM Ralph Costa Teixeira 
wrote:

> Meio enrolado, vou escrever meio vagamente.
>
> Eu sugiro olhar primeiro para os caras com indice impar. Sao eles:
> a1=1/1
> a3=1/2
> a5=2/3
> a7=3/5
> a8=5/8
> ...
> Ou seja, mostre que eles sao quocientes de numeros de Fibonacci
> consecutivos (os caras de indice par sao os inversos desses). Agora tem
> varias maneiras de continuar:
>
> -- Voce pode mostrar que os numeros de Fibonacci consecutivos sao primos
> entre si; portanto cada fracao dessas fica unicamente determinada por
> numerador e denominador, e (como os numeros de Fibonacci formam uma
> sequencia crescente) vao ser distintos entre si;
> -- Se voce nao quiser entrar no merito do Fibonacci, tente mostrar (pode
> ser por inducao) que a3 < a7 < a11 <... < ... < a13 < a9 < a5 < 1  (phi ali seria (raiz(5)-1) / 2, acho).
>
> De qualquer forma, como a_(2n+1)<1, a1=1 e os "a_2n" sao os inversos dos
> "a_2n+1, vao ser todos diferentes.
>
> Abraco, Ralph.
>
>
> On Sat, Feb 13, 2021 at 5:56 PM Jeferson Almir 
> wrote:
>
>> Amigos, peço ajuda em provar a injetividade dessa sequência que seria uma
>> saída para provar a unica ocorrência do racional que aparece nela. Estou
>> andando em círculos tentando montar uma possível indução.
>>
>>
>> Dado a sequência a_1 = 1 e a_2n = a_n  + 1 e a_2n+1 = 1/a_2n.
>>
>> Prove que para todo racional positivo que ocorre na sequência, ocorre uma
>> única vez.
>>
>> --
>> Esta mensagem foi verificada pelo sistema de antivírus e
>> acredita-se estar livre de perigo.
>
>
> --
> Esta mensagem foi verificada pelo sistema de antivírus e
> acredita-se estar livre de perigo.

-- 
Esta mensagem foi verificada pelo sistema de antiv�rus e
 acredita-se estar livre de perigo.



[obm-l] Re: [obm-l] Re: [obm-l] Indução

2021-02-09 Por tôpico Anderson Torres
Disfarce o Lema da Boa Ordenacao, dado que e equivalente ao principio da
inducao.

Em sex., 5 de fev. de 2021 às 07:31, joao pedro b menezes <
joaopedrobmene...@gmail.com> escreveu:

> obs: só agora fui ver o título :) , se era necessário fazer especialmente
> por indução, por favor desconsidere a minha resposta.
>
> On Fri, Feb 5, 2021 at 7:14 AM joao pedro b menezes <
> joaopedrobmene...@gmail.com> wrote:
>
>> Suponha que d | (a^(2)^n ) + 1. Então a^2^n = -1 (mod d). Pegue um primo
>> tal que p| d, então a^2^n = -1 (mod p). Mas temos: a^2^(n+1) = 1 (mod p).
>> Logo
>>  ord(p)a | 2^(n+1), mas ord(p)a não divide 2^n, logo ord(p)a = 2^(n + 1).
>> Isso é um absurdo, pois ord(p)a < p <= d <= 2^(n + 1).
>> obs: tenho quase certeza que já perguntaram a mesma coisa nessa lista.
>> Portanto acho que vale a pena ir procurar a resposta anterior também :)
>>
>> On Thu, Feb 4, 2021 at 11:20 PM Heitor Gama Ribeiro <
>> heitor...@hotmail.com> wrote:
>>
>>> Prove por indução que se 3<= d <= 2^(n+1), então d não divide
>>> [a^(2)^(n) + 1] para todo inteiro positivo a.
>>>
>>>
>>> Sent from my iPhone
>>>
>>> =
>>> Instruções para entrar na lista, sair da lista e usar a lista em
>>> http://www.mat.puc-rio.br/~obmlistas/obm-l.html
>>> =
>>>
>>

-- 
Esta mensagem foi verificada pelo sistema de antiv�rus e
 acredita-se estar livre de perigo.



[obm-l] Re: [obm-l] Re: [obm-l] Re: [obm-l] Função parte inteira

2021-02-03 Por tôpico Ralph Costa Teixeira
Hm, confere o enunciado - era parte inteira, ou inteiro mais proximo?

On Wed, Feb 3, 2021, 18:39 joao pedro b menezes 
wrote:

> Obrigado pela dica! Honestamente creio que existe um erro nesse problema.
> Fazendo alguns casos na mão é possivel perceber que isso sempre resulta em
> 8n + 7. Essa é a prova:
> "Provar que ( n^(1/3) + ( n + 2)^(1/3) )³  < 8n + 8. Abrindo a potência,
> temos:
> 2n + 2 + 3 * ( (n² ( n + 2))^(1/3) + (n(n + 2)²)^(1/3)) < 8n + 8
>   (n² ( n + 2))^(1/3) + (n(n + 2)²)^(1/3)   < 2n + 2
> Porém temos que  (n² ( n + 2))^(1/3) < n + 2/3  , e  (n(n + 2)²)^(1/3) <
> n + 4/3 ( eu testei elevando ambos os lados ao cubo deu certo) . Isso
> confirma a inequação inicial.
> Agora se 8n + 7 <  ( n^(1/3) + ( n + 2)^(1/3) )³  o exercício acaba. De
> fato, trabalhando a expressão:
>(n² ( n + 2))^(1/3) + (n(n + 2)²)^(1/3)   > 2n + 5/3
> Mas novamente, tem se que  (n² ( n + 2))^(1/3) > n + 1/2 e  (n(n +
> 2)²)^(1/3) > n + 7/6 para qualquer n > 1 ( no caso n =1 basta testar na
> mão). E como 1/2 + 7/6 = 5/3 ,  tem se que ela é verdade, logo:
> 8n + 7 <  ( n^(1/3) + ( n + 2)^(1/3) )³  < 8n + 8 ==> [ ( n^(1/3) + ( n +
> 2)^(1/3) )³ ] = 8n + 7"
> Eu estranhei bastante porque nunca tinha acontecido de um exercicio do
> POTI estar errado.
> obs: Se a minha solução estiver errada de alguma forma, adoraria saber!
>
> On Wed, Feb 3, 2021 at 12:42 PM Ralph Costa Teixeira 
> wrote:
>
>> Sem tempo agora, mas olhando por alto eu aproximaria o que estah dentro
>> do () por 2(n+1)^(1/3), o que levaria imediatamente a 8(n+1). Serah que a
>> parte inteira daquela coisa eh 8(n+1)?
>>
>> Entao eu tentaria abrir os cubos, subtrair 8(n+1), e mostrar que o que
>> sobra eh menor que 1.
>>
>> Serah que funciona?
>>
>> On Wed, Feb 3, 2021 at 10:03 AM joao pedro b menezes <
>> joaopedrobmene...@gmail.com> wrote:
>>
>>> Olá, estava tentando fazer esta questão:
>>>   Prove que [ ( n^(1/3) + (n + 2)^(1/3)  )³] é divisível por 8.
>>> obs: não tinha a tecla de função parte inteira, por isso escolhi [ ]
>>> Se alguém tiver alguma dica ou souber como resolver, ajudaria bastante.
>>>
>>


[obm-l] Re: [obm-l] Re: [obm-l] Função parte inteira

2021-02-03 Por tôpico joao pedro b menezes
Obrigado pela dica! Honestamente creio que existe um erro nesse problema.
Fazendo alguns casos na mão é possivel perceber que isso sempre resulta em
8n + 7. Essa é a prova:
"Provar que ( n^(1/3) + ( n + 2)^(1/3) )³  < 8n + 8. Abrindo a potência,
temos:
2n + 2 + 3 * ( (n² ( n + 2))^(1/3) + (n(n + 2)²)^(1/3)) < 8n + 8
  (n² ( n + 2))^(1/3) + (n(n + 2)²)^(1/3)   < 2n + 2
Porém temos que  (n² ( n + 2))^(1/3) < n + 2/3  , e  (n(n + 2)²)^(1/3) <
n + 4/3 ( eu testei elevando ambos os lados ao cubo deu certo) . Isso
confirma a inequação inicial.
Agora se 8n + 7 <  ( n^(1/3) + ( n + 2)^(1/3) )³  o exercício acaba. De
fato, trabalhando a expressão:
   (n² ( n + 2))^(1/3) + (n(n + 2)²)^(1/3)   > 2n + 5/3
Mas novamente, tem se que  (n² ( n + 2))^(1/3) > n + 1/2 e  (n(n +
2)²)^(1/3) > n + 7/6 para qualquer n > 1 ( no caso n =1 basta testar na
mão). E como 1/2 + 7/6 = 5/3 ,  tem se que ela é verdade, logo:
8n + 7 <  ( n^(1/3) + ( n + 2)^(1/3) )³  < 8n + 8 ==> [ ( n^(1/3) + ( n +
2)^(1/3) )³ ] = 8n + 7"
Eu estranhei bastante porque nunca tinha acontecido de um exercicio do POTI
estar errado.
obs: Se a minha solução estiver errada de alguma forma, adoraria saber!

On Wed, Feb 3, 2021 at 12:42 PM Ralph Costa Teixeira 
wrote:

> Sem tempo agora, mas olhando por alto eu aproximaria o que estah dentro do
> () por 2(n+1)^(1/3), o que levaria imediatamente a 8(n+1). Serah que a
> parte inteira daquela coisa eh 8(n+1)?
>
> Entao eu tentaria abrir os cubos, subtrair 8(n+1), e mostrar que o que
> sobra eh menor que 1.
>
> Serah que funciona?
>
> On Wed, Feb 3, 2021 at 10:03 AM joao pedro b menezes <
> joaopedrobmene...@gmail.com> wrote:
>
>> Olá, estava tentando fazer esta questão:
>>   Prove que [ ( n^(1/3) + (n + 2)^(1/3)  )³] é divisível por 8.
>> obs: não tinha a tecla de função parte inteira, por isso escolhi [ ]
>> Se alguém tiver alguma dica ou souber como resolver, ajudaria bastante.
>>
>


[obm-l] Re: [obm-l] Re: [obm-l] Ângulos de um triângulo

2020-12-06 Por tôpico Anderson Torres
Em sáb., 5 de dez. de 2020 às 07:15, Julio César Saldaña Pumarica <
saldana...@pucp.edu.pe> escreveu:

> É verdade, 30 graus é o DAB, más a pergunta era DAC
>
> o  DAC=18
>
>
> On Fri, Dec 4, 2020, 19:23 Julio César Saldaña Pumarica <
> saldana...@pucp.edu.pe> wrote:
>
>> Tenho uma solução com traços auxiliares. Resposta: 30°
>>
>> Tem como passar uma foto nesta lista?
>>
>

Use um site de compartilhamento de fotos, como o IMGUR.

Ou faça melhor: descreva a figura como se fosse explicar para um cego.



>
>> On Mon, Nov 30, 2020, 19:42 Professor Vanderlei Nemitz <
>> vanderma...@gmail.com> wrote:
>>
>>> Boa noite!
>>> Alguém conhece uma saída para o seguinte problema?
>>> Muito obrigado!
>>>
>>> *Num triângulo isósceles ABC, AB = AC.*
>>> *Seja D um ponto interno tal que os ângulos DBC, DCB, DBA e DCA medem,
>>> respectivamente, 12°, 18°, 54° e 48°. *
>>> *Determine a medida do ângulo DAC.*
>>>
>>>
>>> 
>>>  Livre
>>> de vírus. www.avast.com
>>> .
>>>
>>> <#m_-3450317920574181893_m_-1377089701738023066_m_6233189016624438778_m_1892860381202400781_DAB4FAD8-2DD7-40BB-A1B8-4E2AA1F9FDF2>
>>>
>>


[obm-l] Re: [obm-l] Re: [obm-l] Re: [obm-l] Ângulos de um triângulo

2020-12-04 Por tôpico Pedro Henrique de Almeida Ursino
Seja x a medida do ângulo DAC (logo DAB mede 48 -x). Por trig Ceva

sin x * sin 18 * sin 54 = sin (48-x) * sin 12 * sin 48.

Pode-se deduzir que sin 54 = (1+ sqrt(5))/4 e sin 18 = (sqrt(5)-1)/4. Logo,
sin 54 * sin 18 = 1/4. Assim, nossa equação fica

sin x / sin (48-x) = 4 * sin 12 * sin 48

usando Werner, temos que 2 *sin 12 * sin 48 = cos(48 -12) - cos (48 + 12) =
cos 36 - cos 60 = cos 36 - 1/2
Desse modo, nossa relação fica

sinx/ sin(48-x) = 2*cos 36 - 1 = 2*sin 18

Daí é muito trivial ver que 18 é solução. Esta solução é única pois f(x) =
sin x/ sin(48-x) é crescente para x entre 0 e 48 (graus).

Essa solução usa muitas relações trigonométricas não tão conhecidas
assim Essa foi uma das soluções dadas pelo prof. Sandro do Canal: A
hora do Bizu em seu último vídeo. Ele também dá uma solução sintética para
o problema. Enfim, vale a pena conferir.

On Fri, Dec 4, 2020 at 4:13 PM Armando Staib 
wrote:

> Não querendo polemizar, mas de acordo com o exercício, é, na minha
> opinião, impossível ser 30 o ângulo pedido  pq se fosse o triângulo DBC
> teria o lado oposto ao ângulo de 18 menor do que o lado oposto ao ângulo de
> 12.
>
> Se me enganei poderiam me mostrar, onde eu errei?
>
> Em sex., 4 de dez. de 2020 às 14:06, Claudio Buffara <
> claudio.buff...@gmail.com> escreveu:
>
>> Aliás, de posse da expressão para BAD e CAD, um exercício razoavelmente
>> fácil de programação (até em planilha), é descobrir para quais triângulos
>> isósceles com ângulos inteiros (em graus) e quais ângulos DBC e DCB
>> inteiros, BAD (e obviamente CAD) também são inteiros.
>>
>> Daí, um problema (não mais um exercício!) é descobrir o padrão por trás
>> destes triângulos especiais.
>>
>> On Fri, Dec 4, 2020 at 1:42 PM Claudio Buffara 
>> wrote:
>>
>>> Usando áreas - em particular, área(ABC) = (1/2)*AB*AC*sen(A) - você
>>> consegue, com alguma facilidade, expressar a tangente de DAC em termos de
>>> senos e cossenos dos ângulos dados.   Daí, é só calcular (com calculadora
>>> ou computador - eu uso Excel ou Wolfram Alpha).  E, de fato, AD divide BAC,
>>> que mede 48 graus, em dois ângulos: um medindo 30 e o outro 18 graus.
>>>
>>> O que não dá é - em 2020 - ficar manipulando aquelas fórmulas de
>>> prostaférese ou identidades trigonométricas obscuras envolvendo ângulos
>>> múltiplos de 3 graus. Isso é coisa do século 19...
>>>
>>> []s,
>>> Claudio.
>>>
>>> On Mon, Nov 30, 2020 at 7:28 PM Professor Vanderlei Nemitz <
>>> vanderma...@gmail.com> wrote:
>>>
 Boa noite!
 Alguém conhece uma saída para o seguinte problema?
 Muito obrigado!

 *Num triângulo isósceles ABC, AB = AC.*
 *Seja D um ponto interno tal que os ângulos DBC, DCB, DBA e DCA medem,
 respectivamente, 12°, 18°, 54° e 48°. *
 *Determine a medida do ângulo DAC.*


 
  Livre
 de vírus. www.avast.com
 .

 <#m_-9147473276317047412_m_6354815198100344298_m_-4747407596740689255_m_4608836649714424769_DAB4FAD8-2DD7-40BB-A1B8-4E2AA1F9FDF2>

>>>


[obm-l] Re: [obm-l] Re: [obm-l] Ângulos de um triângulo

2020-12-04 Por tôpico Armando Staib
Não querendo polemizar, mas de acordo com o exercício, é, na minha opinião,
impossível ser 30 o ângulo pedido  pq se fosse o triângulo DBC teria o lado
oposto ao ângulo de 18 menor do que o lado oposto ao ângulo de 12.

Se me enganei poderiam me mostrar, onde eu errei?

Em sex., 4 de dez. de 2020 às 14:06, Claudio Buffara <
claudio.buff...@gmail.com> escreveu:

> Aliás, de posse da expressão para BAD e CAD, um exercício razoavelmente
> fácil de programação (até em planilha), é descobrir para quais triângulos
> isósceles com ângulos inteiros (em graus) e quais ângulos DBC e DCB
> inteiros, BAD (e obviamente CAD) também são inteiros.
>
> Daí, um problema (não mais um exercício!) é descobrir o padrão por trás
> destes triângulos especiais.
>
> On Fri, Dec 4, 2020 at 1:42 PM Claudio Buffara 
> wrote:
>
>> Usando áreas - em particular, área(ABC) = (1/2)*AB*AC*sen(A) - você
>> consegue, com alguma facilidade, expressar a tangente de DAC em termos de
>> senos e cossenos dos ângulos dados.   Daí, é só calcular (com calculadora
>> ou computador - eu uso Excel ou Wolfram Alpha).  E, de fato, AD divide BAC,
>> que mede 48 graus, em dois ângulos: um medindo 30 e o outro 18 graus.
>>
>> O que não dá é - em 2020 - ficar manipulando aquelas fórmulas de
>> prostaférese ou identidades trigonométricas obscuras envolvendo ângulos
>> múltiplos de 3 graus. Isso é coisa do século 19...
>>
>> []s,
>> Claudio.
>>
>> On Mon, Nov 30, 2020 at 7:28 PM Professor Vanderlei Nemitz <
>> vanderma...@gmail.com> wrote:
>>
>>> Boa noite!
>>> Alguém conhece uma saída para o seguinte problema?
>>> Muito obrigado!
>>>
>>> *Num triângulo isósceles ABC, AB = AC.*
>>> *Seja D um ponto interno tal que os ângulos DBC, DCB, DBA e DCA medem,
>>> respectivamente, 12°, 18°, 54° e 48°. *
>>> *Determine a medida do ângulo DAC.*
>>>
>>>
>>> 
>>>  Livre
>>> de vírus. www.avast.com
>>> .
>>>
>>> <#m_6354815198100344298_m_-4747407596740689255_m_4608836649714424769_DAB4FAD8-2DD7-40BB-A1B8-4E2AA1F9FDF2>
>>>
>>


[obm-l] Re: [obm-l] Re: [obm-l] Ângulos de um triângulo

2020-12-04 Por tôpico Carlos Victor
 

Use a lei dos senos e o fato de que sen(54º)-sen(18º)=sen(30º). 

Em 04/12/2020 1:50, Anderson Torres escreveu: 

> Em seg., 30 de nov. de 2020 às 19:28, Professor Vanderlei Nemitz 
>  escreveu: 
> 
>> Boa noite!
>> 
>> Alguém conhece uma saída para o seguinte problema? 
>> Muito obrigado! 
>> 
>> NUM TRIÂNGULO ISÓSCELES ABC, AB = AC. 
>> SEJA D UM PONTO INTERNO TAL QUE OS ÂNGULOS DBC, DCB, DBA E DCA MEDEM, 
>> RESPECTIVAMENTE, 12°, 18°, 54° E 48°. 
>> DETERMINE A MEDIDA DO ÂNGULO DAC.
> 
> Eu ainda nao resolvi, mas sei que e 30 graus. 
> 
>> [1]
>> Livre de vírus. www.avast.com [1].

 

Links:
--
[1]
https://www.avast.com/sig-email?utm_medium=emailutm_source=linkutm_campaign=sig-emailutm_content=webmail

[obm-l] Re: [obm-l] Re: [obm-l] Ângulos de um triângulo

2020-12-04 Por tôpico Armando Staib
ou 18!?

Em sex., 4 de dez. de 2020 às 02:08, Anderson Torres <
torres.anderson...@gmail.com> escreveu:

>
>
> Em seg., 30 de nov. de 2020 às 19:28, Professor Vanderlei Nemitz <
> vanderma...@gmail.com> escreveu:
>
>> Boa noite!
>> Alguém conhece uma saída para o seguinte problema?
>> Muito obrigado!
>>
>> *Num triângulo isósceles ABC, AB = AC.*
>> *Seja D um ponto interno tal que os ângulos DBC, DCB, DBA e DCA medem,
>> respectivamente, 12°, 18°, 54° e 48°. *
>> *Determine a medida do ângulo DAC.*
>>
>
> Eu ainda nao resolvi, mas sei que e 30 graus.
>
>
>
>>
>>
>> 
>>  Livre
>> de vírus. www.avast.com
>> .
>>
>> <#m_7328427911962770194_m_-7214013898439594478_DAB4FAD8-2DD7-40BB-A1B8-4E2AA1F9FDF2>
>>
>


[obm-l] Re: [obm-l] Re: [obm-l] Re: [obm-l] Re: [obm-l] Re: [obm-l] Re: [obm-l] Relação de girard

2020-11-16 Por tôpico Claudio Buffara
Sugestão: proponha pra eles o problema de determinar se é possível atribuir
sinais "+" ou "-" a cada um dos números:
1  2  3  4  5  6  7  8  9  10
de modo que a soma algébrica (com sinal) destes números seja igual a zero.
Isso é um desafio e é razoavelmente lúdico, apesar de envolver conceitos
que uma criança de 8 anos entenderia.

On Sat, Nov 14, 2020 at 4:22 PM Israel Meireles Chrisostomo <
israelmchrisost...@gmail.com> wrote:

> Desculpe é q eu queria propor algo q fosse lúdico, mais um desafio,
> voltada para jovens adolescentes, algo descompromissado, sem muitas
> complicações com formalidades
>
> Em qui, 12 de nov de 2020 09:10, Anderson Torres <
> torres.anderson...@gmail.com> escreveu:
>
>>
>>
>> Em sáb., 7 de nov. de 2020 às 16:44, Israel Meireles Chrisostomo <
>> israelmchrisost...@gmail.com> escreveu:
>>
>>>   o objetivo dessa proposta é recriar o ambiente vivido por Euler na
>>> época.
>>>
>>
>> E naquele tempo eles não usavam indução? Formalização é algo bem recente
>> na matemática.
>>
>> Sua exigência me parece algo tão surreal quanto exigir rigor na geometria
>> do tempo de Euclides.
>>
>>
>>
>>>
>>> Em sáb., 7 de nov. de 2020 às 15:10, Israel Meireles Chrisostomo <
>>> israelmchrisost...@gmail.com> escreveu:
>>>
 Na verdade eu estava elaborando um problema que dependia disso.O
 problema é esse aqui:

 Desafio do ano: resolver o problema da Basiléia sem usar derivadas,
 integrais, série de potências, produto infinito do seno ou cosseno, ou
 mesmo indução ou números complexos.

 Em sáb., 7 de nov. de 2020 às 15:07, Israel Meireles Chrisostomo <
 israelmchrisost...@gmail.com> escreveu:

> Na verdade eu estava elaborando um problema que dependia disso.O
> problema é esse aqui:
>
> Desafio do ano: resolver o problema da Basiléia sem usar derivadas,
> integrais, série de potências, produto infinito do seno ou cosseno, ou
> mesmo indução.
>
> Em sáb., 7 de nov. de 2020 às 14:47, Israel Meireles Chrisostomo <
> israelmchrisost...@gmail.com> escreveu:
>
>> conheço uma que usa o teorema de d'lambert
>>
>> Em sáb., 7 de nov. de 2020 às 12:50, Bernardo Freitas Paulo da Costa <
>> bernardo...@gmail.com> escreveu:
>>
>>> On Thu, Nov 5, 2020 at 9:26 PM Artur Costa Steiner
>>>  wrote:
>>> >
>>> > Para facilitar, suponhamos que o polinômio de grau n P seja
>>> mônico. Sejam z_1, , z_n suas n raízes não necessariamente 
>>> distintas.
>>> Para todo complexo z, temos que
>>> >
>>> > P(z)  = ( z - z_1) (z - z_n)
>>> >
>>> > Desenvolvendo e aplicando o chamado produto de Stevin, vc tem as
>>> relações de Girard.
>>>
>>> Eu não conhecia o produto de Stevin, mas de forma geral quando você
>>> usa "..." tem, muitas vezes, um argumento por indução que está
>>> subentendido.  Pode ser que o produto de Stevin "faça a indução pra
>>> você" (calculando os termos \sum \prod z_i que vão aparecer como
>>> coeficientes dos monômios z^k), mas é "quase" como se você estivesse
>>> empurrando a indução um andar abaixo ;-)
>>>
>>>
>>> Israel: qual a demonstração por indução que você conhece?  E porque
>>> você gostaria de outra??
>>>
>>> Abraços,
>>> --
>>> Bernardo Freitas Paulo da Costa
>>>
>>>
>>> =
>>> Instru�ões para entrar na lista, sair da lista e usar a lista em
>>> http://www.mat.puc-rio.br/~obmlistas/obm-l.html
>>>
>>> =
>>>
>>
>>
>> --
>> Israel Meireles Chrisostomo
>>
>
>
> --
> Israel Meireles Chrisostomo
>


 --
 Israel Meireles Chrisostomo

>>>
>>>
>>> --
>>> Israel Meireles Chrisostomo
>>>
>>


[obm-l] Re: [obm-l] Re: [obm-l] Re: [obm-l] Re: [obm-l] Re: [obm-l] Relação de girard

2020-11-14 Por tôpico Israel Meireles Chrisostomo
Desculpe é q eu queria propor algo q fosse lúdico, mais um desafio,
voltada para jovens adolescentes, algo descompromissado, sem muitas
complicações com formalidades

Em qui, 12 de nov de 2020 09:10, Anderson Torres <
torres.anderson...@gmail.com> escreveu:

>
>
> Em sáb., 7 de nov. de 2020 às 16:44, Israel Meireles Chrisostomo <
> israelmchrisost...@gmail.com> escreveu:
>
>>   o objetivo dessa proposta é recriar o ambiente vivido por Euler na
>> época.
>>
>
> E naquele tempo eles não usavam indução? Formalização é algo bem recente
> na matemática.
>
> Sua exigência me parece algo tão surreal quanto exigir rigor na geometria
> do tempo de Euclides.
>
>
>
>>
>> Em sáb., 7 de nov. de 2020 às 15:10, Israel Meireles Chrisostomo <
>> israelmchrisost...@gmail.com> escreveu:
>>
>>> Na verdade eu estava elaborando um problema que dependia disso.O
>>> problema é esse aqui:
>>>
>>> Desafio do ano: resolver o problema da Basiléia sem usar derivadas,
>>> integrais, série de potências, produto infinito do seno ou cosseno, ou
>>> mesmo indução ou números complexos.
>>>
>>> Em sáb., 7 de nov. de 2020 às 15:07, Israel Meireles Chrisostomo <
>>> israelmchrisost...@gmail.com> escreveu:
>>>
 Na verdade eu estava elaborando um problema que dependia disso.O
 problema é esse aqui:

 Desafio do ano: resolver o problema da Basiléia sem usar derivadas,
 integrais, série de potências, produto infinito do seno ou cosseno, ou
 mesmo indução.

 Em sáb., 7 de nov. de 2020 às 14:47, Israel Meireles Chrisostomo <
 israelmchrisost...@gmail.com> escreveu:

> conheço uma que usa o teorema de d'lambert
>
> Em sáb., 7 de nov. de 2020 às 12:50, Bernardo Freitas Paulo da Costa <
> bernardo...@gmail.com> escreveu:
>
>> On Thu, Nov 5, 2020 at 9:26 PM Artur Costa Steiner
>>  wrote:
>> >
>> > Para facilitar, suponhamos que o polinômio de grau n P seja mônico.
>> Sejam z_1, , z_n suas n raízes não necessariamente distintas. Para 
>> todo
>> complexo z, temos que
>> >
>> > P(z)  = ( z - z_1) (z - z_n)
>> >
>> > Desenvolvendo e aplicando o chamado produto de Stevin, vc tem as
>> relações de Girard.
>>
>> Eu não conhecia o produto de Stevin, mas de forma geral quando você
>> usa "..." tem, muitas vezes, um argumento por indução que está
>> subentendido.  Pode ser que o produto de Stevin "faça a indução pra
>> você" (calculando os termos \sum \prod z_i que vão aparecer como
>> coeficientes dos monômios z^k), mas é "quase" como se você estivesse
>> empurrando a indução um andar abaixo ;-)
>>
>>
>> Israel: qual a demonstração por indução que você conhece?  E porque
>> você gostaria de outra??
>>
>> Abraços,
>> --
>> Bernardo Freitas Paulo da Costa
>>
>>
>> =
>> Instru�ões para entrar na lista, sair da lista e usar a lista em
>> http://www.mat.puc-rio.br/~obmlistas/obm-l.html
>>
>> =
>>
>
>
> --
> Israel Meireles Chrisostomo
>


 --
 Israel Meireles Chrisostomo

>>>
>>>
>>> --
>>> Israel Meireles Chrisostomo
>>>
>>
>>
>> --
>> Israel Meireles Chrisostomo
>>
>


[obm-l] Re: [obm-l] Re: [obm-l] Re: [obm-l] Re: [obm-l] Relação de girard

2020-11-12 Por tôpico Anderson Torres
Em sáb., 7 de nov. de 2020 às 16:44, Israel Meireles Chrisostomo <
israelmchrisost...@gmail.com> escreveu:

>   o objetivo dessa proposta é recriar o ambiente vivido por Euler na
> época.
>

E naquele tempo eles não usavam indução? Formalização é algo bem recente na
matemática.

Sua exigência me parece algo tão surreal quanto exigir rigor na geometria
do tempo de Euclides.



>
> Em sáb., 7 de nov. de 2020 às 15:10, Israel Meireles Chrisostomo <
> israelmchrisost...@gmail.com> escreveu:
>
>> Na verdade eu estava elaborando um problema que dependia disso.O problema
>> é esse aqui:
>>
>> Desafio do ano: resolver o problema da Basiléia sem usar derivadas,
>> integrais, série de potências, produto infinito do seno ou cosseno, ou
>> mesmo indução ou números complexos.
>>
>> Em sáb., 7 de nov. de 2020 às 15:07, Israel Meireles Chrisostomo <
>> israelmchrisost...@gmail.com> escreveu:
>>
>>> Na verdade eu estava elaborando um problema que dependia disso.O
>>> problema é esse aqui:
>>>
>>> Desafio do ano: resolver o problema da Basiléia sem usar derivadas,
>>> integrais, série de potências, produto infinito do seno ou cosseno, ou
>>> mesmo indução.
>>>
>>> Em sáb., 7 de nov. de 2020 às 14:47, Israel Meireles Chrisostomo <
>>> israelmchrisost...@gmail.com> escreveu:
>>>
 conheço uma que usa o teorema de d'lambert

 Em sáb., 7 de nov. de 2020 às 12:50, Bernardo Freitas Paulo da Costa <
 bernardo...@gmail.com> escreveu:

> On Thu, Nov 5, 2020 at 9:26 PM Artur Costa Steiner
>  wrote:
> >
> > Para facilitar, suponhamos que o polinômio de grau n P seja mônico.
> Sejam z_1, , z_n suas n raízes não necessariamente distintas. Para 
> todo
> complexo z, temos que
> >
> > P(z)  = ( z - z_1) (z - z_n)
> >
> > Desenvolvendo e aplicando o chamado produto de Stevin, vc tem as
> relações de Girard.
>
> Eu não conhecia o produto de Stevin, mas de forma geral quando você
> usa "..." tem, muitas vezes, um argumento por indução que está
> subentendido.  Pode ser que o produto de Stevin "faça a indução pra
> você" (calculando os termos \sum \prod z_i que vão aparecer como
> coeficientes dos monômios z^k), mas é "quase" como se você estivesse
> empurrando a indução um andar abaixo ;-)
>
>
> Israel: qual a demonstração por indução que você conhece?  E porque
> você gostaria de outra??
>
> Abraços,
> --
> Bernardo Freitas Paulo da Costa
>
>
> =
> Instru�ões para entrar na lista, sair da lista e usar a lista em
> http://www.mat.puc-rio.br/~obmlistas/obm-l.html
>
> =
>


 --
 Israel Meireles Chrisostomo

>>>
>>>
>>> --
>>> Israel Meireles Chrisostomo
>>>
>>
>>
>> --
>> Israel Meireles Chrisostomo
>>
>
>
> --
> Israel Meireles Chrisostomo
>


[obm-l] Re: [obm-l] Re: [obm-l] Re: [obm-l] Relação de girard

2020-11-07 Por tôpico Israel Meireles Chrisostomo
  o objetivo dessa proposta é recriar o ambiente vivido por Euler na
época.

Em sáb., 7 de nov. de 2020 às 15:10, Israel Meireles Chrisostomo <
israelmchrisost...@gmail.com> escreveu:

> Na verdade eu estava elaborando um problema que dependia disso.O problema
> é esse aqui:
>
> Desafio do ano: resolver o problema da Basiléia sem usar derivadas,
> integrais, série de potências, produto infinito do seno ou cosseno, ou
> mesmo indução ou números complexos.
>
> Em sáb., 7 de nov. de 2020 às 15:07, Israel Meireles Chrisostomo <
> israelmchrisost...@gmail.com> escreveu:
>
>> Na verdade eu estava elaborando um problema que dependia disso.O problema
>> é esse aqui:
>>
>> Desafio do ano: resolver o problema da Basiléia sem usar derivadas,
>> integrais, série de potências, produto infinito do seno ou cosseno, ou
>> mesmo indução.
>>
>> Em sáb., 7 de nov. de 2020 às 14:47, Israel Meireles Chrisostomo <
>> israelmchrisost...@gmail.com> escreveu:
>>
>>> conheço uma que usa o teorema de d'lambert
>>>
>>> Em sáb., 7 de nov. de 2020 às 12:50, Bernardo Freitas Paulo da Costa <
>>> bernardo...@gmail.com> escreveu:
>>>
 On Thu, Nov 5, 2020 at 9:26 PM Artur Costa Steiner
  wrote:
 >
 > Para facilitar, suponhamos que o polinômio de grau n P seja mônico.
 Sejam z_1, , z_n suas n raízes não necessariamente distintas. Para todo
 complexo z, temos que
 >
 > P(z)  = ( z - z_1) (z - z_n)
 >
 > Desenvolvendo e aplicando o chamado produto de Stevin, vc tem as
 relações de Girard.

 Eu não conhecia o produto de Stevin, mas de forma geral quando você
 usa "..." tem, muitas vezes, um argumento por indução que está
 subentendido.  Pode ser que o produto de Stevin "faça a indução pra
 você" (calculando os termos \sum \prod z_i que vão aparecer como
 coeficientes dos monômios z^k), mas é "quase" como se você estivesse
 empurrando a indução um andar abaixo ;-)


 Israel: qual a demonstração por indução que você conhece?  E porque
 você gostaria de outra??

 Abraços,
 --
 Bernardo Freitas Paulo da Costa


 =
 Instru�ões para entrar na lista, sair da lista e usar a lista em
 http://www.mat.puc-rio.br/~obmlistas/obm-l.html

 =

>>>
>>>
>>> --
>>> Israel Meireles Chrisostomo
>>>
>>
>>
>> --
>> Israel Meireles Chrisostomo
>>
>
>
> --
> Israel Meireles Chrisostomo
>


-- 
Israel Meireles Chrisostomo


[obm-l] Re: [obm-l] Re: [obm-l] Re: [obm-l] Relação de girard

2020-11-07 Por tôpico Israel Meireles Chrisostomo
Na verdade eu estava elaborando um problema que dependia disso.O problema é
esse aqui:

Desafio do ano: resolver o problema da Basiléia sem usar derivadas,
integrais, série de potências, produto infinito do seno ou cosseno, ou
mesmo indução ou números complexos.

Em sáb., 7 de nov. de 2020 às 15:07, Israel Meireles Chrisostomo <
israelmchrisost...@gmail.com> escreveu:

> Na verdade eu estava elaborando um problema que dependia disso.O problema
> é esse aqui:
>
> Desafio do ano: resolver o problema da Basiléia sem usar derivadas,
> integrais, série de potências, produto infinito do seno ou cosseno, ou
> mesmo indução.
>
> Em sáb., 7 de nov. de 2020 às 14:47, Israel Meireles Chrisostomo <
> israelmchrisost...@gmail.com> escreveu:
>
>> conheço uma que usa o teorema de d'lambert
>>
>> Em sáb., 7 de nov. de 2020 às 12:50, Bernardo Freitas Paulo da Costa <
>> bernardo...@gmail.com> escreveu:
>>
>>> On Thu, Nov 5, 2020 at 9:26 PM Artur Costa Steiner
>>>  wrote:
>>> >
>>> > Para facilitar, suponhamos que o polinômio de grau n P seja mônico.
>>> Sejam z_1, , z_n suas n raízes não necessariamente distintas. Para todo
>>> complexo z, temos que
>>> >
>>> > P(z)  = ( z - z_1) (z - z_n)
>>> >
>>> > Desenvolvendo e aplicando o chamado produto de Stevin, vc tem as
>>> relações de Girard.
>>>
>>> Eu não conhecia o produto de Stevin, mas de forma geral quando você
>>> usa "..." tem, muitas vezes, um argumento por indução que está
>>> subentendido.  Pode ser que o produto de Stevin "faça a indução pra
>>> você" (calculando os termos \sum \prod z_i que vão aparecer como
>>> coeficientes dos monômios z^k), mas é "quase" como se você estivesse
>>> empurrando a indução um andar abaixo ;-)
>>>
>>>
>>> Israel: qual a demonstração por indução que você conhece?  E porque
>>> você gostaria de outra??
>>>
>>> Abraços,
>>> --
>>> Bernardo Freitas Paulo da Costa
>>>
>>> =
>>> Instru�ões para entrar na lista, sair da lista e usar a lista em
>>> http://www.mat.puc-rio.br/~obmlistas/obm-l.html
>>> =
>>>
>>
>>
>> --
>> Israel Meireles Chrisostomo
>>
>
>
> --
> Israel Meireles Chrisostomo
>


-- 
Israel Meireles Chrisostomo


[obm-l] Re: [obm-l] Re: [obm-l] Re: [obm-l] Relação de girard

2020-11-07 Por tôpico Israel Meireles Chrisostomo
Na verdade eu estava elaborando um problema que dependia disso.O problema é
esse aqui:

Desafio do ano: resolver o problema da Basiléia sem usar derivadas,
integrais, série de potências, produto infinito do seno ou cosseno, ou
mesmo indução.

Em sáb., 7 de nov. de 2020 às 14:47, Israel Meireles Chrisostomo <
israelmchrisost...@gmail.com> escreveu:

> conheço uma que usa o teorema de d'lambert
>
> Em sáb., 7 de nov. de 2020 às 12:50, Bernardo Freitas Paulo da Costa <
> bernardo...@gmail.com> escreveu:
>
>> On Thu, Nov 5, 2020 at 9:26 PM Artur Costa Steiner
>>  wrote:
>> >
>> > Para facilitar, suponhamos que o polinômio de grau n P seja mônico.
>> Sejam z_1, , z_n suas n raízes não necessariamente distintas. Para todo
>> complexo z, temos que
>> >
>> > P(z)  = ( z - z_1) (z - z_n)
>> >
>> > Desenvolvendo e aplicando o chamado produto de Stevin, vc tem as
>> relações de Girard.
>>
>> Eu não conhecia o produto de Stevin, mas de forma geral quando você
>> usa "..." tem, muitas vezes, um argumento por indução que está
>> subentendido.  Pode ser que o produto de Stevin "faça a indução pra
>> você" (calculando os termos \sum \prod z_i que vão aparecer como
>> coeficientes dos monômios z^k), mas é "quase" como se você estivesse
>> empurrando a indução um andar abaixo ;-)
>>
>>
>> Israel: qual a demonstração por indução que você conhece?  E porque
>> você gostaria de outra??
>>
>> Abraços,
>> --
>> Bernardo Freitas Paulo da Costa
>>
>> =
>> Instru�ões para entrar na lista, sair da lista e usar a lista em
>> http://www.mat.puc-rio.br/~obmlistas/obm-l.html
>> =
>>
>
>
> --
> Israel Meireles Chrisostomo
>


-- 
Israel Meireles Chrisostomo


[obm-l] Re: [obm-l] Re: [obm-l] Re: [obm-l] Relação de girard

2020-11-07 Por tôpico Israel Meireles Chrisostomo
conheço uma que usa o teorema de d'lambert

Em sáb., 7 de nov. de 2020 às 12:50, Bernardo Freitas Paulo da Costa <
bernardo...@gmail.com> escreveu:

> On Thu, Nov 5, 2020 at 9:26 PM Artur Costa Steiner
>  wrote:
> >
> > Para facilitar, suponhamos que o polinômio de grau n P seja mônico.
> Sejam z_1, , z_n suas n raízes não necessariamente distintas. Para todo
> complexo z, temos que
> >
> > P(z)  = ( z - z_1) (z - z_n)
> >
> > Desenvolvendo e aplicando o chamado produto de Stevin, vc tem as
> relações de Girard.
>
> Eu não conhecia o produto de Stevin, mas de forma geral quando você
> usa "..." tem, muitas vezes, um argumento por indução que está
> subentendido.  Pode ser que o produto de Stevin "faça a indução pra
> você" (calculando os termos \sum \prod z_i que vão aparecer como
> coeficientes dos monômios z^k), mas é "quase" como se você estivesse
> empurrando a indução um andar abaixo ;-)
>
>
> Israel: qual a demonstração por indução que você conhece?  E porque
> você gostaria de outra??
>
> Abraços,
> --
> Bernardo Freitas Paulo da Costa
>
> =
> Instru�ões para entrar na lista, sair da lista e usar a lista em
> http://www.mat.puc-rio.br/~obmlistas/obm-l.html
> =
>


-- 
Israel Meireles Chrisostomo


[obm-l] Re: [obm-l] Re: [obm-l] Relação de girard

2020-11-07 Por tôpico Bernardo Freitas Paulo da Costa
On Thu, Nov 5, 2020 at 9:26 PM Artur Costa Steiner
 wrote:
>
> Para facilitar, suponhamos que o polinômio de grau n P seja mônico. Sejam 
> z_1, , z_n suas n raízes não necessariamente distintas. Para todo 
> complexo z, temos que
>
> P(z)  = ( z - z_1) (z - z_n)
>
> Desenvolvendo e aplicando o chamado produto de Stevin, vc tem as relações de 
> Girard.

Eu não conhecia o produto de Stevin, mas de forma geral quando você
usa "..." tem, muitas vezes, um argumento por indução que está
subentendido.  Pode ser que o produto de Stevin "faça a indução pra
você" (calculando os termos \sum \prod z_i que vão aparecer como
coeficientes dos monômios z^k), mas é "quase" como se você estivesse
empurrando a indução um andar abaixo ;-)


Israel: qual a demonstração por indução que você conhece?  E porque
você gostaria de outra??

Abraços,
-- 
Bernardo Freitas Paulo da Costa

=
Instru��es para entrar na lista, sair da lista e usar a lista em
http://www.mat.puc-rio.br/~obmlistas/obm-l.html
=


[obm-l] Re: [obm-l] Re: [obm-l] Re: [obm-l] Teoria dos Números

2020-10-26 Por tôpico Otávio Araújo
De nada mano.

Em seg, 26 de out de 2020 09:40, joao pedro b menezes <
joaopedrobmene...@gmail.com> escreveu:

> Muito obrigado pela ajuda! Entendi o exercício agora.
>
> Em dom, 25 de out de 2020 às 19:59, Otávio Araújo <
> otavio17.ara...@gmail.com> escreveu:
>
>> Vc resolve essa questão mostrando q p=n^2+n+1.  Se n=1 acabou. Se n>1,Já
>> que p divide n^3-1 e é primo, temos que p divide n-1 ou n^2+n+1. Não
>> podemos ter p dividindo n-1 pois n divide p-1 -> n<= p-1 n-1> Portanto p divide n^2+n+1. Faca n^2+n+1 = kp, k inteiro positivo. Temos que
>> kp=n^2+n+1 é congruente a 1 módulo n. Do enunciado temos p congruente a 1
>> módulo n,  mas p é congruente a 1 módulo n e é diferente de 1(pois é primo)
>> -> p>= n+1 e  k será congruente a 1 módulo n também. Suponha que k>1,
>> k>1 implica k>= n+1 daí kp>=(n+1)^2 > n^2+n+1, contradição. Portanto
>> k=1 e p=n^2+n+1.
>>
>> Em dom, 25 de out de 2020 17:37, joao pedro b menezes <
>> joaopedrobmene...@gmail.com> escreveu:
>>
>>> Olá, boa tarde.
>>> Estou com dúvida nesse exercício:
>>> " Sejam n um inteiro positivo maior que 1 e p um primo positivo tal que
>>> n divide p − 1 e p divide n 3 − 1. Mostre que 4p − 3 ´e um quadrado
>>> perfeito."
>>> Já agradeço pela ajuda e pelo tempo!
>>>
>>>


[obm-l] Re: [obm-l] Re: [obm-l] Teoria dos Números

2020-10-26 Por tôpico joao pedro b menezes
Muito obrigado pela ajuda! Entendi o exercício agora.

Em dom, 25 de out de 2020 às 19:59, Otávio Araújo 
escreveu:

> Vc resolve essa questão mostrando q p=n^2+n+1.  Se n=1 acabou. Se n>1,Já
> que p divide n^3-1 e é primo, temos que p divide n-1 ou n^2+n+1. Não
> podemos ter p dividindo n-1 pois n divide p-1 -> n<= p-1 n-1 Portanto p divide n^2+n+1. Faca n^2+n+1 = kp, k inteiro positivo. Temos que
> kp=n^2+n+1 é congruente a 1 módulo n. Do enunciado temos p congruente a 1
> módulo n,  mas p é congruente a 1 módulo n e é diferente de 1(pois é primo)
> -> p>= n+1 e  k será congruente a 1 módulo n também. Suponha que k>1,
> k>1 implica k>= n+1 daí kp>=(n+1)^2 > n^2+n+1, contradição. Portanto  k=1
> e p=n^2+n+1.
>
> Em dom, 25 de out de 2020 17:37, joao pedro b menezes <
> joaopedrobmene...@gmail.com> escreveu:
>
>> Olá, boa tarde.
>> Estou com dúvida nesse exercício:
>> " Sejam n um inteiro positivo maior que 1 e p um primo positivo tal que n
>> divide p − 1 e p divide n 3 − 1. Mostre que 4p − 3 ´e um quadrado perfeito."
>> Já agradeço pela ajuda e pelo tempo!
>>
>>


[obm-l] Re: [obm-l] Re: [obm-l] Re: [obm-l] Re: [obm-l] Ajuda em teoria dos números

2020-10-22 Por tôpico Pedro José
Boa tarde!
Na verdade: 2^a=64; a= 6 e y=12.

Em qui., 22 de out. de 2020 às 11:17, Pedro José 
escreveu:

> Bom dia!
> Recebi esse problema hoje: 615 + x^2 = 2^y., para x,y inteiros Não saberia
> fazer, como não soube resolver esse, acima. Mas devido a solução do colega
> Esdras, pensei:"já vi algo parecido".
> Basta restringir y aos pares.
> Se y é ímpar x^2=2 mod3, absurdo então y é par. Logo y=2a, com a inteiro.
> (2^a + x) (2^a-x)= 615= 1*615=3*205=5*123=15*41 e como a soma dos fatores
> necessita ser uma potência de 2, só serve para 123 e 5.
> Logo 2^y=64 e y=6 e x= 59 ou x=-59.
> Uma resolução levou a outra, não pelo talento nato, mas por aprendizado, o
> que é válido.
> Teve uma feita que estava tentando provar que o triângulo órtico, era o
> triângulo de menor perímetro que poderia ser inscrito em um triângulo
> acutângulo. Tentei por geometria analítica e só levando tinta. Tinha
> desistido. Quando me deparei com um problema que não consegui resolver, mas
> que tinha um caminho para a solução. Quando vi o rebatimento feito, pensei
> está resolvido. O curioso, é que, quando desisti, pesquisei na internet e
> não achei nada. Depois que consegui resolver, achei duas soluções, e
> infelizmente e como esperado, a minha não era novidade, era clássica.
> Obrigado, Esdras! Sem a sua solução, certamente, não teria resolvido essa
> última questão.
>
> Cordialmente,
> PJMS
>
> Em sex., 24 de jul. de 2020 às 12:19, Prof. Douglas Oliveira <
> profdouglaso.del...@gmail.com> escreveu:
>
>> Obrigado Claudio e Esdras, fatoração show
>>
>>
>> Em sex., 24 de jul. de 2020 às 11:12, Esdras Muniz <
>> esdrasmunizm...@gmail.com> escreveu:
>>
>>> Se for solução inteira positiva, acho que só tem 3 e 4. Vc supõe spdg x
>>> maior ou igual a y, vê que y=1 não tem solução e x=y tb não. Daí, x>y>1.
>>> Fatorando a expressão, fica: (xy-8-(x-y))(xy-8+(x-y))=15. Como
>>> (xy-8-(x-y))>(xy-8+(x-y))>-2. Temos que ou (xy-8-(x-y))=1 e (xy-8+(x-y))=15,
>>> o que não tem soluções inteiras positivas, ou (xy-8-(x-y))=3 e 
>>> (xy-8+(x-y))=5,
>>> cujas únicas soluções inteiras são x=4 e y=3.
>>>
>>> Em sex, 24 de jul de 2020 10:36, Claudio Buffara <
>>> claudio.buff...@gmail.com> escreveu:
>>>
 Pelo que entendi, a solução é a porção dessa curva algébrica situada no
 1o quadrante.
 Dá pra fazer isso no Wolfram Alpha, com o comando plot (x*y-7)^2 - x^2
 - y^2 = 0.

 []s,
 Claudio.

 On Fri, Jul 24, 2020 at 9:58 AM Prof. Douglas Oliveira <
 profdouglaso.del...@gmail.com> wrote:

> Preciso de ajuda para encontrar todas as soluções não negativas da
> equação
> (xy-7)^2=x^2+y^2.
>
> Desde já agradeço a ajuda
> Douglas Oliveira
>
> --
> Esta mensagem foi verificada pelo sistema de antivírus e
> acredita-se estar livre de perigo.


 --
 Esta mensagem foi verificada pelo sistema de antivírus e
 acredita-se estar livre de perigo.
>>>
>>>
>>> --
>>> Esta mensagem foi verificada pelo sistema de antivírus e
>>> acredita-se estar livre de perigo.
>>
>>
>> --
>> Esta mensagem foi verificada pelo sistema de antivírus e
>> acredita-se estar livre de perigo.
>
>


[obm-l] Re: [obm-l] Re: [obm-l] Re: [obm-l] Re: [obm-l] Ajuda em teoria dos números

2020-10-22 Por tôpico Pedro José
Bom dia!
Recebi esse problema hoje: 615 + x^2 = 2^y., para x,y inteiros Não saberia
fazer, como não soube resolver esse, acima. Mas devido a solução do colega
Esdras, pensei:"já vi algo parecido".
Basta restringir y aos pares.
Se y é ímpar x^2=2 mod3, absurdo então y é par. Logo y=2a, com a inteiro.
(2^a + x) (2^a-x)= 615= 1*615=3*205=5*123=15*41 e como a soma dos fatores
necessita ser uma potência de 2, só serve para 123 e 5.
Logo 2^y=64 e y=6 e x= 59 ou x=-59.
Uma resolução levou a outra, não pelo talento nato, mas por aprendizado, o
que é válido.
Teve uma feita que estava tentando provar que o triângulo órtico, era o
triângulo de menor perímetro que poderia ser inscrito em um triângulo
acutângulo. Tentei por geometria analítica e só levando tinta. Tinha
desistido. Quando me deparei com um problema que não consegui resolver, mas
que tinha um caminho para a solução. Quando vi o rebatimento feito, pensei
está resolvido. O curioso, é que, quando desisti, pesquisei na internet e
não achei nada. Depois que consegui resolver, achei duas soluções, e
infelizmente e como esperado, a minha não era novidade, era clássica.
Obrigado, Esdras! Sem a sua solução, certamente, não teria resolvido essa
última questão.

Cordialmente,
PJMS

Em sex., 24 de jul. de 2020 às 12:19, Prof. Douglas Oliveira <
profdouglaso.del...@gmail.com> escreveu:

> Obrigado Claudio e Esdras, fatoração show
>
>
> Em sex., 24 de jul. de 2020 às 11:12, Esdras Muniz <
> esdrasmunizm...@gmail.com> escreveu:
>
>> Se for solução inteira positiva, acho que só tem 3 e 4. Vc supõe spdg x
>> maior ou igual a y, vê que y=1 não tem solução e x=y tb não. Daí, x>y>1.
>> Fatorando a expressão, fica: (xy-8-(x-y))(xy-8+(x-y))=15. Como
>> (xy-8-(x-y))>(xy-8+(x-y))>-2. Temos que ou (xy-8-(x-y))=1 e (xy-8+(x-y))=15,
>> o que não tem soluções inteiras positivas, ou (xy-8-(x-y))=3 e 
>> (xy-8+(x-y))=5,
>> cujas únicas soluções inteiras são x=4 e y=3.
>>
>> Em sex, 24 de jul de 2020 10:36, Claudio Buffara <
>> claudio.buff...@gmail.com> escreveu:
>>
>>> Pelo que entendi, a solução é a porção dessa curva algébrica situada no
>>> 1o quadrante.
>>> Dá pra fazer isso no Wolfram Alpha, com o comando plot (x*y-7)^2 - x^2 -
>>> y^2 = 0.
>>>
>>> []s,
>>> Claudio.
>>>
>>> On Fri, Jul 24, 2020 at 9:58 AM Prof. Douglas Oliveira <
>>> profdouglaso.del...@gmail.com> wrote:
>>>
 Preciso de ajuda para encontrar todas as soluções não negativas da
 equação
 (xy-7)^2=x^2+y^2.

 Desde já agradeço a ajuda
 Douglas Oliveira

 --
 Esta mensagem foi verificada pelo sistema de antivírus e
 acredita-se estar livre de perigo.
>>>
>>>
>>> --
>>> Esta mensagem foi verificada pelo sistema de antivírus e
>>> acredita-se estar livre de perigo.
>>
>>
>> --
>> Esta mensagem foi verificada pelo sistema de antivírus e
>> acredita-se estar livre de perigo.
>
>
> --
> Esta mensagem foi verificada pelo sistema de antivírus e
> acredita-se estar livre de perigo.


[obm-l] Re: [obm-l] Re: [obm-l] Animação do site da OBM

2020-10-19 Por tôpico Anderson Torres
O algoritmo de animação não está exatamente disponível, mas o artigo da OBM
sobre o Porisma de Steiner explica bem a sua ideia: invertendo um par de
círculos concêntricos, é possível produzir qualquer configuração de Steiner.

Em sáb., 17 de out. de 2020 às 15:41, Leonardo Borges Avelino <
lbor...@gmail.com> escreveu:

> Trata-se do tema de inversão e tem um artigo na Eureka 4
> https://www.obm.org.br/content/uploads/2017/01/eureka4.pdf
>
> Abs
>
> On Sat, Oct 17, 2020 at 3:14 PM Felippe Coulbert Balbi <
> felippeba...@hotmail.com> wrote:
>
>> A muitos anos atras durante um coloquio de matemática no IMPA, no grupo
>> de olimpíada, estavam resolvendo um problema se não me engano da IMO e
>> durante a resolução houve um comentário dessa resolução com a animação do
>> logo no site da OBM.
>>
>> Eu estou fazendo um programa que tem haver com vídeos e queria colocar
>> essa animação no meu programa. Eu procurei meus materiais antigos sobre o
>> algoritmo para gerar ele mas não achei nada.
>>
>> Alguém consegue dar uma força?
>>
>> Até,
>> Felippe
>>
>>
>> 
>>  Livre
>> de vírus. www.avg.com
>> .
>>
>> <#m_-2163421921828132999_m_-8807507600612609615_DAB4FAD8-2DD7-40BB-A1B8-4E2AA1F9FDF2>
>>
>


[obm-l] Re: [obm-l] Re: É um número?

2020-08-27 Por tôpico Esdras Muniz
Para um número n natural, podemos definir a^n como a.a.a...a n vezes. Se
a!=0, a^(-n)=(1/a)^n. E a^(1/m) como o real b tal que b^m=a. Como esse b
nem sempre existe, devemos tomar um certo cuidado. Só não vai ter solução
se a<0 e m for par (é fácil mostrar isso usando polinômios). Daí, seguindo
essa linha de raciocínio, podemos definir naturalmente a^(n/m). Agora
podemos tentar generalizar de forma que a função f(x)=a^x seja contínua.
Daí, como o conjunto dos racionais é denso nós reais e, da forma como foi
definida, essa função definida para x racional é monótona, temos a^x
definido também para x irracional. Mas isso custa um preço, não podemos
mais tomar a negativo. Por exemplo, você poderia dizer que (-1)^(1/3) =-1,
pois (-1)(-1)(-1)=-1. Mas assim, se essa f é contínua, o limite quando n
vai pro infinito de (-1)^((n)/(3n+1)) deveria ser -1, mas esse limite nem
mesmo existe.

Em qui, 27 de ago de 2020 20:00, Maikel Andril Marcelino <
maikel.marcel...@ifrn.edu.br> escreveu:

>
> Marcone, qual das duas opções a < 0 ou x pertencente aos irracionais? Ou
> as duas opções juntas?
>
>
> Atenciosamente,
>
> *Maikel Andril Marcelino*
> *(84) 9-9149-8991 (Contato)*
> *(84) 8851-3451 (WhatsApp)*
>
> --
> *De:* owner-ob...@mat.puc-rio.br  em nome de
> marcone augusto araújo borges 
> *Enviado:* quinta-feira, 27 de agosto de 2020 18:14
> *Para:* obm-l@mat.puc-rio.br
> *Assunto:* [obm-l] É um número?
>
> Faz sentido a^x, se a< 0 e x é irracional positivo?
> --
> Esta mensagem foi verificada pelo sistema de antivírus e
> acredita-se estar livre de perigo.
>
> --
> Esta mensagem foi verificada pelo sistema de antivírus e
> acredita-se estar livre de perigo.
>

-- 
Esta mensagem foi verificada pelo sistema de antiv�rus e
 acredita-se estar livre de perigo.



[obm-l] Re: [obm-l] Re: [obm-l] Re: [obm-l] Sequência das médias ponderadas

2020-08-27 Por tôpico Artur Costa Steiner
Será que isso vale se (a_n) tiver termos negativos? Me parece que sim

Artur

Em qua, 26 de ago de 2020 21:55, Esdras Muniz 
escreveu:

> Dado e>0, existe n0 tq m>=n0 então a-e
> Sn= c+(am+...+an)/(p1+...+pn)
>
> Daí:
>
>
> c+a(pm+...+pn)/(p1+...+pn) -e
> Daí, fixando m e mandando n pro infinito, c vai pra zero e 
> (pm+...+pn)/(p1+...+pn)
> vai pra 1. Então o limite de Sn é a.
>
>
> Em qua, 26 de ago de 2020 20:19, Claudio Buffara <
> claudio.buff...@gmail.com> escreveu:
>
>> Acho que isso tá mal formulado.
>> Por exemplo,quanto é s_3?
>>
>> On Tue, Aug 25, 2020 at 3:49 PM Artur Costa Steiner <
>> artur.costa.stei...@gmail.com> wrote:
>>
>>> Isso me foi dado como verdadeiro, mas ainda não cheguei a uma conclusão.
>>>
>>> Sejam (a_ n) uma sequência de reais positivos e (s_n) a sequência das
>>> médias ponderadas de (a_n,) com relação aos pesos positivos (p_n).
>>> Suponhamos que lim p_n = p, 0 < p < oo, e que a sequência das médias
>>> aritméticas de (a_n) convirja para o real a. Então, s_n --> a.
>>>
>>> Abraços
>>> Artur
>>>
>>> --
>>> Esta mensagem foi verificada pelo sistema de antivírus e
>>> acredita-se estar livre de perigo.
>>
>>
>> --
>> Esta mensagem foi verificada pelo sistema de antivírus e
>> acredita-se estar livre de perigo.
>
>
> --
> Esta mensagem foi verificada pelo sistema de antivírus e
> acredita-se estar livre de perigo.

-- 
Esta mensagem foi verificada pelo sistema de antiv�rus e
 acredita-se estar livre de perigo.



[obm-l] Re: [obm-l] Re: É um número?

2020-08-27 Por tôpico Artur Costa Steiner
Por exemplo, i^i = e^(i Ln(i)), sendo Ln o log principal de i. Como o
argumento principal de i é pi/2, então Ln(i) = ln(1) + i pi/2. = i pi/2.
Logo, i^i = e^(-pi/2), um número real

(-1)^raiz(2) = e^(raiz(2) Ln(-1)). Como o argumento principal de -1 é pi,
Ln(-1) = ln(1) + I pi = i pi. Assim, ,(-1)^(raiz(2) = e^(raiz(2) pi i) =
cos( pi raiz(2)) + i sen(pi raiz(2))  um complexo não real.

Artur




Em qui, 27 de ago de 2020 20:36, Artur Costa Steiner <
artur.costa.stei...@gmail.com> escreveu:

> Corrigindo, o que eu quis dizer é que, na fórmula dada, ln(r) é o único
> log real de r.
>
> Artur
>
> Em qui, 27 de ago de 2020 20:32, Artur Costa Steiner <
> artur.costa.stei...@gmail.com> escreveu:
>
>> Quando vc tem base negativa, entramos no domínio complexo. De modo geral,
>> se u não nulo e v são números complexos, então define-se u^v por u^v = e^(v
>> ln(u)),
>>
>> Todo complexo não nulo tem uma infinidade de logaritmos, por isso
>> costuma-se escolher o chamado logaritmo principal, que está associado ao
>> argumento principal. Se u é um complexo não nulo de valor absoluto r.
>> então, para cada argumento a de u, ln(,r) + ia é um log de u. Na definição
>> de u^v, escolhe-se o argumento principal. ln(r) é o único log real de u.
>>
>> Logs de números que não sejam reais positivos é um assunto um tanto
>> complicado. Sr vc tiver interesse nisso, que é muito bonito, estude análise
>> complexa. Mas respondendo objetivamente a sua pergunta, sim, faz sentido
>> sim no domínio complexo.
>>
>> Artur
>>
>>
>> Em qui, 27 de ago de 2020 20:00, Maikel Andril Marcelino <
>> maikel.marcel...@ifrn.edu.br> escreveu:
>>
>>>
>>> Marcone, qual das duas opções a < 0 ou x pertencente aos irracionais? Ou
>>> as duas opções juntas?
>>>
>>>
>>> Atenciosamente,
>>>
>>> *Maikel Andril Marcelino*
>>> *(84) 9-9149-8991 (Contato)*
>>> *(84) 8851-3451 (WhatsApp)*
>>>
>>> --
>>> *De:* owner-ob...@mat.puc-rio.br  em nome
>>> de marcone augusto araújo borges 
>>> *Enviado:* quinta-feira, 27 de agosto de 2020 18:14
>>> *Para:* obm-l@mat.puc-rio.br
>>> *Assunto:* [obm-l] É um número?
>>>
>>> Faz sentido a^x, se a< 0 e x é irracional positivo?
>>> --
>>> Esta mensagem foi verificada pelo sistema de antivírus e
>>> acredita-se estar livre de perigo.
>>>
>>> --
>>> Esta mensagem foi verificada pelo sistema de antivírus e
>>> acredita-se estar livre de perigo.
>>>
>>

-- 
Esta mensagem foi verificada pelo sistema de antiv�rus e
 acredita-se estar livre de perigo.



[obm-l] Re: [obm-l] Re: É um número?

2020-08-27 Por tôpico Artur Costa Steiner
Corrigindo, o que eu quis dizer é que, na fórmula dada, ln(r) é o único log
real de r.

Artur

Em qui, 27 de ago de 2020 20:32, Artur Costa Steiner <
artur.costa.stei...@gmail.com> escreveu:

> Quando vc tem base negativa, entramos no domínio complexo. De modo geral,
> se u não nulo e v são números complexos, então define-se u^v por u^v = e^(v
> ln(u)),
>
> Todo complexo não nulo tem uma infinidade de logaritmos, por isso
> costuma-se escolher o chamado logaritmo principal, que está associado ao
> argumento principal. Se u é um complexo não nulo de valor absoluto r.
> então, para cada argumento a de u, ln(,r) + ia é um log de u. Na definição
> de u^v, escolhe-se o argumento principal. ln(r) é o único log real de u.
>
> Logs de números que não sejam reais positivos é um assunto um tanto
> complicado. Sr vc tiver interesse nisso, que é muito bonito, estude análise
> complexa. Mas respondendo objetivamente a sua pergunta, sim, faz sentido
> sim no domínio complexo.
>
> Artur
>
>
> Em qui, 27 de ago de 2020 20:00, Maikel Andril Marcelino <
> maikel.marcel...@ifrn.edu.br> escreveu:
>
>>
>> Marcone, qual das duas opções a < 0 ou x pertencente aos irracionais? Ou
>> as duas opções juntas?
>>
>>
>> Atenciosamente,
>>
>> *Maikel Andril Marcelino*
>> *(84) 9-9149-8991 (Contato)*
>> *(84) 8851-3451 (WhatsApp)*
>>
>> --
>> *De:* owner-ob...@mat.puc-rio.br  em nome de
>> marcone augusto araújo borges 
>> *Enviado:* quinta-feira, 27 de agosto de 2020 18:14
>> *Para:* obm-l@mat.puc-rio.br
>> *Assunto:* [obm-l] É um número?
>>
>> Faz sentido a^x, se a< 0 e x é irracional positivo?
>> --
>> Esta mensagem foi verificada pelo sistema de antivírus e
>> acredita-se estar livre de perigo.
>>
>> --
>> Esta mensagem foi verificada pelo sistema de antivírus e
>> acredita-se estar livre de perigo.
>>
>

-- 
Esta mensagem foi verificada pelo sistema de antiv�rus e
 acredita-se estar livre de perigo.



[obm-l] Re: [obm-l] Re: É um número?

2020-08-27 Por tôpico Artur Costa Steiner
Quando vc tem base negativa, entramos no domínio complexo. De modo geral,
se u não nulo e v são números complexos, então define-se u^v por u^v = e^(v
ln(u)),

Todo complexo não nulo tem uma infinidade de logaritmos, por isso
costuma-se escolher o chamado logaritmo principal, que está associado ao
argumento principal. Se u é um complexo não nulo de valor absoluto r.
então, para cada argumento a de u, ln(,r) + ia é um log de u. Na definição
de u^v, escolhe-se o argumento principal. ln(r) é o único log real de u.

Logs de números que não sejam reais positivos é um assunto um tanto
complicado. Sr vc tiver interesse nisso, que é muito bonito, estude análise
complexa. Mas respondendo objetivamente a sua pergunta, sim, faz sentido
sim no domínio complexo.

Artur


Em qui, 27 de ago de 2020 20:00, Maikel Andril Marcelino <
maikel.marcel...@ifrn.edu.br> escreveu:

>
> Marcone, qual das duas opções a < 0 ou x pertencente aos irracionais? Ou
> as duas opções juntas?
>
>
> Atenciosamente,
>
> *Maikel Andril Marcelino*
> *(84) 9-9149-8991 (Contato)*
> *(84) 8851-3451 (WhatsApp)*
>
> --
> *De:* owner-ob...@mat.puc-rio.br  em nome de
> marcone augusto araújo borges 
> *Enviado:* quinta-feira, 27 de agosto de 2020 18:14
> *Para:* obm-l@mat.puc-rio.br
> *Assunto:* [obm-l] É um número?
>
> Faz sentido a^x, se a< 0 e x é irracional positivo?
> --
> Esta mensagem foi verificada pelo sistema de antivírus e
> acredita-se estar livre de perigo.
>
> --
> Esta mensagem foi verificada pelo sistema de antivírus e
> acredita-se estar livre de perigo.
>

-- 
Esta mensagem foi verificada pelo sistema de antiv�rus e
 acredita-se estar livre de perigo.



[obm-l] Re: [obm-l] Re: [obm-l] Re: [obm-l] Re: [obm-l] Re: [obm-l] Re: [obm-l] Geometria plana com desigualdade de médias?

2020-08-27 Por tôpico Anderson Torres
Em qua., 26 de ago. de 2020 às 18:29, Pedro José  escreveu:
>
> Boa noite!
> Anderson,
> achei legal a sua visão. Mas não consegui evoluir com nada.
> Todavia, fiquei com uma dúvida. Como x+y é um dos ângulos do triângulo temos 
> a restrição 0 E entendo que tanto para cotg(x) + cot(y) , como para tg(x) + tg(y) ocorrerá 
> um mínimo em x=y=K/2, onde x+y=k,k sendo um constante.
> Não acompanhei a sua dedução d quando um é mínimo o outro é máximo.

Eu não fui muito claro.

Você converteu o problema em "calcule o valor mínimo de cot(x)+cot(y)
com x+y fixo". Isso é essencialmente o mesmo que resolver o problema
"calcule o valor mínimo de tan(a)+tan(b) com a+b fixo" - pois sabendo
resolver um é só usar a mesma solução para x=90-a, y=90-b.

>
> Saudações,
> PJMS
>
> Em qui., 20 de ago. de 2020 às 22:40, Anderson Torres 
>  escreveu:
>>
>> Em qui., 20 de ago. de 2020 às 22:03, Anderson Torres
>>  escreveu:
>> >
>> > Em ter., 18 de ago. de 2020 às 19:51, Pedro José  
>> > escreveu:
>> > >
>> > > Boa noite!
>> > > Cláudio,
>> > > não consegui nada geométrico.
>> > > O máximo que atingi foi:
>> > > a/ha + b/hb + c/hc= [cotg(A1) +cotg (A2)]  + [cotg(B1) +cotg (B2)] + 
>> > > co[tg(C1) +cotg (C2)] com A1 + A2 = A; B1 + B2 + B e C1 + C2 = C.
>> > > Para ser mínimo cada termo entre colchetes deve ser mínimo, o que ocorre 
>> > > quando A1 = A2; B1 = B2 e C1 = C2. Logo P seria o encontro das 
>> > > bissetrizes e logo I.
>> > > Onde: A1= PAB e A2=PAC; B1=PBA e B2=PBC; C1=PCA e C2=PCB.
>> >
>> > Acho que daqui poderia sair uma interpretação mais escamoteada.
>> > Afinal, trigonometria é uma espécie de "ponto de contato" entre a
>> > geometria analítica e a sintética, entre a nuvem de desenhos e a de
>> > números.
>> >
>> > Acredito que a solução aqui seria arranjar uma interpretação
>> > geométrica desses colchetes de co-tangentes. Acredito que possamos
>> > apelar para Ptolomeu em algum momento ou para um macete de
>> > semelhanças, pois as projeções de um ponto sobre duas retas criam um
>> > quadrilátero cíclico.
>>
>> Acrescentando mais coisas: se queremos minimizar cot(x) +cot(y) com
>> x+y fixo, isto é equivalente a minimizar tan(90-x)+tan(90-y) com
>> 90-x+90-y fixo. Ou como maximizar tan(x) + tan(y) com x+y fixo.
>>
>> Geometricamente, tangente é cateto oposto dividido por cateto
>> adjacente. Logo uma soma de tangentes com catetos adjacentes iguais
>> equivale a uma soma de catetos opostos! Assim sendo, nosso problema
>> pode ser pensado da seguinte forma:
>>
>> Dados um ponto A e uma reta d fixos, temos que construir duas retas x
>> e y, com ângulo 'alfa' entre elas, ambas passando por A e tais que a
>> distância entre os pontos X e Y, que elas geram ao intersectar d, seja
>> mínima.
>>
>> Daí fica fácil argumentar que a altura por A também tem que ser a
>> bissetriz por A.
>>
>> No fundo do fundo é uma forma de geometrizar a solução trigonométrica.
>> A trigonometria se torna apenas um atalho.
>>
>> Vou formalizar isso mais tarde, com desenhos e tudo.
>>
>>
>>
>> >
>> > Isso até me lembra o famoso artigo do Shine sobre geometria cearense
>> > VS geometria paulista:
>> > https://cyshine.webs.com/geometria-2005.pdf
>> >
>> >
>> > >
>> > > Saudações,
>> > > PJMS
>> > >
>> > > Em ter., 18 de ago. de 2020 às 11:34, Claudio Buffara 
>> > >  escreveu:
>> > >>
>> > >> Será que tem uma demonstração mais geométrica e menos algébrica disso? 
>> > >> E que torne o resultado mais intuitivo?
>> > >> É razoável que o ponto P não esteja muito próximo de qualquer dos 
>> > >> lados, pois neste caso, se P se aproximasse do lado a, por exemplo, 
>> > >> a/h_a cresceria e a expressão se afastaria do valor mínimo.
>> > >> Mas, com lados não necessariamente congruentes, não é óbvio, a priori, 
>> > >> que P deva ser equidistante dos três.
>> > >> De fato, seria razoável esperar que P estivesse mais próximo do maior 
>> > >> lado e conjecturar, por exemplo, que o P que minimiza a expressão é tal 
>> > >> que a/h_a = b/h_b = c/h_c.
>> > >> O fato de P ser o incentro não me parece a conjectura mais evidente 
>> > >> neste caso.
>> > >>
>> > >>
>> > >> On Sun, Aug 16, 2020 at 10:11 AM Matheus Secco  
>> > >> wrote:
>> > >>>
>> > >>> Olá, Vanderlei.
>> > >>> Por Cauchy-Schwarz, temos
>> > >>>
>> > >>> (a/ha + b/hb + c/hc) * (a*ha + b*hb + c*hc) >= (a+b+c)^2.  (#)
>> > >>>
>> > >>> Como (a*ha + b*hb + c*hc) = 2S, onde S é a área de ABC, segue que a 
>> > >>> expressão a/ha + b/hb + c/hc é pelo menos 2p^2/S, onde p é o 
>> > >>> semi-perimetro.
>> > >>>
>> > >>> Por outro lado, a igualdade em (#) ocorre se, e somente se, ha = hb = 
>> > >>> hc, ou seja, quando P é o incentro do triângulo
>> > >>>
>> > >>> Abraços,
>> > >>> Matheus
>> > >>>
>> > >>> Em dom, 16 de ago de 2020 08:59, Professor Vanderlei Nemitz 
>> > >>>  escreveu:
>> > 
>> >  Bom dia!
>> > 
>> >  Tentei utilizar alguma desigualdade de médias aqui, mas não tive 
>> >  êxito. Alguém ajuda?
>> >  Muito agradecido!
>> > 
>> >  Seja P um ponto no 

[obm-l] Re: [obm-l] Re: [obm-l] Sequência das médias ponderadas

2020-08-26 Por tôpico Esdras Muniz
Dado e>0, existe n0 tq m>=n0 então a-e
escreveu:

> Acho que isso tá mal formulado.
> Por exemplo,quanto é s_3?
>
> On Tue, Aug 25, 2020 at 3:49 PM Artur Costa Steiner <
> artur.costa.stei...@gmail.com> wrote:
>
>> Isso me foi dado como verdadeiro, mas ainda não cheguei a uma conclusão.
>>
>> Sejam (a_ n) uma sequência de reais positivos e (s_n) a sequência das
>> médias ponderadas de (a_n,) com relação aos pesos positivos (p_n).
>> Suponhamos que lim p_n = p, 0 < p < oo, e que a sequência das médias
>> aritméticas de (a_n) convirja para o real a. Então, s_n --> a.
>>
>> Abraços
>> Artur
>>
>> --
>> Esta mensagem foi verificada pelo sistema de antivírus e
>> acredita-se estar livre de perigo.
>
>
> --
> Esta mensagem foi verificada pelo sistema de antivírus e
> acredita-se estar livre de perigo.

-- 
Esta mensagem foi verificada pelo sistema de antiv�rus e
 acredita-se estar livre de perigo.



  1   2   3   4   5   6   7   8   9   10   >